0% found this document useful (0 votes)
2K views

Step 2 CK NBME 6 (Q)

A 18-year-old woman comes in for a routine checkup. She has regular periods and was sexually active with one partner using condoms inconsistently. She has a family history of breast cancer, colon cancer, and heart disease. Her examination shows no abnormalities. The most appropriate screening test for this patient would be testing for Neisseria gonorrhoeae and Chlamydia trachomatis given her sexual activity.

Uploaded by

M. Baidar Saeed
Copyright
© © All Rights Reserved
We take content rights seriously. If you suspect this is your content, claim it here.
Available Formats
Download as PDF, TXT or read online on Scribd
0% found this document useful (0 votes)
2K views

Step 2 CK NBME 6 (Q)

A 18-year-old woman comes in for a routine checkup. She has regular periods and was sexually active with one partner using condoms inconsistently. She has a family history of breast cancer, colon cancer, and heart disease. Her examination shows no abnormalities. The most appropriate screening test for this patient would be testing for Neisseria gonorrhoeae and Chlamydia trachomatis given her sexual activity.

Uploaded by

M. Baidar Saeed
Copyright
© © All Rights Reserved
We take content rights seriously. If you suspect this is your content, claim it here.
Available Formats
Download as PDF, TXT or read online on Scribd
You are on page 1/ 184

I I I'

I 1:'
I ' ' I I
1. A healthy18-year-old woman comes for a routine health maintenanceexamination.Menses occur at regular 28-day intervals.Her last menstrualperiod was 2 weeks·ago. S~ is sexually
active with one partner, and she and her partner use condoms inconsistentlyfor contraception.Her maternal grandmotherwas diagnosed with breast canc~r at the a,geof 65 year~.her
paternal ~r~ndfath~rwas diagnos~d with colon cancer at the age of 72 years, and her maternal grandfatherdied of a myocardial infarction at t~e age of-69.years. Examinati~_n s~ows·~o.
abnormalities. Which of the following 1sthe most appropriate screening test for this patient? ' • ,: , ,. .); ;· :i
' l1' I \\1ti' l11' i ·\ •.. ''! ,11
O A) Complete blood count ·11 'i \. \• I·'l i! ; l; • 1' 1\}'.
" I
,:.;_:,,
:'• :• • •
•• •
0 B) Fasting serum lipid studies ·! \ • l ,, ' • '
1 1
i
,,
0 C) Test of the stool for occult blood I j II I I
I l' '
' ll •
0 D) Testing for Neisseria gonorrhoeae and Chlamydia trachomatis I I
' Ji I'
'.
I',

0 E) Urinalysis I
' ,I . "
' '' •l I
1
;•' I 11
l
,.
' ,I
I h
; ·\ I \ '
I l ' .
I
I l i11
. .'
I \'
I I I \· I' ,I
I

.,
I : I I
I l' 11I
'' ' ..
'

r
' ' I ,
-:
.,
\: I
,
• ,
I
I'
,, I !

" •IIi. • I
•I • '
'

.
.. ....
l:-

,I ,1
I


:1 I I ill !ll HI II
1
I
'\1
,, l
1
1 -: .I
''I ' '
i'
, I
l •I
l

'
I
I
2. A 14-month-oldgirl is brought to the physician because of a 14-hour history of irritability and episodes of drawing her knees toward her chest. Du'ringthis p.eriod,she has vomited
nonbilious fluid twice and had a bowel movement containing a sn,all amount of blood. She had an upper respiratorytract infection 2 weeks ago. She is listless except for intermittent
episodes of discomfort. Her temperature is 38°C (100.4°F). Abdominal exan1inationshows right-sided tenderness without guarding or rebound; bowel ~ourds are present.Rectal
examination shows bright red blood and mucus. An x-ray of the abdomen shows no abnormalities. Which of the following is the most appropriate nef s~epin,managem~nt?

O A) X-ray of the upper gastrointestinal tract with contrast \ \ I ,l \


O B) Water-soluble contrast enema
O C) Corticosteroid enemas . I
O D) Admission to the hospital for total parenteral nutrition
O E) Immediate laparotomy

'I

Il I
1

'I

.,
itI, I
I I
I
II11
11
1_, I
C
I
,J
l
,11
i,
I I \1\
ij

I I
I'
I I
'I 'I
1I
I
'
I I
1

.' .
'
l

I • ..
rl!

• '~ - -· ..
f~~pons~-
.. ,.. . - l,1
the,next 2 it~ms ~re the ·sa~e. Select one answer for each item in the set.
·<j>pti:o'ns;for . ,
,::•r·· ·t 1 1I.~'"•' , + ' I

For each patient with cognitive impairment, select the most likely diagnosis.

O A) Acute stress disorder Q I) Major depressive disorder


0 B) Dementia, Alzheimer's type 0 J) Multi-infarct(vascular) dementia
0 C) Dissociative amnesia 0 K) Niacin deficiency
0 0) General paresis 0 L) Normal-pressure hydrocephalus 1\
0 E) Head trauma 0 M) Parkinson's disease
0 F) Hepatolenticulardegeneration (Wilson's disease) 0 N) Pick's disease
0 G) HIV encephalitis 0 0) Schizophrenia, catatonic type
0 H) Huntington'sdisease 0 P) Norn,al aging

3. An 82-year-old woman is brought to the physician by her granddaughter because of a 6-week history of increasing forgetfulness. She is a retired schoolteacher and lives independently.
Her granddaughter is concerned because on several occasions she has left the stove on when she went to bed, During conversations with her granddaughter, she has difficulty
remembering past events and seems unconcerned about her memory lapses. The patient describes trouble sleeping through the night and has had a decreased appetite resulting in a
4.5-kg (10-lb) weight loss over the past month. She has a history of similar sympton,s 2 and 5 years ago _thatwere successfullytreated with medication. She appears unkempt and has
poor personal hygiene. Her temperature is 37°C (98.6°F), blood pressure is 110/70 mm 'Hg, and pulse is 80/niin and regular. Mental status examination sho~s psychomotor retardation, a
flat affect, impaired ability to recall past events, and trouble repeating three numbers in seque_nce.She is unable to recall the names of recent presidents. Her serum urea nitrogen
concentrationis 25 mg/dl, and serum creatinine concentration is 1.7 n,g/dl. '1'
11

'\ I I
l I I I
l· I I \ \\
., I I I I I II '1 ' ' I
T I I,
I I
' I'
I I I I
I I I
I
•I '
'
I I •I
For each patient with cognitive impairment, select the most likely diagnosis.

O A) Acute stress disorder 0 I) Major depressive disorder


O B) Dementia, Alzheimer's type 0 J) Multi-infarct (vascular) dementia
0 C) Dissociative amnesia 0 K) Niacin deficiency
0 D) General paresis 0 L) Normal-pressure hydrocephalus
0 E) Head trauma 0 M) Parkinson's disease
O F) Hepatolenticular degeneration (Wilson's disease) O N) Pick's disease
O G) HIV encephalitis 0 0) Schizophrenia, catatonic type
0 H) Huntington's disease 0 P) Normal aging

4. A 42-year-old computer science professor is brought to the physician by her husband, who reports insidious changes in his wife's personality and behavior. He reports that she believes
that aliens have been speaking to her and tampering with their heating and air-conditioning systems. He says that she was upset when she turned 40 years old, and her symptoms have
developed since that tin1e. She was adopted, and her family history is unknown. Physical examination shows vermicular moven1ents of the tongue and bilateral writhing motions of the
upper extremities. Mental status examination shows indifference to her condition and mild to moderate difficulty with n1emory and calculations.

I I
II \ I\
I

'
I
II \
~~· \ \
,,
I I

I
I • I' '
·:I' ... ·1• ' ,•
••·•• ··1··-·
I' : \i
1 , , •

. . l ' :· . •' • ' ', . .


• I ' '' 1
. ' I~ •

5. A 15-year-old girl is brought to the physician by her parents because of an 8-hour history of difficulty breathing.She has a history of asthma treat~d with oral mon~elukastand inh~l~p•••1' •• ,
• • • • • • -- -.· t ..-;1 • , ,_

salmeterol and flut1casone.She has been hospitalized three times for exacerbation of asthma;the most recent hospitalizationwas 3 monthsago. She appe,ars.to be 1nmoder~ted\stress. . •',
Examination sh_owslabored breathin_gw!th intercostal retractions. Expiratory wheezes are heard. The patient says th~t she sometimes_forge~sto ta_ke.h~r~edicat!on.and ~o.es,not',b~lie_ve._ '•. i
that she needs 1t Which of the following 1sthe most appropriate next step to increase the likelihood that this patient will become com~llantwith her med.1cati~r:iteg1men?· \1 , Ii', .·. \\\-' :,,(·: t,

. • I ·,l:· ,t• I
O A) Recommend that her parents reward or punish the patient based upon compliance l I I 1
!
\
i
• \
i1 1 \\_I_ ,., ....
11
.•

O B) Negotiate a contract regarding medication compliance


'1 \
O C) Refuse to see the patient if she continues to be noncompliant 11I \ 1 \ii
O D) Begin clonidine therapy
iI I .. .
II I
\I
I
I!
l '
' .
O E) Begin fluoxetine therapy
II I l '

I
' •
.

O F) Begin methylphenidate therapy


O G) Recommend psychiatric evaluation '•
I
1, \I
I •

I ii \

. -~!
I
'I I ' . •

I l, I
' I H I:

t I
,. 1
6. A 24-year-old man is brought to the emergency department by police 1 hour after his ex-wife found him stumbling around in the yard. His blood pressure is. 100170mm 1;-fg,pulse is 90/min,
and respirations are 16/min. The pupils are equal and reactive to light; the sclerae are injected. During the examination, he laughs without obvious reason, makes.religious statements, and
asks if there is anything to eat or drink. Mental status examination shows a broad range of affect; there is no evidence of thought disorder except for mild paranoia. Which of the following is
the most appropriate next step in management? • '
I'

O A) Observation in the emergency department i I


I '

' I I
O B) Intramuscularadministration of naloxone I .
0 C) Intravenousadministration of 50% dextrose
O D) Intravenousadministration of lorazepam "
' '
0 E) Oral administration of chlorpromazine
11I


7. An asymptomatic 57-year-old man comes to the physician for a routine health maintenance examination. He has smoked one pack of cigarettes daily for 37 years. His blood pressure is
180/112 mm Hg, and pulse is 82/min. Abdominal examination shows a bruit in the right upper quadrant and no masses. His hematocrit is 42%, serum urea nitrogen concentration is
23 mg/dl, and serum creatinine concentration is 1.4 mg/dl. Which of the following is the most likely cause of this patient's bruit?

0 A) Accumulation of lipids in the arterial wall


O B) Hypertrophy of the arterial wall media
O C) Infiltration of arterial wall by giant cells
O D) Infiltration of round cells in the arterial wall
0 E) Reflex vasodilation

.,

r
,, \
l I I
\I i I 'I I
,l I
'I
' .
\ I
I· \I \
,I
I' '
I '
I
' '
'
8. A 52-year-old woman comes to the physician because of decreased libido; this symptom began 8 months ago, after she underwent a total abdominal hysterectomy and bilateral salping0--
oophorectomy for leiomyomata uteri and menorrhagia. She has been taking hormone replacement therapy with conjugated estrogen since the operation. Examination
1 shows f:lmoist,
rugated vagina. Which of the following is the most likely cause of these findings? .\ • )\ , '
1 1
1
I•' ' \ I, 1) I \\ , I

O A} Decreased androgens 0 G} Increased androgens


'
,1l \ , \1' \
I
l ,. \ I

,I \
O B} Decreased estrogen 0 H} Increased estrogen 1\·
,1

N, , \\ ,, 11' 1 ' .
O C} Decreased follicle-stimulating hormone (FSH} 0 I) Increased FSH I
O D} Decreased luteinizing hormone (LH} 0 J} Increased LH I\
' \\ ' /1
,\ ' I 1\' ii
\ \1 •
0 E} Decreased progesterone O K) Increased progesterone I
\
I '
O F} Decreased prolactin 0 L) Increased prolactin l I ! I\
.I
I'
I
I
I· I
I
I '
,· •I\'
' \\ \' I
l I , I
I •

..
• I

I
I
I

I 1
ii

,, I I
I
'I
I
i I .'
I
I'
1\1 ., I .
' ii ,, l
I

' 1'\ 1 I
I
'II
'j' I
I ·\ •I
,, I' II
I .

,, I I I'
:I

.i
'
,j,
11 I

I I
I

.1
., \ l\i''
l .
II \ I ., r
.;
'
,. \I 'I
'
,1 I
II '
I

\\ • ..

I I\\,,\' I\ .\ I ,il \, ,\\ ,\1 1\

1/!
II I
I
1'
i
11· ' '
\. : r
I
\1
1\' I
\
',
, L • ,!,

"'!lm!!'lii• I\ ~' ' !l;,l•\jl..l;ll=;;


9. A 32-year-oldwoman comes to the physician because she and her husband have been unable to conceive for 2 years. They have sexual intercourse every 2 days. Menarche wa_sat.the,·
age of 14 years. Menses occur at irregular 35- to 50-day intervals and last 1o days; they are not painful. Menstrualflow is normal. She has no history of seriou·sillness and takes no
medications. She used an oral contraceptive for 16 years for menstrual cycle regulation but discontinued it 2 years ago. Her husband's sperm count is within the reference range. The
patient is 152 cm (5 ft) tall and weighs 72 kg (160 lb); BMI is 31 kg/m2_Physical examination shows acne vulgaris over the face, upper shqulders,.and back..The remainder oftl;le . • •
examination,including peMc examination, shows no abnormalities. Serum thyroid-stimulatinghormone, free testosterone, dehydroepiandrosterone:sulfate(DHEAS), lut$inizing,hormone,
and follicle-stimulatinghormone concentrations are within the reference ranges. Which of the foliowing is the most likely diagnosis? I • i\ •
1' ' ' "
I ,I '
II \1,' 1~\ I I 11
O A) Endometriosis \ 1 1 v 1
, •

0 B) Gonadal dysgenesis 45,X (Turner syndrome)


I
0 C) Hyperprolactinemia ,,
II
·\
..'
r
l
I I' ' '.
0 0) Polycystic ovarian syndrome ., 'I .,
I ' I I, I ,..,·
0 E) Premature ovarian failure ' I
i
I

I . I .\
: -
'' '
I,
\
\. .I

.
I k ., ,i '
I '
\' ' r ' •'' .. ...
,,
,, I
•I
II \
.. .
,. '
I I•
I
.. .' I

I• I I i\ ' I ·-
I' I ' 1- I I
1
I . .•
I .,
1\
I I '' '
I
I
,. 'I \
I 'I \1 •
I
I .
.'.I '
I I
I , 1
I Ii' ' I it' I I
11 ,I I\
I' I 'lI
I,

I. I
'I
I
._1 , 'I I_' . II ·,
1
I
.
'I
1
I•·
, I
10. A 28-year-old woman at 28 weeks' gestation reports excessive fatigability and dyspnea. Her blood pressure is 118174 mm Hg, pulse is 110/min and regular, and lungs are clear to
auscultation. The cardiac apex is not palpable. S 1 is loud, and there is a sharp sound after S 2 A low-frequency diastolic murmur is heard at the apex that increases in intensity before S 1•
Which of the following is the most likely diagnosis?

O A) Aortic regurgitation
0 B) Ebstein's anomaly
O C) Mitral regurgitation
O D) Mitra! stenosis
~

O E) Tricuspid regurgitation

., I

\ Ii \\ ]' 1-!
I '\ , I,I' ii,I I
[ 1
\
i I
·1·
I
. l1 1\ I 1\
I !I • '
\i1 l
1
.
1
1\ •
I
1
·\
\

+ I .I , •\ ' •
,. ' +" • 1: I • . , I •. ,
' '
I

11. An otherwisehealthy4-month-oldgirl is broughtto the physicianbecauseof a birthmarkon her armthat ha.sincreasedin size over,thepast severalweeks.rf0fiif11ination showsa.
~anagegient? 111i • i1 1
3 x 2-cm, bright red, raised, soft, nontender,compressiblepatch overthe leftforearm.Whichof the followingis the mostappropriatenextsteP.•\n
I I

., 1,\ •• 1'
' t,
1 1 11
O A) Topical corticosteroidtherapy ,1 • . , , 'll\11,,\.1.., ,. \ii : \1 ii ·"1 I
C

l1 '' I I
I I I 11 \
'' '' ., ' 1' \\ I ]
' '\\'
\ ' I
O B) Biopsy 1,1
, • \'.l•, 1\, 1 1
1 ' ' • " : ,.
,\ i\ ! 'I·
O C) Laser therapy
I .
0 D) Excision " ' \
,; 11,

"
,,
I ' !'"

O E) No interventionis necessary ' .\,,,11


\ I I I

,,
1 I ' ,I '
\',

''

' \
, I
I I \ \. ,\\ I' ,
,\\ \ ''

I I
I I I I •'
I \ '•
' \; ' \ '
' ' .'
1111I I I 1 \I I I '\
11
.l l ','1 ,\
I
' , l \ '
I
11'
I I '•
\1\ \1\I \\ •I I I \\ \ \\

\I \ \
I
I
.!I l . ' 1
' . ,,

.\\,' ,a. ii·


I ill \\I,' I ' ;~\
I
.\
I
I l II
,I '\ ,.I I

\
I:
I \. \'
' \
''
\
12. A 67-year-old man is examined 3 days after undergoing uncomplicated coronary artery bypass grafting. He has hypertension,congestive heart failure, and hypercholesterolemia.His
medications are patient-controlled morphine, a laxative, lisinopril, metoprolol, furosemide, and lovastatin. He is alert and fully oriented. His temperature is 37.8°C (100°F), pulse is 67/min,
respirations are 18/min, and blood pressure is 128/72 mm Hg. Examination shows clean, dry, well-healing surgical incisions over the sternum and right lower extremity.Mental status
examination shows a bright affect. A urinary catheter is in place. The patient is able to move from the bed to a chair with the help of physical therapy. He will soon be transferredfromthe
intensive care unit. Which of the following is the most appropriate next step to prevent iatrogenic complications in this patient?

O A) Remove the urinary catheter


O 8) Begin famotidine therapy
O C) Begin subcutaneous heparin therapy
O D) Discontinue furosemide therapy
O E) Switch from morphine to acetaminophen-hydrocodone

"

I . I \ ,.

13. A 72-year-oldman comes to the emergency department after a 5-minute episode of blindness in the right eye. Over the past month,he has had headache and pain in the jaw with
chewing.His temperature is 38.1°C (100.6°F). Examinationshows tender, nodulartemporal arteries with decreased pulses. His erythrocytesedimentation rate is 92 mm/h.Which of the
followingis the most appropriate immediate step in management?

O A) Carotid ultrasonography
O 8) Aspirin therapy
0 C) Cefotaximetherapy
O D) Corticosteroidtherapy
. O E) Temporal artery biopsy


14. A previously healthy 10-year-old boy is brought to the physician because of a 2-day history of a pruritic rash on his arms and legs. He says he
first noticed the rash 24 hours after he played in a wooded area in his neighborhood and that the rash.Hasbeen spreading. He has not had , .i
fever. There is a pet cat at home. The patient is alert and is scratching his arms and legs. His temperature is 37°C (98.6°F). Examination of 1 ' . ,i1\\\
the upper and lower extremities shows severe erythema and bullous lesions with discharge; there is a sharp line of demarcation between the [
rash and the unaffected skin. A photograph of the left lower extremity is shown.Which of the following is the most appropriat~ 1 111,1

recommendation to prevent recurrence of this type of rash? ,\'1,


'
O A) Avoidance of the wooded area tiI •l
I': II
0 B) Thorough washing after playing with the cat •I ,I'
'.
O C) Prophylaxis with an antibacterial agent
O D) Prophylaxis with a topical corticosteroid
0 E) Prophylaxis with an oral antihistamine

,,

,l ''
.
,,
.
'
•I

: • • ♦ .• ••
•1
I' ' !!
,. '
1 \ '
'
l, l •
' j ,·

'

• \' I
Il
i f
·1•· II{ I t : ..~· ~!•, :•'.'j•lt 1 •U,t.J .. ,f1 .,.
,I ,, ' . I
. • • • • ' • •
.
15. A 37-year-oldwoman is broughtto the emergencydepartmentbecause she has been unableto see out of her right eye since aw~kening2 hours ago. She states that ~nymovementof
to' l!ght Theright pupilis •
the eye is painful. Examinationshows visual acuity of 20/200 in the right eye and 20/20 in the ,lefteye. The optic fundi ~re normal.'Theleft r:,ypilreaqts nom1:a11y 'i
poorly reactive to direct light. The remainder of the eye examinationshows no abnormalities)Whichof the followingi's-themost lif<elysite of the lesfon? /. , ·, ;;,.. · ,, ·:
,. .• 1 I! I ! I• • I
I ', '.:I It• :,,,_ • •

0 A) Left optic nerve 0 F) Retina ,i,. I •/ ·'1'1 i l;_,I


1i_l,·1. J::. • :ill,.:.' ,,llii1'1· ;'•1.'. ':i 1 ' •

0 B) Left optic radiation 0 G) Right optic nerve i i' / • 11 I ,::I i, 11 I'.·, I ,ii..l\i!',' ' : :•. ' !\llh•.
'\ .
O C) Left optic tract 0 H) Right optic radiation .. I t!: •
I .I I Ii'\:;;
0 D) Left visual cortex 0 I) Right optic tract
I'I ..
I
'i I'
O E) Optic chiasm 0 J) Right visual cortex 11 i i,\ •
'
I i 'l• l . . ...
.
I j". ' . I
l l ,,
11
l
I • j;. I

1
II,I·
;!11:
I I I • I•
I I
I ,. ;, I I
'l
"
d
I
I
'
.. ' 'I
'
i I.
' I;I
'I1. It
,.
I ., 1] I •
1
I
I'.
I
l II..I .' i . 11.
I I : ,,.
• ' l I
..',r
I • ... I' '
\~ t

......
I :.t

'

l : '. .. .,,
+ i!

I. .
"
l~

I '' 'I

I. I
I I
j,1 J •:~
I •
.I .I .
1 '
I !\\

l

16. A 27-year-oldman comes to the physician because of a 1-week historyof shortnessof breath with exertion,paroxysmalnocturnaldyspnea, and swelli'ngof his feet. He has not had Chest
pain or palpitations. He has been healthyexcept for a "bad cold" 1 monthago that resolved spontaneouslyafter 1Odays. His temperature is 37°C (98.6°F), blood pressure is
90/60 mm Hg, pulse is 120/min, and 'respirations are 24/min. Examinationshowsjugular venousdistention to 8 cm. Bilateral basilar crackles are heard. Cardiac examinationshows a •

diffuse, laterallydisplaced point of maximal impulse.There is a normal S 1 and S2 and an S3. Examinationshows 2+ pretibial edema bilaterally.An E.CGs~ows no abnormalities.
Echocardiographyis most likely to show which of the following?
•I
O A) Asymmetric septal hypertrophy 11 \I H
O B) Bicuspid aortic valve with stenosis 'ti j J
O C) Diffuse hypokinesia and dilation of the ventricles
\
I I l
O D) Oyskinesia of the left ventricular apex li l
O E) Mitra! valve prolapse
ll 'I
,I'
\I ' . ' I

1,
.I
I

II .t\\ •
Ii I
I,
!1 I.I
I ,I
' ''
ii I ji!
'!
I
I
I· I 1,!;

I
I I..'

I'
11
I
d, I
'iI
'
..•
I 'I
I 'l .
!,
l ii 1

~l 1·
I '
II 'I
'
\ .
I i' I• r • •
l' , .. . •
,. 1. ,·
" ; ; • •
• '
' j
• •

I,
,

17. A 67-year-old woman is hospitalized because of ·abdominal pain and persistent copious vomiting for 24 hours.Two weeks ago, she was hospitalizedfor treatme~ o(atriaJp~rillation; .
after cardioversion to a normal sinus rhythm, she began treatment with warfarin.Yesterdayat a follow-upvisit, her INRwas 6, and her medicationwas dlsc:bntinu~d.:Shetakes no other • :
medications. Her temperature is 37°C (98.6°F), blood pressure is 100/78 mm Hg, pulse is 120/min and regular,and respirationsare 20/min.The•abdor;n'en !s ~istend~ ~~moderately·

blood is negative. Her hemoglobin concentration has decreased from 13 g/dl yesterdayto 7.8 g/dl today. An ECG shows a normalsinus rhythm.Which ofthe followingis the r(lost·likely J
explanation for this patient's abdominal symptoms? '11 : , i • ,1·1,
• 1! 1 :
1!
1
.'
1 I ( .! 1· ,j 1i ;; ·:
1 ' I
·•·I'·•'' . i,!
II i .
I ,

1
1 1 1 ·11:
;·1 l !lidhd •111iF !1 r·
O A) Internalsmall-bowel herniation .I . 1:jl: I" 1
''l:'' : '\ilhl,:, qI I1'11· 1'
I I ' 1' I I I ' ti I • I j: 1··,1.
··1•1111··
1' P,:i1 it! I' ' .• I

. ':' 'I r:! 11r1'!i ·.·\ j! I; •. ·:;. I '.i


O B) Intestinal ischemia from a cardiac embolus
O C) Intramural hematoma of the proximal small bowel
;i I I 1'.11' 1!i Ii' .,il:'l
...1, 1' ,1.1ft"1 '

O D) lntussusception of the small bowel


•'" • C
I
'
~
Iq
.
tLi
' ): ... '

;-1;
.
\! •
1,,,1
I·• • . •

I
t t

O E) Malrotation of the small bowel 111


I I ,, • j ·!'i!l !):!,; ';!·; '
1lllt":111iif I I :
1

II 1'
I
• i
I' ·,1 1
11
1i I
l• ' ,,
;, •
i 11,;J!.,·.1
r"1
I'
1 ,ll ;= ! i ; :/
1

,,I / I'ii . )1 •
1I
1
,

'i:• '

'
II I ' 11I I ' • :i1: I ! : •

1
1 ! ,1 j,
J •I
t•·1··•,,·,•1
t· ! • ,j
I
1!1sllll,.• , • , , l
• , 11 1,ilI·11 I ;i 1ll!.
1!1j_ 1 1 1 1'F '_Lii
: ; • . ·.
.,
II
. 1
I :•
!I
, 1I'.
;:1'!i1:
• i , •.
•.
• jl 11 ., '.:I !11! . i ,
I ..
·1

'i I 'l. !l ! ,.;, l


I
I I

!1
..1 : I'
I
I I '
'1'./ i;/i'l .'!, !1 ' ;
-11,:,=-~
·f •
ii
-1• 'l!

II
I 11 pl

:

'
' •

..
! l•.':: 1 1

l :ifi
i'lj'1 ,,,,,, ·•.·

iI I I • 'r 1

'i .'
·1 • ,: :l • 1 •
1
1/iJ • ,_'I , ii//!( /. :i '

I I
'
I 'I . 1·
I
II
i ',
!

• I

• •
. I 1!11
••111./1·j11/11//'I!
: ,·: . ' ..
' i,-1! ;., ',.•• . '·
"1 '
. i

'
Il ·: ,,
,I.:,
I I jI • . . '
l

, . ,·i•l'r'i rd'1'Ii,; • • 1 • 11iJ• 1 ti


,, 'I j 'I I i~ .. . .•
' I ' ! I ' • '

18. t:,25-year-01dwoman comes_to!he physician b~tause of a 3-month history of ~heuhexplained!ur~eto_eata',few,~able spo9ns of co_r.pstarch daily. The am9u~t of cornstarchif"'9e5!edhas
1
increased graduallydunng this time. She has le1omyomatauteri; she 1sotherwise healthy.Her.weight1sunchangedfrom her11as:tv1~1t1_ye:arago; she we1_ghs_p1 1 kg (1~5-lb)and 1s . r
168 cm (66 in) tall. Her blood pressure is 120/80 mm Hg, and pulse is 100/min. Physicalexamination shows mild pallor.,Newrolcigitexamination shows no abnormalities. She is •
embarrassed about her problem. She has no compulsive behavior or obsessive thoughts.An .ECG showsisinu$tacl;lycardia,.Whic~ ofthe fpll9wi_ngJsthe IT1o,pt
·" I I

appropriate .™:t,xt step !.ri


management? , I 1
1 11 ! , 1 !,
1 1
' 11 'I .I :· ' 11• , ., ,Ii ,. • ::·: ,:''. , .,
11 " •;;!! 1i ,11 ' 111, ... '' ' .•
1
! 11 ;:, ,,,1
1
0 A) Psychiatricassessment ·11
·l• I '
11 ' ·•
;'
'I
' I ' I
11
1,1··· I ;\ll '•
. : • ..I ' • •
I ' . f !

0 B) Complete blood count I· I 'I' ·I '


I

'
11 •• •
'I II 11 !I
0 C) Thyroidfunctionstudies
0 D) Urine toxicology screening I ' ':1 • I
,J
I .,
!'

·11 '
1:1 I I
I,
rl
•''
11
'II •I I lji. . I

0 E) Admission to an eating disorders clinic l .


I ••

0 F) Antipsychotictherapy Ii
·r '
' µ,1 jl
; ·11
., • . !
• I
,.
1, 11 lj IiI '
I

0 G) Folic acid supplementation


0 H) Selective serotonin reuptake inhibitor therapy
,!
i
'
.
I ii
,. ii
;,
r 1111

,.
J I
,, '
l!I -II
.• I I
' I

11
I

I I ,, . ! '.
I ' I
:11
·1
,, I·
I
I 1..
I
!i· .
:
I 'i

Ii
''
' .II .I I
I
I
I
••
'' '

,, I ..
'I •I

I II .,: "
Ii
' I
I
!
.
:

·1 ' '
'
19. A healthy5-year-0ldgirl is brought to the physician 5 weeks prior to a family trip to Egypt.The family plans to visit the child's grandparentsin a small village.-Herimmunizations-~re~t~
date. Examinationshows no abnormalities. Which of the following is the most appropriate prophylaxisfor this child during her trip to Egypt? , , ·
0 A) Acyclovir ••
• • •·~•

O B) C-eftriaxone
0 C) lsoniazid
0 D) Mebendazole
0 E) Mefloquine
O F) Metronidazole
0 G) Penicillin
20. A 24-year-oldprimigravid woman at 18 weeks' gestation comes for a routineprenatalvisit. She has had increasedbowelmovementsoverthe past 9 weeks;the stoolsaresometimes
coveredwith mucusand blood. Use of over-the-counterantidiarrhealdrugs has not relievedher symptoms.Pregnancyhas.beenotherwiseuncompliciited.She has nevertrave~ outside
the USA. Examinationshows erythematous,tender nodulesover the anteriorsurfaceof both lowerextremities;some of the noduleshavea violaceoushue.The uterusis consisterrt.i~ size
with an 18-weekgestation. Rectal examination.showsno hemorrhoidsor fissures. Fetal hearttones are audible by Doppler:Whichof the followingis the most likely~!~gnosis? '. .
• ,,
I' I I I '' I '
O A) Amebiasis " 1
I
· ,j\11 , .1'I,,.· '
O B) Diverticulitis , .
O C) Hyperperistalticdiarrhea \,11 :··!-..
'.
O D) Inflammatorybowel disease _,, -.~..,,,,;;,, , 11 1: 1,

O E) Viral gastroenteritis 1
1 ·11
......
, , Ill\" _ \\\ • '_ ti '
'
1
-•
' i'
1
1
! I
I 1
'I .'
!
".,.I.. .
! ,,
I
I. r
I_,

' ti'' I
'
I I
I
,

'I ' '' ( '

I ;· ~
t . '-~
'
,.
'
•1' . !I

, .. ;
. I .,
' ,,
1]'' . .
.
j 1'. ' • I

' . '
,21_ A programfor the primary preventionof coronary artery disease is implementedin a communityin the USA, Assumingthat diagnostic proceduresand detection remainthe same,which
of the following measures involvingthe disease is most effective in monitoringthe program?
'
I O f\.) Case fataljty
I
, O B) Hospitalization
O C) Incidence
O D) Mortality
0 E) Prevalence

22. Two hours ago, a 24-year-old man had the sudden onset of pain in the right side of his chest that has become increasinglysevere. •
He is now having difficulty breathing. His temperature, blood pressure, and pulse are normal.An x-ray of the chest is shown.Which
of the following is the most appropriate next step in management?
O A) Bed rest and sedative therapy
O B} Antibiotic therapy I :.
l
O C) Anticoagulanttherapy
OD) Tube thoracostomy
O E) Immediatethoracotomy
f
t

'I
''

..,j

' • '
I
I •
l,' .

jl 11
1
I 1 I · .

23. A 27-year-old primigravid woman at 38 weeks' gestation is admitted in labor. Her pregnancyhas been uncori1plicated,and a routine prenatal visit 2 days aqo show.edno af?noff1)alities:: •.
On admission, fetal heart tones cannot be heard. Ultrasonographyshows little amniotic fluid, fetal edema, and.no eyidence of.a fetal heartbeat.After 1½ hours;.,shedelivers a • •'.' . •
3175-g {7-lb) stillborn infant; examination of the infant shows no obvious abnormalities except fcjrmild e,dema._1'Thep1acentaand membranes appear nprmal{1Which of the foflowingis~;·
1 1

1
most appropriate immediate course of action? 1
,1 • , , ,, ,i,1, •; , • , 1 '
I' I I ,, I \I i' l I ,i"I ,:
'j\ I
I
•I JI' ·;,:t :· ;I 'I:\' \ • l
1' I I, ':\I ,1' ',' ,, ,, .1.·,r. ·!":l.'·,
O A) Notifythe hospital liability department I' d1 •I
,,
• ' i ' ,I .: ,
,, '
! • 1.
.
O B) Obtain consent for fetal organ donation from the parents ,I
I
'
II:
I

I

''
I'

0 C) Recommend autopsy of the infant I.,. ! ,I' \ '
I
I
O D) Tell the mother not to worry since she can get pregnant again lI l_l_
I
I

0 E) Tell the parents that there is a 1 in 4 chance of recurrencein future pregnancies I. i' ;1
•li-
j.
d 1' •
I
I'1I I
I
I
I ,:_I I'•)H\

\ . .
r/1
,.
' ' .. . . I

I
• I

'I l
.
,, '

I .,'I •'
,I !
I • ;'
"'
lj-
.
I I
1[ \
·'
'1\
,
;

II I ': I ·;
I 1,

"
I ' •I
\
11 l I '
"I . .:


I l
'
I Il I '
11'
. ..
!' • •
;
. \
ti . .'
I I I i'
I I I
"'I' . ' ''
I ,I ,1 !r I •:1. .
l II I

I .II
i . ' '
• " i•

,··;·.:;,.
• I 1 • I
I'I I'
i
l. I -1+
.' , •
.
'
!i
·1·1 •' "l "• I\1
1,1 ii' I1
1I I I I1I 1I\
1 '
\'ii
,,. •
:I' "' '
•I . ''I ' l .t -111
• ' l •:;~
:' •' ' ' I ' ' ': ' • I ' 'I
, , ;; I " ., :· • ' 1! , I ' • : • •• \ - •r.

24. A 32-year-oldwoman comes to the physician because of lethargy and boredom since the birth of her son 5 rnonthsago. She worries about her 'abil/ty,toc~r~ for hi~·anq has h<:id~eq~m: ..; 1•
palpitations. She is ~nable to fall back asleep after n_ighttimefee_dings.She stopp_edbrea_st-feedin~1 r:nont/1
~go. Her s_priis h_e_althy, hi?·ag~., :\
and gr6wth and d.evel9~r,e'nt.~re ~onyialfO.f 1

Her blood pressure 1s122/80 mm Hg, pulse 1s58/min, and resp1rat1onsare 18/min. Phys1cijlexam1nat1on shqws n,,0abnorm~llties..She r~mernber~ o,ne··o~,oJthre.e.op~ect~ ~fter. \;," ,'·, ~)i
5 minutes. Her serum cholesterol concentration is 265 mg/dl. The most appropriate next step in diag~osis is measurement of which.of the following serum ·conc~ntrationsf?/, .:' •, . 1 1• •· i'
: I , 1
i. , ,,

0 A) Cortisol 1\1 11,I 1\• \\i !I\ •1 \\\ ,, ·'\ ,. 'ii \, ' ,! l I :.' :,, ,•,-·, ;,:,1')
0 B) Estrogen ,, . ' ' ' " ..
I , I
" ,.' .,•
i'
I
•1· "
. ' I ...
• • I t

I
I .. t·•

0 C) Progesterone
t' '. '

1 1. 1\ ' '\'
•I • ' ' I :l
·1 • ' •\"'I
I

i ' . I
,I
•I
\l
I ,,.. .I• t
i,.
O D) Prolactin 1\l
I
t ' ;
II
~:1

J
I'
I
' \\ \I ·w
'.
0 E) Thyroid-stimulatinghormone '

I .;t
' I
r ' I •' " • '
'
I
1
,1\iI
"1\\I 1
I I ,• t '
. . •t . 1

. ' ,I
; '!
l• ; ~ \,' ' .,
'
' '
.' . ' ..'
II ii 1' I I 1\
''
,
I. ' l '1;
..
'
' lI
I (.
'II
'. .
'
...
'
1 l, ~' ·'''
I
1 '
I
' II
i ,l
, , '.II'
' I ' 1: • •Il

1\1
ll l '
'

' '
I ..
\I
l\'I I \1\\ I, . ; .
l
I I ' It!
" I

I

lI ' ",,
•I t

1l
I
25. A 20-year-old man comes for a routine health maintenance examination. He has a 15-year history of difficulty relaxing his hands after tightly gripping objects or after shaking hands. His
father has cataracts and frontal baldness. Examination shows thin forearms. There is moderate weakness of the hands and difficulty releasing after gripping with his hands. Which of the
following is the most likely diagnosis? ' . , ,
'' I
I
0 A) Amyotrophic lateral sclerosis
0 B) Cervical spondylosis • I
'•
O C) Multiple sclerosis •
• 11

0 D) Myasthenia gravis I '1\\


\
'
0 E) Myasthenic (Lambert-Eaton) syndrome '' I ·•
,\
0 F) Myotonic muscular dystrophy I,

0 G) Polymyalgia rheumatica '


' I

0 H) Polymyositis
O I) Progressive neuropathic (peroneal) muscular atrophy
I I 11
' .
' I

II
''
I
\ '
II• l
I I I \\ I I I
,, ' • \
\ •

1\ .'
I
l
I
\I .\ ' '.
\
II .
l l
I I \
\ \' I
I
I
'
I ' ,'
' I ' '
I
' I\
. • I
'.,

I I
'
.I I
l' '
I \ ' ),. I I• l • •
L ',

1•
~ '
.• I . I\ \ l
I \l
II

I l • I
i
' \\, '
~~~~~~:.:,,i:
,

'

. l' .. , ,
26. A 2-month-oldboy is broughtto the physician for a well-childexamination.He smiles spontaneouslyand vocalizeswithout crying, but he does not appearto laugh or squeal. He will not
work for a toy that is out of his reach. Which of the following is the most appropriate assessmentof languageand psychosocialdevelopment? .
Language development Psychosocial development
I ' \! I
,I '
OA) Normal normal \1
0 B) Normal delayed
I, .
0 C) Delayed normal
0 D) Delayed delayed
\i l
'

li l "•

I .,
I•
·I
1!
I
I I
..•I
..
I
27. A 77-year-old woman comes to the physician because of a 2-day history of cramping abdominal pain and distention accompanied by nausea and vomiting. She is otherwise healthyand
has no history of abdominal operations. Her temperature is 37 .4°C (99.4°F), blood pressure is 110/86 mm Hg, pulse is 112/min, and respirations are 2;4-/min.Cardiopulmonary
examination shows no abnormalities. E_xaminati_on of the abdomen ~hows distention and mild diffuse _tendern~ss;_bowel sounds are_high-p!tched.An x-ray ofthe abdomen shows air-fluid
1
levels throughoutthe small bowel and air 1nthe liver; there 1sno gas 1nthe colon or free air. Which of the following 1sthe most likely d1agnos1s? ,1
'
I' I
0 A) Adhesive small-bowel obstruction I

O B) Cecal cancer
0 C) Gallstone ileus I !,
I "'
'
I!
I

ii
0 l'
D) lntussusception I
I I l l I
0 E) Mesenteric infarction I
0 F) Ruptured appendicitis I ii I ,,ii l
·,,

I,II
I'
I '
11 . I
I
0 G) Small bowel lymphoma II '
II 1
I I

I' I I '
11
I 1' '
I I I II
l Il I

'
1!
,, :l

l1 !\ I.
I
" I
I, I ' 1
l ' I •
I

.i
1• ll I l \
I
II

I • ]I '
j.
28. A 32-year-old woman comes to the physician because of a 1-year history of increasingly severe dull pain in her lower back and buttocks. She also has had morning stiffness of the lower
=- back that lasts for 1 to 2 hours. There is no history of trauma. Her job does not require heavy lifting. Her blood pressure is 11orro mm Hg, pulse is 68/min and regular, and respirations
are 16/min. Cardiopulmonary examination shows no abnormalities. There is tenderness to palpation over the sacroiliac joints bilaterally and decreased flexion and extension of the
lumbar spine. An x-ray of the lumbosacral spine shows sclerosis of the sacroiliac joints. Laboratory studies are most likely to show which of the following findings in this patient?

O A) Decreased erythrocyte sedimentation rate


0 B) Histocompatibility human leukocyte antigen 827
O C) Increased serum antinuclear antibody titer
0 D) Increased serum carcinoembryonic antigen (CEA) concentration
0 E) Positive serum rheumatoid factor


'. .,
I , • ' ; 1 • . • :i ~ , f!~1 ! \ '

29. A previouslyhealthy52-year-old man comes to the physician because of a .3-monthhistory of:increased urinaryvolume and intreased urinaryfrequency at night. He.has had a\\•:, .,
6.8-kg (15-lb) weight loss during this period despite no change in appetite. His father has hypertension,and his mother has hypertensionand type 2-dic1.betes
mellitus.'He currently.weigh~
• I
95 kg (210 lb) and is 178 cm (70 in) tall. His blood pressure is 160/85 mm Hg in both arms. Exan1inationshows no other abnormalities. His nonfastingserum glucose c9ncentrati,onis •
280 mg/dl. Which of the following serum concentrationsis most likely to be increased in this patient? • I .:' i'\
1
Ji; ..\\\\.', _1. ,\
1 .
1 1
I •j1\\• •
O A) Bicarbonate .'
1 •
1•• • I
"
iii I i
I I'
0 B) Glucagon Il

0 C) HDL-cholesterol ,I \
' •
0 D) Insulin
I
'ii \;.: .:•i .,. ·\•.' ... '
'
•.
.. i
0 E) Ketones

;iI
I
l
l 1

I
I,

I I

It

1l
I
'I
,
, • ' !

30. A 3-year-old boy is brought to the physician because of a 7-day history of fever and a painful swollen lymphnode in his groin. This is his sixth episode of lymph node swelling; the previous
episodes resolved after drainage and prolonged antibiotic therapy_He also had pneumonia at the age of 12 monthsthat required chest tube placement for d~ainage.A maternal uncl_e
died during childhood of recurrent infections. The patient is at the 5th percentile for height and weight. His temp·eratureis 38.5°C (101.3°F). Examinat(onshows a warm, tender, 1 /
erythematouslymph node in the right inguinal area. There are several healed incisions over the inguinal area.and neck from old drainage sites. Laboratory studies $h'ow, •·'i
. '
1
Hematocrit 35%
Leukocyte count 17,000/mm3 ,I l
\ .
I I
Segmented neutrophils 65% 'I
. . '
Bands 10% I I'

Lymphocytes 25% I ' .l •' '


lt1
350,000/mm3
• l
Platelet count \!l \II

( I
' . ' '
}'
II

I
, • ' :. • I

A Gram stain of the lymph node aspirate shows numerous segmented neutrophilsfilled with ba~teria; culturesgrow Staphylococcus!aureus. Which of the following is the mpst hke!y.,,;:I .
• ' .•'
mechanismfor these findings? • :· • \, ' ;;1 \'1 ::,·;w!·:I
1
I I • .·,-:!,' l I.\ I I
' ' 1\l .:
, I'

O
1

A) Adenosine deaminase deficiency l :i 1l \I I


H I
• ••
1
O B) Consumption of complement I ':i11 :1.\.
l\
II 1!
'
11·;,.\\l_l • ·, ',: .',
O C) Defective opsonization " '

,. "..
'I
0 D) Destruction of C04+ T lymphocytes I Ij
I
'
"
• .•
.
•t \
O E) Developmentalarrest of maturation of B lymphocytes , l I, ' l\ i •

O F) Oysmorphogenesisof the third and fourth pharyngeal pouches ..I .'\!' .


\
it

' ' '


'' j\l .''
O G) Impaired chemotaxis l
II
II
.'
:\
'I
:I , I'1 1
j

0 H) Impaired phagocytic oxidative metabolism I ' '


'
II i'
'I j
''I
I
,:
• ,. , :.
' I j, '•
!
'
l ' : ' .. i .'
'I
.1-' ',:,i.
' •·(r
1

31 _ A 57-year-old man is brought to the emergency department 30 minutes after he was found on ,the flo~r of his house;'he has left hip pain and\?hortness 9f bre~th. He has ~enalfaili:JrebtJti~',;!~
ha.smissed his last two dialysis treatments. His renal failure was caused by inadvertent ingestion of ethylene glycol. His rehal function did not improve, and he is currently on the ·,
transplantation list. Medications include amlodipine and doxazosin_ On arrival, his temperature i~ 37.5°C (99.5°F) blood pressure is 150/100 mm Hg, pulse is ,95/min ~fld regular, and , ijli
respirations are 24/min. His breathing is rapid and deep. Crackles are heard in the lung bases. E.xamination shdws a soft a'bdonien. Bowel sounds a're normafi The left ,lowe( ezjremify ts:;!·;:
externally rotated_ Laboratory studies show: , '. • .!I . ,., ' ', ";, •1 \! ·11:(I
' I\, :: ' ' . ·11 .• ' !, . • ·ijlt ' : '
Serum ' 1. i· , ' ,,,' ; , 1·: ,;!J: 1 .:1111. •'.'
Na· 135 mEq/L "· ,11·, ' ·i:r rn,·,:
• :1 ~
t
Cl- 102,mEq/L
K• 7.1 mli:q/L I . I
I
HC0 3- 12 niE~/L . I ..

. I

Arterial blood gas analysis on 4 Umin of oxygen by nasal cannula:


.,
I
7'.22

'
'
•"
I;,
32. A 6-month-oldgirl is brought to the physician because of poor feeding and labored breathingfor 2 months.She has had recurrentrespiratorytract infections since birth. Examination I

shows a to-and-fro murmur in the second left intercostal space, a loud S2, boundingperipheral pulses, and a widened pulse pressure. Which of the following is the most likely diagnosis?'
I
O A) Atrial septa! defect (ostium primum type)
.. I

0 F) Patent ductus arteriosus


O B) Atrial septal defect (ostium secundum type) 0 G) Tetralogy of Fallot
O C) Atrioventricularcanal 0 H) Transposition of the great arteries
O D) Coarctation of the aorta 0 I) Tri cuspid atresia
O E) Hypoplastic left heart syndrome O J) Ventricular septal defect

·I

l'
~I
''
'
33. A 4-year-oldboy is broughtto the emergency department20 minutes after being involvedin a motor vehicle collision. He was an unrestrainedpassenger. On arrival, his blood pressureis
11OrTOmm Hg, pulse is 100/min, and respirations are 32/min with gruntingand retractions.Examinationshows multiplebruises over the chest.Arterial blood gas analysiswhile breathing
40% oxygenshows:
pH 7.38
PC02 34 mm Hg
Po2 66 mm Hg

An x-rayof the chest obtained 4 hours later shows diffuse infiltrateson the right side. Which of the followingis the most likely diagnosis?

O A) Acute respiratory distress syndrome


O B) Aspiration pneumonia
O C) Fat embolism

0 D) Hemothorax
0 E) Pulmonarycontusion
!
,,' •
I. •
''

!
'I ,I l
.
•-- •·••• •• .... , -•-.• •• ••- ..,.,,-1' 11'''' I 1 !11°;-r--:---•r,-• t'1'♦••n "1,..iITI rri-rir1111f U I' I u, t• •,,--rr,n
34. An 18-month-oldboy is broughtto the emergencydepartment because he has not used his left arm since he fell while walking and holding hands with his 8-year-old sister 2 hours ago. On
examination,he holds his left upper extremityat his side with his forearm pronated.There is no tenderness of the left lower extremity,but there is restricted movementof the elbow. The
remainderof the examinationshows no abnormalities.Which of the following is the most appropriate initial step in management? I I
O A) Passive hypersupinationof the forearm
O B) Application of figure-of-8 strap
l·l. •

O C) Administration of analgesics and application of ice


O D) Aspiration of the elbow joint
0 E) In-placesplint immobilization of the elbow
'
I
I

I
.I'
I
I

1
,. I
'
I
I,

l ,.
I' I I . 1
'
35. Four hoursafter undergoinga cesarean delivery at term followed by tubal ligation, a 37-year-oldwoman,gravida 2, para 2, has dizzinessand confusion.The operationwas
uncomplicated,and blood loss is estimated to be 800 ml. Patient-controlledepidural analgesiahas been moderatelyeffectivefor pain. Her blood pressurenow,,is80/40 mm·Hg, •
decreased from 120/72 mm Hg intraoperatively,and pulse is 152/min,increasedfrom 96/min intraoperatively.Breathsounds are decreased bilaterally.No murmursare heard. 1 _1 . ..
Abdominal examinationshows distention and tenderness. Bowel soundsare absent.The incision is intactiwithr,ipdrainage.:$he;is disorienteq to person,place, and:trme.Her herhatdcr.it ·•:.
1 1
is 23%; preoperative hematocritwas 35%. Which of the following is the most likely cause of the hemodynamicchanges? i.! ,. .,
• I
:1!). I, i, "f , .. j1. . ,t \

•·111,.1·'. ' : ' '.,i \ "•


O A) Epidural-relatedhypotension l'
' ' I 'I
t I
,d
I

O B) Insufficientintraoperativefluid replacement
+, ~:
I
• j
;1 :1
'1!,, ii
' ! •• 'I

I!
O C) Postoperativeintra-abdominalhemorrhage I
OD) Supine hypotensivesyndrome
O E) Underestimatedintraoperativeblood loss
,'
I
.
•I

'
: I

I·i I I I
••j 'I ·1'
I:
• II f
''
! •I I '
: lI f
'
• •,I '

I ,' •Ii
• I
; ·!IJ I II '
''
: Il •
I,
''
I
.•
r!
, I
.;
I "I
I
I 1.
II
l' I
.'I .
I ,I
,I' I
I
.
, ...

.I
I I'
l:
"'
1 :,,

I
, :l
., - • - • ,, • j ' ,,, • • 1 • .. . . . ., .

Theresponse ~ptions for the next 2 it~m;S~,r~;t~e·~i'roe.


.-~ ••
Select one·aru;werfor each item in the .set•
.;·, •• :·::-:;::::;:::::· ·; • ·, ,"t:':, ,.· i. .:,!\ . : ::

For each child with fever and cough, select the most likely diagnosis.
I
.I
0 A) a 1-Antitrypsindeficiency I
0 B) Cystic fibrosis
0 C) Pneumothorax
O D) Pulmonaryalveolar proteinosis l
I ? '
O E) Pulmonaryaspergillosis I
I ! II '•
O F) Pulmonaryhemorrhage I

0 G) Pulmonarytuberculosis

36. A 20-month-oldgirl is brought to the physician because of fever and cough for 2 days. She has had several:similarepisodes since the age of 4 months.Three months·1=1go, she and her
family visited her grandmother in Finland for 2 weeks. She is at the 25th percentilefor lengthand 5th percentilefor weight. She appears thin and pale. Her temperature is 38°C (100.4°F),
pulse is 150/min, and respirations are 40/min. Examinationshows mild clubbing.Wheezingand bilateral crac~lesare heard at the lung bases. An x-ray of the chest shows streaky
:
densities bilaterally with mild hyperinflation. •

iI . I
II I
, :I I
'

·I
ll'

I I

l
til ...
frc,1 p , .,
[),,-,., .. .....
,,_ V r Lab Values Review Help Pause
•I" .
II , '" ',.' '11
',! ' , ..
' :1'• 1 I 1' .:1•
'
l 'I !
• I '' : "
For each child withfever and cough, select the most likely diagnosis.. l ' ,, '
..' ' .I '
t ,
.
I

,. ,· ,, ,,'
0

I ' . ......
'. .
il 1 I
,I \
II
I
.'
0 A) a 1-Antitrypsindeficiency ·I
I
i·.11 I I
I' 'l' :l
I
''
O B) Cystic fibrosis ,. ',
I I
..
,,' ! !+
O C) Pneumothorax ' '
I-.
,' I I.,
,. I ,I- ,. :
I!
I
I' :\ ' I
.
' :I
I :I I \1! '' ''

l
' '
• •I
I '' l
O D) Pulmonaryalveolar proteinosis ' '
I l 'I
I'
I

I
I
I
I
'' I
' '
' ', '

O E) Pulmonaryaspergillosis ' I l 11
'

0 F) Pulmonaryhemorrhage , I
.I '
I
I
I
'; I i. I ...'
:: '
I "
" l
<' 'I:·
,,• I
'
..
•11 ·l}.

0 G) Pulmonarytuberculosis I I I
I
,.,
'
I ' {.
• i I p
h
p~ I
.!. '
,:
,
I ;t'
'
,, :•1
I~ T ••

37. A previouslyhealthy 16-year-oldboy is broughtto the physician because of fever and coughwith right-sided chest pain for 2 weeks. Six month$ago, he visited·.hisgrandparents in
; .J ·1 , I

Albania for 2 weeks. He weighs 54 kg (120 lb) and is 173 cm (68 in) tall. He appears thin and pale. His temperatureis 38:2°C (100.8°F), pulse is 76/min, and r~spirations are 36/min.
Examinationshows shallow respirations with decreased breath sounds at the right lung base. An x-ray of the chest shows a right pleural effusion and hila~adenopathy. ,•1 :. ••
'
"

:l .l .:
..
" ;

l
,, I

I 1 1• :, I' .I 'I'
•I
' \1
·1
1\1\1
'' '
'
I
'I" 1 .,
Ii ' ;::i, .
'I'! ' I
:\ii . 1I'"' ..f
• :1.

.,\.I !i •: ,~
I I. II
• 1· H·
,II
I II
'1'
I . ;,, '
,
11 I' I, ' ' I . I ' •
" i"'
.•
.

- ' ' I

!II ' If •
' .. ,I
...
"
I-I'
I
' :I l~ •
I

' ' ..
38. Duringthe past 4 months,a 32-year-oldwoman has had pain during sexualintercourse.She has notedvaginal drynessduring sex despite a healthylibido and adequate foreplay. For
2 months,she has had a gritty sensation in her eyes. During the past 6 months,she has had three dental cavities repaired. She has had well-controlledtype 1 diabetes mellitusfor
10 years. She has never had an abdominal operation. Examinationof the eyes showserythemaof the conjunctivae.Examinationof the lungs,heart, and abdomen shows no
abnormalities.PeMc examinationshows a normal-appearingintroitus,vagina, and ceNix. The vaginal mucosa.appearsdry. The uterus and adnexa are nontenderand normal in size and
consistency.Which of the following is the most likely diagnosis? I ji

0 A} Endometriosis Ii
0 B} Sjogrensyndrome
0 C} Vaginismus
0 D} Zinc deficiency iI
I
I : I
'
II

l, I

I
\'!·
'
I
•I I
I.
II

l :I!1j
i 111
! .,t'I!
I :

; I' I i
I
'

1, '
.i1'1
,, . ·,l''i ..
,L • :=:.=:w: ,i:u:,
:i=..; :!;ljlil!l,lj/Jl
39. A 32-year-old man has had mild tremulousness, increasing anxiety, and progressive fatigue over the past 2 days. He had been taking lorazepam (2-3 mg daily in divided doses) for
1
acute anxiety since being robbed at gunpoint 4 weeks ago; he stopped taking the medication 3 days ago. He has a history of mitral valve prolapse, seasonal allergies, and dysthymic .'
disorder. He currentlytakes loratadine (60 mg daily) and paroxetine (20 mg daily). Which of the following:i,sthe most like~-explanationfor hfs new symptoms? ' '.il ,.i , • li •I
.. 11
0 A) Discontinuationof lorazepam ' . ti Ti
11 •I •l,. . I
I ' .'

O B) Mitral valve prolapse :



..
~
l :'
.
l• • '
l' 'I
l ·-;, 111! 11 ti . • • I : "·
q• ·,
I,
•''
O C) Panic attacks
ill! •:11 • t

O D) Paroxetine-loratadine interaction •lII ·'


'
0 E) Post-traumatic stress disorder '["' J .I
' I I : !•

iI ; j '
•:~•1 Il
I")I
I I ll
•i

J lI!
I ;:. ::II 1'
'
,
'I':. ·I:.
':
I'
1
.I '
l.
'I
..UL
,
J
" '
" I.J
. !
' '
•1

. ;
'

J

:i.'
.. ,•
:
.
I'
Ii l!I
11 l;h I
''
I: I:
J'.,


'• .

I' • I.
II • ••
l
40. A 67-year-oldman comes to the emergency department 1 hour after the onset of vertigo, nausea,and imbalance. He has a 20-year history of poorly controlled hypertension.His pulse is
70/min, respirationsare 20/min, and blood pressure is 210/115 mm Hg. Examinationshows a small right pupil, mild right ptosis, and nystagmus.Neurologic examination shows
weaknessof the right palate. Sensation to pinprick is decreased over the right side of the face and left extremities.There is incoordination on finger-nose testing and heel-knee-shin
testing on the right. Which of the following arteries is most likely to be occluded?

0 A) Anterior spinal 0 G) Left vertebral


0 B) Basilar 0 H) Posterior spinal
0 C) Left.anteriorcerebral 0 I) Right anterior cerebral
0 D) Left internalcarotid 0 J) Right internal carotid
O E) Left middle cerebral 0 K) Right middle cerebral
0 F) Left posterior cerebral 0 L) Right vertebral

,I

!'
41. A 25-year-old woman comes to the physician for a routine health maintenance examination. During the examination, she says that she is unhappy because she has been unable to find a
mate. She says that it is easy to find someone to date but that all the men she dates terminate their relationship with her, despite her requests to continue dating. She feels so desperate
when she is alone that she sometimes repeatedly calls or secretly follows her former boyfriends. She has three sisters; they all were raised in separate foster homes and are not close.
She is 163 cm (5 ft 4 in) tall and weighs 52 kg (115 lb); BMI is 20 kg/m2_Her pulse is 76/min, and blood pressure is 120/78 mm Hg. Physical examination shows well-healed, superficial
lacerations on both wrists. No other abnormalities are noted. She tells the physician that he seems like a man who would always be available to her. Which of the following is the most
likely diagnosis?

O A) Adjustment disorder
0 B) Borderline personality disorder
0 C) Delusional disorder
0 D) Gender identity disorder
O E) Major depressive disorder
42. A previously healthy32-year-old woman comes to the physician because of a 10-day historyof persistent cough that keeps her awake at night and is worse with physicalactivity.The
cough is sometimes productive of white sputum in the early morning. Her symptoms began with an upper respiratorytract infection characterizedby low-gradefever, sore throat,and
malaise 10 days ago. Her only medication is an over-the-countercough suppressant.She has smoked one pack of cigarettes daily for 16 years. She is employed as a skilled nursing
care assistant She is not in distress but has a rasping cough. Her temperature is 37.4°C (99.4°F),pulse is 78/min, respirations are 12/min,and blood pressure is 130/80 mm Hg.
Examination shows no pharyngitis or facial tenderness. Scattered end-expiratorywheezesare heard bilaterally.Her peak expiratoryflow rate is mildly decreased. ~n x-ray of the chest
shows no abnormalities. Which of the following is the most appropriate next step in pharmacotherapy? 1; ,

' h
\I
O A) Nonsedating oral antihistamine ' I I
•1

O B) Oral antibiotic I
I I lr
O C) Inhaled ~Tadrenergic agonist •I •
•.•
O D) Inhaled anticholinergic lI .. . I '
I'
'' I
I t
I II
:11 i!
O E) Inhaled mast cell stabilizer
ll
I l
1 I
I< \ I'
I
' I! \ji
II . I' II 111
ll h
I
l
1'
I
I
•l ' :r,,,•I
;

.' . p[I
I I ;t ..
'I L•' I•lI ) i t'
"I 1•

.,, l
1I 1•
1111
l

, j , If

i
..'
ii .
.I l iI
...
' . .
..
l
'I : ' i• < 'i <
• II 1
1.I ''
I

,'
)rn.. •
;. l
•I
>

I
.I

I :·..Il!I 111
• . . Il
I!\'
; Hi I '
. . l,
I
I
...l.;
i
I •
Ii
:'
,, ,. ..'
.I • . .
I
•i ., r
'
I• •
'
' ' .
• . ..
'
.
,1"
I I .
;
'l •I
'" ' • . .
~)

.
..
•I

< I.
. ! 1·1 I' I
,.j ·11··
..r,·
'\
··-
1•· • l 1
I ti~!': l: ·1
. 1j!
t,1 ,l•
••.
, '
I
..
•• '. '. >I
1
'
43. A 5-year-old boy is brought to the emergency department 30 minutes after he fainted at home after standing up from a sitting position. His symptomsbegan 3 days ago with diarrhe~and
vomiting. He has had no urine output for 18 hours. He is alert but quiet. His temperatureis 37.5°C (99.5°F),blood pressure is 75/45 mm Hg, pulse is 120/min,and respirationsare
28/min. Examination shows dry lips and tenting of the skin. There is no abdominal tenderness. Bowel soundsare hyperactive.The remainderof the examinationshows no abnormalities.
His capillary refill time is 5 seconds. Intravenousbolus doses of 0.9% saline are administered. Bladder catheterizationyields 5 ml of urine. Urinalysisis most likelyto show whichof the j
following?

0 A) Blood
' .
O B) Erythrocytecasts
O C) Hyaline casts
OD) Leukocyte casts ' j

O E) Oxalate crystals al"


. ~

.
..
r
44. A 37-year-old woman comes to the physician because of progressive shortness of breath over the past 5 years; she now has fatigue and shortnessof breathwith mild exertion.She ~as. •
a history of mitral stenosis secondary to rheumatic fever at the age of 15 years. She was asymptomaticuntil5 years ago when she developed severe shortnessof,breathduring. i
pregnancy. She was treated with diuretics, low-sodium diet, and bed rest, and she was able to deliver the baby at term. Her only medication is hydrochlorothiazide.Her temperatureis
37°C (98.6°F), blood pressure is 110/80 mm Hg, pulse is 100/min and regular,and respirationsare 26/min. Cardiac examinationshows an obvious opening snap,in S:cA grade 3/6, late· !
diastolic murmur is heard at the apex. A right ventricular lift is palpated along the left sternalborder. Which of the following is most likely increased in this patient? 1 . .
I
I'..I, .
I •

O A) Blood flow to the lower lung fields I


I
.' '
O B) Diastolic filling time I l1Jii
' ,.J! 'I '
' ii
O C) Left-ta-right shunt of blood iI'
O D) Left ventricular end-diastolic pressure ,j
' . ..
O E) Pulmonary artery pressure ,
.

!,l
jI I I
I
I I'

·11 IL

I •
i' .

'•
l,
.
'. Iii '1
II
i
'
1'
'
I I
,I
I I II .. . ti '•
'/11/
l I I • l
I

I I'II
• ..
. ~ .. '
45. A 42-year-old man is brought to the emergency department by a friend because of a 2-hour historyof confusionand difficultywalking.The patient'sfriend says .hewas·weHwoensl1e • , • ·:i,
visited him 2 days ago but found him disoriented when she came to visit him this morning. He has a 10-yearhistoryof hypertensionand has not,peen cpmpliantwith hi:5meditation •• ·, -~1
regimen. He has drunk 6 to 12 beers daily for 10 years. On arrival, he is awake but easily distracted; he is orientedto person but not to place or time. His temperatureis 37.5°C (99.5°F),· •
pulse is 90/min, and blood pressure is 140/90 mm Hg. Examinationshows dysconjugategaze and prominentnystagmus.Musclestrengthis 5/5. Deep tendon reflex~ are diminished
bilaterally.Sensation to pinprick is decreased over the distal aspect of the lower extremities..with the assistance of one person, he is ablEfto stand and walk with a wide b~~e. 1• =. j. . ,
Administration of which of the following is most appropriate at this time? : • , •• i,!li,,11
:" '.. • • "i·. , . ·:c
· ;. ; .. • I
. I, l!'I 'ii:. it •• , ••ii, ' ·,~
11t 'I • , t

0 A) Acyclovir •::,1.: 1
'.1 !,;I !l'l!11 Hli''. I 'l
''1 (' ' I ,I ,·,,,I ,, I'!''l!11t J !I'\ : :i<1
0 B) Ceftriaxone
,,' • • 'I \i !l:,,l!', -! ,:,11! ' .•
O C) Chlordiazepoxide
O D) Lactulose ' ' '

O E) Vitamin B 1 (thiamine) ,,'I j'! '; ,. ;'J


, ,,'l
, I
'. -'·,i I·,· ,. I I ',1 '1'11 l )ti' ..,'i'
'
I

l II
• •l
1;!!
'
I • '
'1· ·I'
·I • I jli
'1 I ·1,
··lfrf'
II
·ii:.
I
•..
' : j

i .; I
i. .

r if I; . i:
i
" I
!

Ii+1 · •i , ,
'
11
.. . , '
, ', l,1'. ll !i 1• !, 1 • , i
' I , 1•
'.(.i'
I
I ' •• ·1:

I i I
I

'' , I

I ' !
l

,. ' .
' ' '' ..
:'II
I.
I ,
I;
J


'' I
I
46. A 70-year-old woman has had increasing abdominal pain over the past 2 days. She has renal failure and has been receiving peritoneal dialysis for 18 months;her last tre~tmentwas
2 hours ago. She appears toxic. Her temperature is 39°C (102.2°F), and blood pressure is 140/90 mm Hg. Her abdomen is distended and diffuselytender to deep palpationwith
rebound tenderness. Leukocyte count is 18,000/mm3. Which of the following is the most appropriate next step? • '

O A) X-rays of the abdomen


'I
O 8) Comparison of abdominal fluid amylase with serum amylase activity
O C) Gram stain of abdominal fluid
O D) Ultrasonography of the abdomen
O E) CT scan of the abdomen and peMs

,,
I

',,'

I"
I ';'
\ I ] I
I
1. A 25-year-oldwoman comes to the physician for evaluation 1 week before a scheduledbiopsy of a mass in her right breast As an infant,she had four operations to manage necrotizjng
enterocolitis. She has no other history of serious illness and takes no medications. She does not smoke cigarettes. She started a new job as a medical assistant 1 monthag·o.Vital s.igns
are within normal limits. Examination shows a patchy erythematousrash over the hands.There is a 1.5-cmfirm, mobile, nontendermass in the upper outer quadri;intof the right breast.
There is a well-healed upper abdominal midline incision without evidence of hernia.The remainder of the examinationshows no abnormalities. Use of which of the followingis i1
contraindicatedduring this patient's operation?
)

O A) Cotton-containingsponges \ \ I
0 B) Cyanoacrylatetopical adhesive
0 C) Iodine preparation solution l)
I
0 D) Latex products I

0 E) Silk suture
rI .'

l
l I I
I

I•
I .I

l
.•
-1

,. •I I ,, .,\
'I •• •l-•
''
ir ,\
.,
,
l
I I I\ .1\I \'1 •1
il \•
ll
"'
\
'\;
1
.'
.' .
.,
I
,1 , r. • 'Ii. .. ,•
•)
i ·.. .,

'~
,'
l•
I ' '
•1 •

2. A 37-year-old woman comes to the physician because of an itchy rash over her trunk for 2 weeks. She has not had fever, chills, shortnessof breath, chest pain; or gastrointestinal· •

symptoms. She has a history of recurrent urinarytract infections and has been taking trimethoprim-sutfamethoxazoleprophylaxisfor the past year. She is in mi!d distress. Her temper~ttire
is 37.5°C (99.5°F), blood pressure is 96/62 mm Hg, pulse is 78/min, and respirations are 14/min. Ex~minationshows a maculoJ)apularerythemat9usrash over the trunk. ~aboratoiy.

studies show: : ! • ,. 1 •
..
I ' •
:i
.. ' ' I'

Leukocytecount 10,500/mm3
'
I
!
' ,
.
•j
I ' •
. .
'
..
I
J
.'
'
' •

'
'
\

,. I

Segmented neutrophils 72% J 11 ,··1· '. 'I ·.tl. •, 4 :\. ,'.~' ; '

•i
' ,
•I II
••
Bands 1% ,

Eosinophils 15% ' 'I, •
l:i •'I
I
II I
Lymphocytes 4% •I I
·-
..


'.'
Monocytes 8% ·1 11 I

Serum •'
Urea nitrogen 12 mg/dl .I ' ' ,. •
Creatinine 0.9 mg/dl I'

Urine
WBC 2,1.hpf
RBC 2/hpf -1
,.
,· !I,I

Which of the following is the most likely cause of these findings? ,,. ' ' 'Ii
'
'ij
0 A) Eczema !.,j
O B) Medication adverse effect ,tI
'
O C) Staphylococcal skin infection '
'I'
O D) Streptococcal skin infection •
! I
"'
O E) Urinarytract infection
I,
I
I
I I'

I .
l •

3. A 32-year-0ldwoman is brought to the emergency department 30 minutes after a motor vehicle collision. On arrival she is agitated and has shortness of breath. Her pulse is'130/min,
respirations are 30/min, and blood pressure is 100/50 mm Hg. Breath sounds are absent on the right, and heart so~nds are normal.An x-ray of the chest shows opac;fficafionof:ttie right
hemithorax.Which of the following is the most likely diagnosis? ·., . .•
l 1'l• ,.
'. '
O A) Acute diaphragmatic hernia 1/!r
I

O B) Hemothorax IIJ

O C) Rupturedbronchus •
0 D) Simple pneumothorax
0 E) Tension pneumothorax

I •• ,

.,
II
I ' .'
: .
,'•
I
I •

••
r
4. An asymptomatic47-year-old man comes for a preemploymentexamination.He has never bee·nhospitalized.He is a computer programmef, an<iihe'PlaY,S handballonce weekly:J1.is,:
maternalgrandmother had type 2 diabetes mellitus, and a paternal uncle had heart disease. Jhe patient's blood pressure is 126/89 mm Hg. Examinatior\shows no a~no,~lities ..His total
serum cholesterol concentration is 225 mg/dl. Which of the following is the most appropriate next
1
step in management? .' ,:·: :,! : ,, ' ·, i • .
11
0 A) Step 2 American Heart Association cardiac diet 11 •1 ,,.,, } • ri/i'}i,1(·,:i:· ;;i ..•·; '}'··:,'.i·:i 1
11
O B) Serum lipid studies while fasting
J' •,
.
O C) Exercise stress test :I II
Ii!
r : •• ••'
I:
0 0) Oral cholestyramineand niacin therapy I ' 11 • 1'
I
' ' " '
I'
.•.. •
.;,
I I,
I.
O E) Oral pravastatin therapy at bedtime 11 I 11
ii 'f I
,. :i I
.' ~ :;!
.•.
·'1 • • •
'
I.
u .
,. "'
,' . I 'd
··- ;I '•·
'ih
I, !-
. r t ..;•

I• '.
·~
II . I
I
I..
iiI ti
'
., I .,
' ,.
"
I' I
·I
I.
11
1:
•I

.'.
. ' .
,,
'
,1 II ,, "[
'
"

Ji''" . '"
I 11' I '
''
; I , · ,,1i
II
I I
I .J
11' '
;I'
"I
j
,I
'I I.'
•'
I I '
I ' f
-·5_A 17-year-oldboy comes to the physician because of a 2-day history of shortness of breath with minimal exertion,nausea,weakness,and fatigue. He has had urinaryfrequencyand
excessive thirst for the past 2 weeks. His father and maternal uncle have type 2 diabetes mellitus.The patient is 173 cm (5 ft 8 in) tall and weighs 66 kg (145 lb); BMI is 22 kg/m2.His pulse
is 92/min, and blood pressure is 100/60 mm Hg while sitting; his pulse is 124/min, and palpable systolic blood pressure is 80 mm Hg while standing.His respirations are 36/min_.
Examination shows no other abnormalities. Serum studies show:
" I
Na+ 128 mEq/L
CI- 82 mEq/L
K+ 6.2 mEq/L •i ., I l•
HCO3- 10 mEq/L I ' I I
Glucose 472 mg/dl l
Ketones positive
h
I
I
Which of the following is the most likely explanation for these findings?

O A) Inadequate production of insulin


O B) Increased production of ACTH
O C) Increased production of cortisol •I ' .
:
'
.,
•'

'
O D) Increased production of glucagon
O E) Peripheral resistance to the effects of insulin "
.lI •.

.. I
l
II
'.

"
'J I
II I •
Ii I
l

I I
1 11 I
~ 11i 11
r 'I
' .
[·•
"l
I

,..,,, 'I
I I
• I'"li''n·' I
"
6. A 27 -year-old woman comes to the physician because of feelings of anxiety about attending her 10-year high school reunion. She has a 2-year history of profound anxiety, palpitations, and
'
sweating associated with an uneasiness around people; she avoids family gatherings and visiting friends because she is afraid of being embarrassed. She acknowledges that this fear is
unreasonable. She does not use illicit drugs but says that alcohol makes her more comfortable around people. Her blood pressure is 130/90 mm Hg, and pulse is 88/mih. On physical
examination, she appears healthy and well nourished. Occasional wheezing is heard over the left lung field. The remainder of the examination shows no abnormalities..On mental status
examination, she appears worried. Her leukocyte count is 9000/mm3 with a normal differential. Which of the following is the most likely diagnosis?

O A) Alcohol abuse .
'
O B) Anxiety disorder due to a general medical condition
I ' I
11
II I
O C)Asthma
I
I
..•'
O D) Generalized anxiety disorder
I
'
I \'
O E) Panic disorder with agoraphobia \ , 1' ,
I I
0 F) Social phobia
I , I

\ 1 l I 1
l.11 11\
11 ' •11\ j
' Ij I' I1
l'\
,ll 1 1:, I .
1
11 ,iL .1 I
-1 1\l,1 '
)' •'

,. I

l I,
.~\
]
I! '
•1
I'
'' I 11!
I.

1
l r' ·i l ' ! I
' I \:,
,I ii1
I I

' I III I ! '


.
1!
I Ill .
I

I I 1
I ! , ,
7. An asymptomatic 32-year-old man comes for a routine health maintenanceexamination.He has a 10-yearhistoryof frequentsinus and pulmonaryinfections.He had an anaphylaetic
reaction to a blood transfusion following a motor vehicle collision 3 years ago. His temperatureis 37°C (98.6°F).Examinationshows mild erythemain the posterior pharynx:The lungsare
clear to auscultation.A complete blood count and serum protein electrophoresis are within normal limits. Which of the following is the most likely cause of the frequentinfTctions?
l ••
0 A) Colonization with Streptococcus pneumoniae 'I ·' 11 •

O B) Common variable immunodeficiency I I


O C) HIV infection
O D) Selective lgA deficiency j
l •
O E) X-linked agammaglobulinemia
1

t !
I l
I

,I l I
•I
I
I l
i•
I
r
Ij
I,
I I I
,
..
Ii
l '
• ' (


I
,)
•'
' 'I
'I

I ...
•I I
iI
I I/
• I

I
I !
d .
•I .. ii
i

1• 1 I;I ' ' '• '


. '
I

'" 8. A 23-year-old woman has pain, cramping, and swelling of the right calf 3 days after an uncomplicated labor and delivery. The right foot is swollen, and there is marked tenderness with
·,·· dorsiflexion and palpation of the right calf. Examination shows no other abnormalities. A complete blood count and serum electrolyte concentrations are within normal limits. Whi~h of the
following is the most likely cause of this condition?

O A) Hypercoagulable state of pregnancy


iit I I I
O B) Hyperuricemia I

O C) Peripheral artery aneurysm ,.


I I I' l
'
O D) Platelet embolus I • j \
'
O E) Prolonged pressure on the vena cava during delivery
I • I
I

.
I ,
,! I I '

I .I l •
I :1

! ' l·,
I. 'I •I
!
'
'

.. '

I .l l I
.I I I • ' f I •' • f I (
1 I • I
' I
..
l • '
I I I
' ..
'• I ' ' .
■ l\.1ark Co1nprehensive Clinical Science Self.Assessment 59 n1in 3 sec
......... - '- 4 ·--~ _;:_ •••:"!aL....;:,,,..~,,;;"'""~ _.,. -~~- ==•ktH1IU:.IRM.lil"f• ~ It! IU1UUll1U!!rnIT1H;, l lltUIJ"allltuH,iuuu,nr'!l:~J;:Tn tHUUCWUU'1UUCWUUU.»J;UUIIIUlfUl,U:lWJ~HUUII.UUUUnJI\! 111 n111n111u.rnn I UI II U~t lt.tl0~'1't~~~J1 l UU.irmiTiii"iO I U 1111 Hl ;,.JjlJ l> • C,)ll -~ ,u,; 1< ~· · •• • 1 ; • •; • '

,. .
I ,, ,:l.
11
,
'
' ' ..
I '

I . !'l. -

~
j

i .! ; ' ' I 1 l • • , ~· •
1
9. A 21-year-old college student comes to the physician because of acne that developed 4 days ago while'shewas taking hermedical dollegeadmission·test She is concernedal)outher •• • ;:j
appearance and plans to be in a wedding in 3 weeks. She has had similar episodes that have resolvedcompletelywithouttreatment.tExaminationshows acute acne over'thef~c~ witha 1
predominance of comedones and pustules. There is no evidence of chronic scarring. Which of the followingis the most appropriate initial step in treatment? . 1 1 ' • l . • 1l '. • : "
,, I It I I ! :\,1 ' • ,' I • : . ·,
,'
I
l
I
I

"I
I

\
.1 i
I j

·1 •
I; ' l1:
'. . , :r i,,. ,., 1·: •
O A) Dietary restriction of chocolates and simple sugars ·1· ,I' '.I • I 1!1 1'
,l )I . :1· I I }' • ;, t\l\Jlt:I Pl 1, ,·
O B) Dietary restriction of milk products
O C) Topical acyclovir
:11 l ,' [':I 1' :i l '•)I
II I
1! i1l1l1\1'll
l!
.r , . •:
IIIa ,
: ,. I , ....
! l : •.• ,l!\1. , , ,.,
O D) Topical hydrocortisone cream , !j .11 \ (i 1
l\, I l ',
t 1l•l
1
• '~ ',
O E) Topical retinoic acid I' '
tI ' :I
I
l I I I ,,
•,t•
l\1
I
1
:
: , •.
,i'!l , ·1,•\l1' i:•111,·11··'!!1··
11
! 1
·'1·
1 ,
1
Pi
1 ••
1
•: ll I l I
· .,i
: 1 1 .' ·I ,'. :i:' •1 :1
.I i1 •l
,1 \:tlr.i ,
ihHP t
1. !' 1:rI 1 ,
I , _: :!r:
1
.i • •.. •
11
:\

'I I I 11 I 'i 1 ' I ,I\, \<l\i d I•

Il
' I
iPH1 ' '~li'j •

d : '!
. -ill 11\·ti
'1 I 11
i', li '•
' ··,1 . I
I ; •
II 1
"
'1
I
_,.11

' I
I! ,l, li I I.: i'
' 11 • J!. • ,,. 1·
I 11·11
l. i1 q :' .
n ' ...
iI
I.
>

.
I

I I I I
I I f'•
I . '

I
' '
'•' .
11
l!.
I
•1!
..


10. A 5-week-old boy is brought to the physician because of vomiting for 3 days. Switchingfrom a cow's milk-based formulato a soy-based formulaand one bottle of an electrolytesolution
has not decreased his vomiting. His mother says that there is no yellow color to the vomitus, but it is forcefuland occurs immediatelyafter he has had 1 to 2 ounces of liquid, He appears
to vomit more liquid than he drank. He has one mustard-colored seedy stool daily. Examinationshows no abnormalities.Which of the following is the most likely explanationfor his
vomiting?

O A) Duodenal atresia
O B) Gastroesophageal reflux
O C) Hypertrophic pyloric stenosis
O D) Lactose intolerance
O E) Protein malabsorption
O F) Rotavirus infection

i I

l I
II '
i1' " •I
'• 'I !:
••~••-••• • • .--. .. ••••••~1•••••~•••••••t1t••~•
. .. • .. ••••!j'''l 0
-•--,.•:••1••l
11. A 62-year-old man has had the gradual onset of fatigue and shortness of breath over the past 3 years. There is strikingjugularvenous distention with a large wave occurringwith S2-The
carotid upstroke is normal. Cardiac examination shows a lifting systolic motion of the sternumand no palpable point of maximalimpulse.A grade 3/6, holosystolic,plateau-shaped
murmurthat is loudest on inspiration is heard at the lower left sternal border. The liver is enlarged and tender, and the abdomen is swollenwith a fluid wave.There is marked ankle
edema. Which of the following is the most likely cause of the murmur?

O A) Aortic stenosis
0 B) Mitra! regurgitation
O C) Mitra! stenosis
O D) Tricuspid regurgitation
O E) Ventricular septa! defect

"
'I
I'
'I
~ I
\+ +
il,1I
t•

12. Four weeks after a low transverse cesarean delivery for cephalopelvic disproportion, a 27-year-old woman, gravida 1, para 1, comes to the physician 9ecause of a pullingfeeling on the
right side of her incision for the past 4 days; the feeling is exacerbated by movement.She was discharged on postoperativeday 3. Within the past 2 weeks;,she has.initiatedan exercise
regimen to get back into shape and has resumed sexual activity. She has been breast-feeding and plans to continuefor another month.She is 168 cm (5 ft 6 in):talland weighs..
82 kg (180 lb); BMI is 29 kg/m2. Her temperature is 37°C (98.6°F). Examinationshows breast engorgement.The abdomen is soft and nontenderwithout rebound in the.right·andleft
upper quadrants. There is mild tenderness to deep palpation just lateral to the right and left aspects of the abdominal incision.The surgical wound is clean, dry, and intact.The uterus,is
nontender on bimanual examination. Which of the following is the most appropriate next step in management? 1 1
;1i,
1, 1: .:•,1 ,, ' •

'1 • II J· r•II ., ' ' .•
••
O A) Reassurance I l'. = ,I ,·' '/ •
O B) Discontinuation of breast-feeding
1
11 !:, .!il; , . !
O C) Ultrasonography of the incision site .! J'
1!111! ;!iL·! ·: ' ,l
I
;

0 D) MRI of the pelvis


'
;
O E) Intravenousheparin therapy I ' • I ;

O F) Oral broad-spectrum antibiotic therapy )l


' •'
'• •
'·1•.
,I I
i,
I ' I

.
/I
"!'
, >I
11
,, II

., .'

'.
13. A 62-year-oldwoman comes to the physician because of generalizedweakness for 2 weeks. She has a 20-year history of arthritis of the hands treated with aspirin and acetaminophen.
5
She had two episodes of urinarytract infections 5 and 11 years ago, respectively.Examinationshows no abnormalitiesexcept for Heberden's nodes on the hands. Laboratorystudies
show:
Erythrocytesedimentationrate 15 mm/h
Serum
Na+ 136 mEq/L
CI- 100 mEq/L
K+ 4.9 mEq/L
HCO3- 20 mEq/L
Urea nitrogen 41 mg/dl
Creatinine 4 mg/dl
Urine 11

.. Protein 1+
WBC 2-4/hpf I
RBC none I
I

Bacteria none
Squamousepithelialcells occasional
Granularcasts occasional
.,
I I

Renal ultrasonographyshows no abnormalities. Which of the followingis most likelyto have prevq?nted
this conqitibn?
,: .
'I., ,II

0 A) Periodic PPD skin testing


0 B) Screeningfor autoimmunecauses of glomerulonephritis I. :,l
'"
,Il'' l!
WI Ii
0 C) Periodic renal ultrasonography 11
'I I 1.1

0 D) Avoidance of analgesics ,I I ,,1,I' I 1:1 '•


I"
,, t~r
,i

0 E) Suppressiveantibiotic therapyfor treatment of urinarytract infections I


' ii r, 'I
'II:
I
,. I •
.
L. :' ., ,ii I
' . .
JI

14. After an uncomplicated laparoscopic cholecystectomy, a 62-year-old man has not had any urine output since the Foley catheter was removed 12 hours ago,1During,the hour before the_
operation, the 40 minutes of operating room time, and the 2 hours in the recovery room, his fluid input was 2.5 Land urine output was 1 L. Since that time, he has been receiving ••
intravenous 5% dextrose in water with 0.45% saline and morphine. He is awake and alert and has a moderate amount of abdominal pain. Preoperative serum studies showed: •

1 \\'
Na+ 137 mEq/L • / f Ii , 1 1:, t Ii
K+ 4.2 mEq/L.
'
Urea nitrogen 18 mg/dL1J , ;!1 1 \ : t· ' II
' I l
Creatinine
"•' I
1.2 mg/d~!· ;J
I
'
\
l . ..
1

' ·1·• 1
,,.\,, I '
'I
' I ' I ' t I
• ''( . ' : ~ • il I '
1
One hour after receiving an intravenous bolus of 0.9% saline, the patient does not produce any urin~, Which of ~~'efollowing is the most appropriate next step in management? , ;! .1, •
·,.' . 1·''
"
. !
.(.t·11•·'
.. . l
I
0 A) Increase in the dose of morphine •I '.
\
O B) Intravenous administration of an additional bolus of 0.9% saline
t 1' \i •• I .. I
0 C) Intravenous administration of doxazosin .I '
0 D) Intravenous administration of furosemide I
'
.. I .I
I, I I I .

O E) Reinsertion of a Foley catheter II '


' I' . \
I\

I I' ,l
I ·l
•l I \

'
I!
l
iI
I :
I f
,\
1 ..
\ 1 ,,
I l \

,I 11

I1 I 1
V
' '.
I I ; .,
·J
•i
I ~
!1
15. A 67-year-old man is brought to the emergency department 4 hours after the onset of severe midlumbar back pain He is anxious, pale, and diaphoretic. His temperature is
37.1°C (98.8°F), blood pressure is 105/65 mm Hg, and pulse is 120/min. Examinationshows no other abnormalities.X-rays of the lumbar spine show degenerative disc disease with
calcifications anterior to the vertebral bodies. Which of the following is the most likely diagnosis?

O A) Aortoiliac occlusion ' '


O B) Herniated nucleuspulposus
0 C) Lu.mbardiscitis
0 D) Lumbar strain
O E) Pyelonephritis
0 F) Rupturedaortic aneurysm
O G) Spinal stenosis

I'

I.

t '!
'' ' '
,

16. A 35-year-old man is brought to the emergency department because of intractable nausea and vomiting of nonbiliousfluid over the past 48 hours. He has a history of duodenal ulcer
disease treated with Hz-receptorblocking agents. His temperature is 37°C (98.6°F), blood pressure is 90/60 mm Hg, pulse is 130/min,and respirations are 10/min. Examinationshows
mild epigastric tenderness. Which of the following are the most likely serum electrolytefindings? 11 . ; . 1 ;\
1
Na+ Cl- K+ HC0 3- lip
·F:
(mEq/L) (mEq/L) (mEq/L) (mEq/L) II \l :, [li
I .
OA) 115 80 4.0 ll
25 I,
0 B) 140 80 2.5 40
liH
.,
•: !
0 C) 145 100 5.0 15 I
I

0 D) 'I 'I !i
150 105 2.5 25
0 E) 160 135 5.0 25 I ! !.
I
I
I
I
I\
•I
? I:
I..l '
I I
'

lI I
I
'. I
II
• 'H
' ..
• ..
...
1
.
,1•
,t
·;,,.
i •
' .
I ... .• ..\
• '!
•! '•
,j
I
,r .. ..

Il,,r·I I
I

l I l
·•

'•
I,
!I !
. ...
•• I
' .
.t• •j I !.
Question llra'1!1 Player I :
I

, I
The item has associated media thatmay require-the.:~~~;
9,t~~aqphorjes : !\
t\1
L
1t 1·1
l'!
I•
I
'·1I • 'I I
.\ li ' ' • 'I
' l ' I ' .!\, I 't ••
17. A previously healthy4-year-old boy is brought to the physician because of a 3-day historyof fever, cough, and runnynose. He has not had wheezing,vomiting, or diarrhea, He•is at the
75th percentile for height and 10th percentile for weight. His temperature is 37.5°C (99.5°F).The skin ls warm and pink. Capillary refill time·is 2 seconds. Examin:ationshows clear , ;.
rhinorrhea. Breath sounds are normal. To view the cardiac examination,click the "Media Player"tab. The remainder of the examination shows no abnormalities. Which of the following is
the most likely diagnosis? '
''
0 A) Congestive heart failure lt! I
!II • I I
.,
• 'l
0 B) Idiopathic pulmonary hypertension ''
ll• ii'• I' I I •

I
'1·,
O C) Pericarditis I ,.

I
l
'.
'' i
'I

O D) Upper respiratory tract infection


·I'
11l•
•I
I
I
• \' I • '
'

Ii
O E) Ventricular septal defect ti •
'!1 I •I' I I I
11 I:I • ! •
I' I'
' I ,.. ••

I,
l
I
II
>;
:i
ii

.I
I 1
.
'
I I, J·•
'I:
.h
'. 11 II;
'
•l '
I l I !1 I
11
I

l'.
I 11
!II·!!
'!:'

I I
u'i1.
H

I•
I ••
I'
I
'
·l '•
II III
i:
I
I: •

·1 I ' I'
'
' I ' 'I
.I '
'
!
18. An 8-year-0ld girt with type 1 diabetes mellitus is brought to the emergency department 1Ominutesafter being involvedin a motorvehiclecolfision.She was in the back seal of a smaJ
automobile that was rear~nded. Initially, she was alert during transport and reported bilateralthigh pain, but then she stoppedtalking,closed her eyes,and became unresponsiwto
voice; on arrival, she responds to noxious stimuli with brief grimaces and no withdrawal.Her blood pressureis 40/palpablemm Hg, pulse is 148/min,and respirationsare 28fmin.Air
entry is symmetric. The pupils are equal and read to light No cardiac murmuris heard.The abdomen is soft. Thereis swellingof the upperportionsof boththighs.Her hematocritis 37o/~
Which of the following is the most appropriate next step in management?

O A) Measurement of arterial blood gases


O B) X-ray of the chest
O C) CT scan of the head
O D) Administration of 50% dextrose in water
O E) Infusion of 0.9% saline
I 1 ,
I ,i I '
l • • •

19. A 13-year-old girl is brought for a well-child examination. Menses have occurred every.othermonth since menarche 1Omonths ago. Her last menstrual period was 1 week ag~-:Sh.eis f10t,
sexually active. Sexual development is Tanner stage 3. Examination shows no abnormalities. Which of the foll9winglis the f00?t-appropriateinext step in management?.« • l • • .'l • ·: l

II
i \I ' \11 I> ' I1·) \t I . I-~I . ! _:! ' ' '
O A) Discussion of pregnancy prevention
'1 I
_,'\j!
i
t,

;·-'1'
1
,, 'L1
I ',.
1 ',

\' 1 1 ' · ! l , 'I • ·1


1, 1i·';,' ,''1·'
· il ' ; :
·.,. ·, i
I

i'. •.•. • •,::


\ •
: •• .
ff:.,. .;

··t
O B) Measurement of serum luteinizing and follicle-stimulating hormone concentrations ..
• I J •
I ' I I • •• •t •
I 11
II 1, I \I 1 .1 1' ! I I \ I l' ' ' •,, , J 1••
0 C) Urine ~hCG test l 'l I . ., •I 1'. ' ! ; i : I 1\ l ' ,:· : :;
O D) PeMc examination ! I I .• : ,

O E) Estrogen therapy I l
I ' '
I
!l . I Iii l •
I.I l ' ,. I \ l I I ' ! •
Ill
O F) Diagnostic laparoscopy l\ll •. , 1ir •.·, 1. l'
1
'.I 11'! 1•.ill'.' ,'
I
11 1111 ,!1!\, 111 ;

•• I
, I " Il •I ·l I I !
'
lI I
..
I 'I
I .II I
= I !
'' I
I lI I
,I
I . r I •

I' I' ! I ••
I
.

.,
I
I
I•
1•I j'

.. ,l '
• I' !I i'' •l
II I' •t . I
.

I I
I

[ .I I ' i '

Ii I
'
I t ·l
.
I l
' I
I •.

t
I ;
•• I
t

20. A 32-year-old woman, gravida 2, para 2, comes to the physician because she has been amenorrheicfor 4 months.Examinationshows a well-estrogenizedvagi~ and no evidenceof '

I I • .
.! •
..i
virilization or other abnormalities. A serum pregnancy test is negative. She is given medroxyprogest~roneand has the onset of bleeding 3 days l~ter.Whi~hof the f91Jowing i_st~~.most '/ 1
' :
likely cause of her condition? 11 1
I. , , ,. 1• •• , • i '11 • , , '

O A) Anovulation Ii
.
·.' /l,; Ii 'f 1 "1
f..
. I I
• -II

O B) Asherman's syndrome ; 'i/ ·11r "' . ,.


.
: .
1
0 C) Hypopituitarism ! I 11 ' 0

• • , 1
!I' f •

OD) Menopause I 11.IJ/ilti.' h


l .I ,! ih! f ' l
O E) Premature ovarian failure .

. I

... ~
. I· . I II
I; I
', . "I•I :I I I•

' I • •'


.,.
•I : I •; I

III 1
I
'I
I/ •
i
I l,

r
;:1 1
I I, I
I
I I• lji . • I ,f •
' t l.'
•• ••
I 'I I . I
I
'
I
l 'I
I/' t If,. :I
I
'I ' I
I
J
' '
I , • I
I I
I 'I I i f l
J I.
,1 j
II 'ti' j
I I, , I '

I .. •

I!• ,l

' I '
' II
I·'
I, I // ./. I I) ,;,
t •I I '
I
·,
'1
I :
j •I '•• .
''
I
·Ii .•I
"
.. ••
21. A healthy21-year-old college student comes for an annual health maintenanceexamination.She has neverbeen sexuallyactive. She does not smoke or use drugs. She drinks one to two
beers each weekend. Two years ago, her mother underwenta mastectomyfor breast cancer at the age of 54 years.The patient is 165 cm (5 ft 5 in) tall and weighs 57 kg (:125lb); BMIi.s
21 kg/m2. Examination shows no abnormalities. Which of the following is the most appropriate screeningtest for this patient? ', • . ·' ; .
ll/ ' 1:
0 A) Pap smear ·Ii .,1 > n I
0 B) PPO skin test 11 .' • .
;

I '-

O C) Test of the stool for occult blood ,
O D) Complete blood count ' •I l /1•

O E) Urinalysis l. 'II
•' 1,!!f
1•
• i;

I lill ' ..
'
,
'
:1.tI
ii
O F) Baseline mammography h I " ..
,
I
1 11 '
'•
: i' ij I
II .. ' • •
,,
.• I /11 1,. ; I,
.
' ,I I '
'.II j'

.. •
l•
'
..
. I
' I
'.
'. I
I /!
f'

.,..
''I'
''
,I

'I
I
I

1/.
; .I I

.
.i
.
•• -
: ◄

•' : . ; I
.. . I I,
. · 22. A previously healthy 52-year-old man comes to the emergency department because of hiccupsfor 1 week. He has smoked two packs of cigarettes daily for 30 years. He does not drink
alcohol. He is alert and oriented. His temperature is 37°C (98.6°F), blood pressure is 150/95 mm Hg, pulse is 70/min, and respirations are 12/min. Physicaland neurologicexaminations
show no abnormalities. His serum sodium concentration is 120 mEq/L. An x-ray of the chest shows a right hilar mass. Which of the following is the most appropriate nextstep in
treatment?

O A) Bisphosphonate therapy 0 I) Hydrocortisonetherapy


O B) Calcitonin therapy 0 J) Lactated Ringer's solution
0 C) Calcium therapy 0 K) Mannitoltherapy
OD) Dexamethasone therapy 0 L) Potassium therapy
O E) 5% Dextrose in 0.225% saline therapy OM) 0.9% Saline therapy
O F) 5% Dextrose in 0.45% saline-therapy O N) 3% Saline therapy
O G) 5% Dextrose in water therapy O 0) Sodium bicarbonate therapy
O H) Fluid restriction

. : 'I I
I
'

I
II
'i j I I i
'

I ..
I, J
iI'
I I

' . 1 I·;ri •
II
I ,I
1 iI ;
f
I
1
lli 1
I ; 1 • i
.'j·
·,• ;
l I
,: p I
I •
, I,
'
I
r
I

. , •I : I
' ; : ;
· ·1 !1 •;JI
1
23. A 22-year-old woman comes to the physician for a health maintenanceexaminationbefore beginningnursingschool.She has no historyof serious illness. Her mediciitions are lorata<1ine.
for seasonal allergies and an oral contraceptive. She received the three-dose series for hepatitis B vaccine 3 years ago. Her measles-mumps-rubellaand tetanus-diphtheria-acellular
pertussis immunizations are up-to-date. She had chickenpox 17 years ago. She does not smoke cigarettes. She drinks up to four glasses of wine weekly.Examina1ionshows,no
abnormalities. Serum studies show: , , I! , '' • i
•'/ • • ' I
j1t/ !, /, r
Hepatitis A lgG positive
I

I I,' l
I

Hepatitis A lgM
HBsAg
negative
negative
I I: !i'i.I •

11[ I
i

, I,
'

I I· • { !
. ,1
HBsAb negative ,··,. l f/
Hepatitis B core antibody negative ,IJJ /1/ :: 1; i/!/
..
Hepatitis C antibody negative 11 1
'.
I :
J: 'i't/ 'j t , I'
11) k1 l
'j,/ •i!!/ ,I
Which of the following is the most appropriate next step in management?
III
I
I
i
I 1·
I
!11•.
t'•· ,;: Ii1!/ I
r !
I ;l l

O A) Complete blood count with differential ,l..j '

O B) Measurement of serum a-fetoprotein concentration


lll 11 • lI l
l I..IIl
11 j l
O C) Ultrasonographyof the abdomen I
l,, I' I
11
O D) Administration of hepatitis A vaccine III
O E) Administration of hepatitis B vaccine I
O F) No intervention is indicated at this time I . rlllll III
I • !11
I,
l . ' '
'I
.. ll l 11/
I I ' j l
'ii . I ,, I I ' .. • I '. . • I
.J I I j t ' l l ;1
' ,, rl ., . 1

"I i· 1//i'!fl
•l

.,.! .
n
.

lj• I :1' 1rili :j •

' I
I I //I /,,'I. I I •
11
//
1'f j
rfii
I I•
•'
II I ••
'•1
' •

' .
,. •I• : 'I • •1! ! •1' •. •
,1• .
·"
11. ' , !'. !•' 1
,
• ,•:
II +1 1
:1J' :J... ~ ' !:. .

I II II
111 ·! 1
11
I I " 11 .I
I
'I
'•
I

.
•• ,:
'·' !
I '11 •
'•.
+
:
••
J
'
,' ,:
IT·,l:t,'

,,,
' I
••
!,,
t,

24. A previously healthy 16-year-old boy is brought to the emergency department 20 minutes after an episode of left arm shaking that lasted approximately 3'miryutes.;0.,er the past2 days;
he has h~d fever ~nd_emotional !ability. On arrival,_~is temperature is 38.9°C (102°F): He is somnolent and disoriented t~ p,er$on, plac~. an~ tim~. t-1~ r,~ponp~ poo& .t~:pain,.
11
Neurolog1c exam1nat1onshows no other abnormalities. Laboratory studies show: ,. , ' , , , 1 ' 1 ',. • ,, i;/;:
1 .. • ' , : .,
•• 1
': _: 'f:.,: ; :,,
1 1
·t 34'¾ ii' .! 'I !· . ;;, ,
Hema t ocn o, ,,
·,I ·1 ,, ·/ ,, '
, 1,
I•

11 ·, • . , ,•,.
; ''
.
II •
1 •• 'i:','' : 1 i,. b,·.
1
,,··,, -•1 , I I I : I I I,

Leukocyte count 6000/mm 3 ,, / ,/ • '1 1· 11 J '


1 1
, ,I,. '',.n; ;,_ ·,. i:,; •
1 :, /' • 1 !1• i:;.·,1 1 1
r : ·~ ••';' '1 :
1 1
1 ,."i. 't· .:::r·,r~?
1 11

Segmented neutrophils 50% f II 1 1/; ii/ '; :! '; /,,w.,;, ';;,',' :;.:=.\ ·.,, •

Lymphocytes 50% / " '


,I • .,
3 11 l .I ' ,,.'
Platelet count 280,000/mm .. ' II/ ' /
I
I
. . 1;=·1
'', ...• 1•• ·,r
1
'I I, II ' I ) I I
:/ ' ' 1/1I fr
I • ·.' '·
11 .
Analysisof cerebrospinal fluid shows: I
. .
I ·1
1
I
1
,.
II '
• I/ (I
,,
' 1 ·,11·
I
/· ;:! 'j• •:
. i· •
f

,.r .
II , 'I ··: ;•;t
.• ' 1
.' .
Leukocyte count 120/mm 3 '
I ' '

Segmented neutrophils 20%


'
Lymphocytes 80% I
I.
I I '
Erythrocyte count 300/mm 3
' I
,, I ,,
I
I ' :1i·'
I I • ..
Glucose 60 mg/dL I I •I
'I
•· I:
I
I• I l, •' '
•J
Protein 400 mg/dl
'
" Ir '
• /I:
II
'
I I '
"·1 I I' I
I ,
..
Which of the following is the most likely cause of this patient's neurologic findings? I• I
,I ~ '/,
',, ' "' 1!,
I
: I
I
:,
II
I· •
I I,
''*'' '
/"
1) •!
O A) Bacterial infection ii
II1, I
I .
I .f'I
I,

O B) Congenital malformation •)
I '' I I I
I ' I
''
I • /J
I ,, II " .
0 C) Fungal infection ·/ II '

0 0) Hemorrhage
'I

O E) Immune-mediated demyelination ,, •, I ;c
' I
O F) Parasitic infection I '
f
O G) Viral infection
25. A 27-year-old woman comes to the physician because of a 2-year history of intermittentdiarrhea and severe crampingabdominalpain. The stools are watery,occasionallyfoul-smelling,
and nonbloody.She is currentlypain-free and has not had diarrhea for 2 days. She also has intermittentconstipation.She has not had fever or weight loss. She returnedfrom a trip to
Mexico 3 months ago. She had an appendectomy at the age of 12 years and a cesareandelivery4 years ago. Examinationshows no abnormalities.Which of the followingis the most
likely diagnosis?

O A) Bacterial gastroenteritis
O 8) Crohn'sdisease
O C) Intermittentsmall-bowel obstruction
O D) Irritable bowel syndrome
O E) Laxative abuse
' lj • I •
I ' 1 ' I' I I • ' ' , I ' ,.
I . , 11 • I •
26. A 24-year-old woman comes to the physician because of constant, severe pain in her neck, shoulders:and back for 3 months.She has been unableto ~njoy her usualactivities because 11
of the pain. Use of over-the-counter ibuprofen and aspirin has not relieved her symptoms. She has a historyof irritable bowel syndrome. Examination:showsmultipl,etender SJ)Ots over the
neck,shoulders, and lumbar spine. Range of motion of all joints is full. There is no evidence of synovitis.Fluorescentserum antinuclearantibody and rheumatoidfactor assays-~e '1 1 l
negative. Which of the following is the most likely diagnosis? \ . , 111 , \ , ii ' .',, i 1 \ ' .' I
O A) Ankylosing spondylitis
1
[ \ 1i ,\i\11
1
l '
1
I \\' \ 1
1\ \ '
1 1
:I·; \ 1• ,.••\''i'. •
i
1 111 11 I ,
O B) Fibromyalgia 1 1 l ,\ ' • \
11

1 1 1 11 I
O C) Polymyalgia rheumatica I 1I J \l I 1\ \ ' 11 l • t \t \1\1 1 \\1\\,i\l \ \
0 D) Polymyositis l I1 \I I I I ' I (11 I I ~ '\11'\\i.' 11 i \ I i • • \ \
O E) Seronegative rheumatoid arthritis ,1 I I 11 I1'\ 1 \ , ,'
1
I \, :i ' •

I I 11
1I
\1 \I
I I
't\
I' 11I I
I 'I
I I

I
\ I l, I

II l I I 11 I I !l' :i
t I I 1\ I
I l
I
1
l lI 1 l1 II 'I I

•1 I I

Il
I'
I II I I: ' I I' I I
il ·l ' 1' l l l

l,III
11
I
II I I I I I l •
11
,J' I
,
I•
'1 , \I
1
1
' I 1
1
11, \.',11 ·\:;
I
.u.iwiwawmi~~~~
27. Five weeks after vaginal delivery of a healthyfull-term newborn, a 22-year-old woman, gravida 1, para 1, is broughtto the physicianby her mother because of depressed·moodfor
2 weeks. Her mother is concerned that her daughter is not able to take care of her infant. Physicalexaminationshows no abnormalities.She is quiet and tearfuland does not engagein
conversation easily. She states that she lives alone with her infant and has had thoughtsof suicide and infanticide.Which of the followingis the most appropriate nextstep in
management? '
ll I

O A) Reassurance t•
O B) Long-term outpatient counseling t i\ , l
II
O C) Anti psychotic therapy
O D) Selective serotonin reuptake inhibitor therapy
O E) Admission to the hospital for treatment
I ;

II
I
l

I I I iI
28. A 7-year-old girl is brought to the physician in September because of fever and sore throat for 1 day. She is in the third week of second grade. Her temperatureis 38.6°C (101.5°F).
Examination shows an erythematous pharynx and slightly enlarged tonsils without exudate.There is no significant cervical lymphadenopathy.A rapid test for group A streptococcusis
negative. Which of the following is the most appropriate next step in management?

O A) Monospot test
0 B) Throat culture
O C) Intramuscularpenicillin therapy
O D) Oral erythromycin therapy
O E) Oral penicillin therapy

I .
'
''
'1'
..
• ,;
. '•
29. A previously healthy4-year-old girl is brought to the physician because of fever and refusalto walk for 1 day. She appears mildlyill. Her temperatureis 38.6°C {101.5°F),pulse is· ·:
120/min, and respirations are 22/min. The right knee is erythematousand swollen.She holds her right knee in:nexipnand resists any attemptedmovementof'her ri~htleg. S,hecri~:when r,
the right knee is moved. Which of the following is the most appropriate nextstep in management? !Iii;
1
• • ,;, , •_, ,.·;!, , 1
11
1
;1 • !ir H 1·\ ' '1'!;·,··11,,:i.
, ; :.:' • •1
O A) Acetaminophen with codeine therapy O G) MRI of the spine 11 ':
1
1 1
1 lii I· I ! · =I 1::,,i:h,
1
: ll !1:!,:_. I;, l
o B) Arthrocentesis O H) Physicaltherapy i ,:· 11,

; lil • 1l1
1
!i
' J
,:, J:'1
iI
lli·i
,I·
f/j!, 111'
'!;i ••.•
•i;I • __ i:. '·· i
1 '.
I ll
O C) Bone marrow aspiration 0 I) Reassurance t'J', ·ti :1 •

• ,. llil1- P;1,, · ••. li :, • i , ·-:'


0 D) Bone scan O J) Serum rheumatoidfactor assay r. I, I!I I 11
i ' 11/f.i'
1
,!Ii I ' .i',, !i ' I i l . i ; 11
O E) Immobilization and traction O K) Systemic antibiotic therapy " " ,l'i1l'j'11• •. I !h
1 I • ,t 'J
J

I l !Llj • I·· I
0 F) Lyme titer I I ,1j 1 ;'!::;' <,•1' •.
I
,I l r111 I • 11 j Ii : i l
1

•. . .·1
I ';

III I I !; ll.· \il


I I ·, :.
''
•i :

I • 'I
'• I ' :, i 1,'1il I I'

, '
I
I
I 'H • li . l I

I I II
. :

Ii
1
,
l t/•
' •

',!
l ' I

' Ji •I l' I J . i
I :j j• .
II'
,I I
'
·1 '
..

IJ 't•
.•I l I;
t•
I
I
'I
h ;I , ! .
' ! I .r .
' _.I
'
Hi•
I,
J •
' I• I l
111I : I I
. I,
'
fIi I '
' I : !I . ' .' '

II !,
•J . ..l }j
,.
J

! I • • -!I
I

' '
;1·•-,c·r1·:; ..
' I
.I ': I I'
. •
' :: ' '
• >
II I: I
' •
I ' '
·I '

30. A 52~year-oldwoman comes to the physician because of difficulty sleeping throughthe night and concentratingduring the d~y si'nceh~r son gradu~te~from high.sshc:>912 mo~ths~go,Pi!· ·.:,i
that time, she and her husband found a comment in their son's yearbook implying that he is gay. Theywere upset i¼ndconfrontedtheir son. He confirmedthat h~ is gay and ~aid he was!),.., 1,
surprised that they di~ not ~now. Since then, she has had constant scenarios runningthroughher mi~d that (nvolveher son and his friends. She occasi~nal~ feels s_hort0f b~fath,and:_! .. 'i(i'./
senses that her heart 1sracing. She has taken a 2-week leave of absence from her Job,and she avoids talking to family members. She saysthatshe still enJpysgoing oµt:w1th,~er,1,: .' ;,),i:n
husband but starts to feel anxious when she is alone. She has no history of serious illness and takes no medications. She drinks a glass of wine with dinner most evenings.'Physi~I 11 ,/ •1 •'.!
1
examination shows no abnormalities. On mental status examination, she appears mildly anxious.Her thought process is logical and goal directed. She is preo~~.upiedwith,her.~pn's1_ ! . i, .;
well-being. Which of the following is the most likely diagnosis? 'I
,
'I ,
I I p'' J'• , . : J, : ' 1 1-1 ';i•-;1,:i d
·11 j. I r . , , .. i ;. •
',t'j ' • '~ • • I lfl
I ·•1 ,
Cl
I ..

. l ' • ,.,
l
I
• . '

O A) Acute stress disorder I ' j Jj I fl'·1·'I'


I :1 ' ' ,· . '·,
"'··:l•
, l·
i I. ,!,'1 I
O B) Adjustment disorder with anxiety ·l , , i)!j 1 1 l, l
' 11 ' '' i· 1;;11:
l f
; '..
;·: j
I
11
·:
;
,·i, ., '!I
•., •
I
O C) Generalized anxiety disorder !'
I I
,I ~ l 1 I
p li
' .:
; I •
. 1

O D) Panic disorder 11.• ,l l 'I . I 1J 1:1/;


'.•,; ! ,;; •. f,f
O E) Post-traumatic stress disorder 11
• jl ·,; '• II 1·:, I:. ;· :iiii' '-'.1 :, . ; ••;:
J
O F) Primary insomnia i' 1·1· 11 i::
, I I I ..
1/:'i, l/, I_ •
lI l I
I i'
II
: I
1 1I .I ' ' ,.
'. ll • , '' ' .• I

:II Ill 11. i:j


1jil]::! !//i Ill)' • : , •'!' '
l I
11
'r
/,·
·.,. . , •.. • I:'
I . ••
.

'

'
I . , I IJ. Ii I:
' l
I,
t ,
,' . .,
'
'.
' •
, :I
j
I1 !I Ii ' •'
'

j ,
;
;i. • : • -~ - • ' ~ - j!

tTI1r
+

re~po~se options for the next~ it~m~;ar~ the.:$ame, Select one answer for each item in t~e set

I
For each patient with papilledema, select the most likely diagnosis. I
j I • . •
0 A) Bacterial meningitis I

O B) Cerebral infarction
O C) Cryptococcal meningitis
•1 •
O D) Glioblastoma multiforme
O E) Herpes simplex encephalitis
I II
O F) Hypertensive encephalopathy
0 G) Idiopathic intracranial hypertension
O H) lntracerebral hemorrhage
0 I) St. Louis encephalitis .•.

31. A 25-year-old woman comes to the emergency department because of increasingly severe bifrorital headaches over the past 6 months. During this period, she has had transient
episodes of blindness lasting 1 to 2 seconds. She has not had nausea or vomiting. She has a long-standing history of difficulty losing weight. She currently weighs 113 kg (250 lb) and
is 152 cm (60 in) tall. Her blood pressure is 120/80 mm Hg. Visual field testing shows enlarged blind spots. The ren1ainder of the neurologic examination shows normal findings. A CT
scan of the head with and without contrast shows no abnormalities. Examination of the cerebrospinal fluid shows: •
Opening pressure 3.00 mm H 20 l
t I I. •
Glucose 70 ,ng/dl
Protein 25 ,ng/dl
WBC 1/rnn13 I il i. '
'
RBC 0/mm~ I ' .I ii 1
1
'l •

i:
!!
1·1,: !
••
• j
.. ' • I
, 'I'
". I •I .I I I I I
Gram stain and cultures are negative. 11
;:
! 1.
II
For each patient with papilledema, select the most likely diagnosis. ! !. ,
,I I It'
1
r,


. r
:' ' • I I
..
0 A) Bacterial meningitis
I
.'. \t 111
1
ll
I l '111
i;.
1ll1!1l1'(lt ./
,..
~=
O B) Cerebral infarction

·ilJlli!J!jl'
l
II!;I11•··11;. . I
'
11( ,. 111',1j,.. • '· :,
0
0
C) Cryptococcal meningitis
D) Glioblastoma multiforme
·;
I
,

·1
I

1I .
,

11111:1 ,,,.,,
• I
I ' / l ill
Iii,I,!II 11,
r. 1
·f\ • : •
t • •(
• ,,1

i
l'
j 1!I !,
,! 11 • '1 : I 11111 1 !tli;•:1•1j 1·.,1 ; ,. I • J
0 E) Herpes simplex encephalitis i1-1I II I· t ·:1: 11 1• -w11: •• • • • ,. • ·I
0 F) Hypertensive encephalopathy : lhl1: ' , ·1i111:!li , ). i: :',!:··1'; I· • ·•j: 11
l''"Ii 'i•l:1
ii q11 I ·1 .-
. r:1:!, !r :'tit1!1!, Ii· ' ;l • : • • 1 • ::iw
' 'l ·1 ' • ,,

0 G) Idiopathic intracranial hypertension I

0 H) lntracerebral hemorrhage ! , 1I1 111 !1'.•1!t1!I i!1II II1• ;l • r~ '


.,, .. l ·l 1 ' f
• • • •.·/,:
·•1 1

O I) St. Louis encephalitis . ·i . ii;,;;; •1{:II I ,: , 1 . ;ii,•. • ;ri


. . . .•
1

! :!'i·
1. 1!1 I '
1 .j:• 1iJ1
:It; 1;.] 11!!..• '! .' Hl• ·I '
l
.
I
32. A 25-year-old man with a history of intravenous drug use comes to the emergencydepartmentbecause of a progressivediffuseheadache,generalizedmalais~.and low-gradefeverfor • ·,1
2 months. During this period, he has had a poor appetite resulting in a 6.8-kg (15-lb)weight loss. His temperatureis 38°C (100.4°F).Examinationshowsneck stiffness.Mentalstatl:is
examination shows no abnormalities. Cranial nerve examination shows weakness of the lateral rectusmuscleon the right and bilateralpapilledema.A CT,scanof the headwith and ,. .
'.
without contrast shows moderate ventricular enlargement. Examination of cerebrospinal fluid shows: ,1, • 'I •
1
- _;( , .'i , .•. i.
I I
1
: ll ' '
Opening pressure 220 mm H20 1 , ,; .( , . i 1
Glucose 35 mg/dl ', l , /I l 'll H 1 :
. .: I

Protein 150 mg/dl:. I


·i
wsc h yt
Lymp oc e$
oooo~r;1m3
11
. ,o 1
I lI ii,_ ·1
'i
,. :, •
I

'

RBC • 3
~·/n1m 1
11
1
''
1
• ' I '
1 I ,'!1lh .
;

I Ii ''
I I •'I '
.; .. :..
.
. ,. . ;;: •
- • • i

I <
■ l\1ark Con1prehensiveClinical Science Self-Assessrnent 39 rnin 33 sec
' .
. ' ; • •' . -. ' .. ' • . . .
: '

33. A 52-year-oldCantonesewoman comes to the physician with her husbandand brotherfor a follow-upvisit 7 days after she underwentremovalof a 10-cm complex,solid, cysticadnexal
mass. The patient insists that the physician disclose the resultsonlyto her husbandand notto her or to otherfamilymembers.Which of the followingis the most appropriate courseof
action?

O A) Consultthe hospital ethics committee


O B) Contactthe hospital attorney
O C) Sharethe resultswith the husband only
O D) Share the resultswith the patient only
O E) Obtain a psychiatricevaluationof the patient

.
.
11

! l ' 'II
I jl

II I/
'
ll
I I I •
'
34. A 42-year-old woman comes to the physician because of a 3-month history of a recurrentvivid dream that several men are assaulting her and her children. Upon awakening,she is
anxious and distressed by the frightening images. She realizes that it is just a dream but is afraid to go back to sleep. She does not know why she is havingthis particulardream because
she has never been the victim of an assault. She drinks two to three cups of coffee each morning. She does not use illicit drugs. Physical examination shows no abnormalities.There is no
evidence of depressed mood or hallucinations. Laboratory studies are within normal limits. Which of the following is the most likely diagnosis?

O A) Acute stress disorder


O B) Nightmare disorder
O C) Panic disorder
0 D) Sleep apnea
O E) Sleep terror disorder

I
II

ll, '
I jI•
q '
.' ti.
ltie responseoptions for the next 2 items are the same. Select one answer for each item in the set.

For each patient with urinary incontinence, select the most likely cause.

O A) Detrusor instability
O B) Interstitial cystitis
0 C) OVerftowincontinence
0 D) Stress incontinence
O E) Urethra diverticulum
0 F) Urinary fistula

35. A previously healthy44-year-old woman, gravida 4, para 4, comes to the physician because of a 9-monthhistory of progressive loss of small amounts of urine while running;she now has
to wear an absorbent pad. Examination shows a second-degree cystourethrocele.

..
...

I I
:II f,;•I I
•l
I
I
I )ii' I
.I!
..

.,
"I.,,
I.ji
J
II

'I
I•
I• "II
:I I •

t ;Ii I

I ,!1 :,

I
IJ •
I I
•j I
' •
I I 'I\
• Ill a I
For each patient with urinary incontinence, select the most likely cause. ·1
I'I I . ..
I •,·

O A) Detrusor instability
\ I ' -i' \I \ti I 'I

O B) Interstitial cystitis
II
',1 :\ \ l
I I •

l ;Iii, 1:\1 I I\,


.'
' I' I
0 C) Overflow incontinence I

0 D) Stress incontinence 1
O E) Urethra diverticulum .h1 i\ 1
\ ,Ii°' I
0 F) Urinaryfistula
1i.JI I1\1 p• , !\:r ' .!l!\ ., ?

'I
111' II I
1
' ' ' '
36. One day after an uncomplicated spontaneous vaginal delivery, a 23-year-oldwoman, gravida 1, d,~ra1, ha~ij~he onset of loss of smali amounts~f urine'.Sh~ received epidural ~nesthesia
during labor and delivery. Examination shows an episiotomy without evidence of hematon1a.:She.is voidihg •§0to 75 ml of urine at a time. Postvoid resid.ualvolume is 300 ml.
1
Ii'
I .
11· 1 ,
l ' •
\,1: 1, lj i' l\
,IJ 11. :I
• I I

ll• ,l; '


.I
•I
lI I •
I 'I
II
II I' . •I
• I•
!
• •
,I
I. , 1I i


: llI l
1' !.. • ,I
I
.
I.
l
. .'

' I •
I
I,
37. A 62-year-old woman is admitted to the hospital because of a 3-day history of progressive shortness of breath and cough productive of yellow sputum. She has a 7-year history 0f c.hronic
obstructive pulmonary disease treated with albuterol and ipratropium inhalers. Increased use of the i'nhalershas not relieved her symptoms. On admission, h~r temperature·is· . .
37.2°C (99°F), pulse is 96/min, respirations are 28/min, and blood pressure is 158/80 mm Hg. Pulse oxim~tryon room air shows an oxygen saturation ot 90%. Breath sounds are •

decreased, and diffuse wheezes are heard bilaterally. Laboratory studies show: ' I \\: .
;l' • I

Leukocyte count 9500/mm3 \ •


Segmented neutrophils 72% ll
, ·;.;·1\11·Ii•,l•,,1 •;\1 l1 1

Eosinophils 1% i'1·
· •• I
Il ' ' .,,·111•11~·' 1•,• ' ' I • - • f ', I ' •
I
Lymphocytes 22% 'I . I
I •

I . ,·•It! I
' I

Monocytes 5%
'
,, . ' •' '
'
I •, I
I•.
..
• '
,

' I' .I J' I,11 !.~;.-•,·


' i.' I : 'i I 1: • I I I

' .
.··1.'
I ,, •,
'; ~ • ' ': : l

An x-ray of the chest shows no abnormalities. Treatment with nebulized ipratropium and albuterolan·9intravenousmethylprednisoloneis begun. Two days later, she•f~el$well. Her pulse
is 86/min, respirations are 18/min, and blood pressure is 135/80 mm Hg. Air movement is good, and occasional wheezes are heard. Laboratory studies tod;:iysho~ ;,.•: . •
,. !i
Leukocyte count • 1' 18;500/mm3 '.
'I
I'
Segmented neutrophils ··180%
Bands 5% I

Lymphocytes 10% .I

Monocytes
I
'
'5%
'
iI
1


Which of the following is the most likely mechanism of this patient's increased leukocyteaount? !.I 1\
I
' I

O A) Differentiation of plasma cells to segmented neutrophils •I1


' •

O 8) Endotoxin-mediated release of interleukin-1 and tumor necrosis factor


O C) Glucocorticoid-induced demargination and storage pool release of segmented neutrophils '
'\\
O D) Increased epinephrine from stress of occult infectious process I
I ,
I • I . .
1\11
nl
.' '
O E) Overproductionof granulocyte-macrophagecolony-stimulatingfactor I
I
:• l • ' •'
I 1 • •
I

" I
' ' ! '

!!!l!l!!~~~
I•
'

I l I 1·;
, I
I '

,, ' '
. .

I
'
I .' ' ' . ..
•,.
Ii • .
i '. '' 1 •• •.,.

I I "

I
'
' I

j "
I I
I
..
. . '
39. A 5-month-old boy is brought for a follow-up examination. He was born at 37 weeks' gestation and has had persistentwheezingsince shortlyaft~rbirthdespite:~reatment withnebulized
and oral bronchodilators and oral corticosteroids. His diet consists of 32 ounces of iron-fortifiedcow's milk-basedformuladaily. He appearswel!.nourishedand·happy..Onexamina1ion;
there is moderate relief of wheezing with extension of the neck. Which of the following is the most likely mechanismof this infant'swheezing? ; ,1 , • ; '
I ''
I,
O A) Allergic reaction to cow's milk
O B) Aspiration of a foreign body
O C) Compression of the airway by a vascular ring
O D) Concurrent upper respiratory tract infection
O E) Persistent immaturity of lungs
....
• '·I

I
•J
40. A 14-year-old boy is brought to the physician by his parents because of a 2-year history of increasing academic problems. His parents say that he has always been hypera~ive and
distractible, but now his acaden,ic perfom,ance has deteriorated to the point that he is failing ninth grade. His teachers say that his hyperactivity is disrupting the classroom. He weighs-
54-kg (120 lb) and is 152 crn (60 in) tall. Sexual development is Tanner stage 5; exan1ination shows n,acro-orchidism, which was not shown on previous examinations. He has a high
forehead and long, protruding ears. He exhibits poor eye contact during the exan1ination. Psychoeducational testing shows an IQ of 70. Which of the following is the most likely
diagnosis?

O A) Attention-deficiUhyperactivity disorder O F) Lesch-Nyhan syndrome


O B) Autistic disorder O G) Pervasive developmental disorder, not otherwise specified
O C) Down syndrome O H) Prader-Willi syndron1e
O 0) Fetal alcohol syndron,e O I) Rett's disorder
O E) Fragile X syndrome O J) Seminiferous tubule dysgenesis (Klinefelter's syndron,e)



,
41. A 57-year-old woman with breast cancer comes to the physician because of increasing neck pain over the past 3 days. She has fallen frequentlybecayse of musclewe~k~s. VitaJ
signs are within normal limits. Examination shows hyperreflexiaof all extremities.There is tenderness over the cervicalspine, Serum calcium concentratiopis 1'1mg/dL,.,X-raysshow
metastases to the cervical spine. Which of the following is the most appropriate next step in management? '
;!._
.:.;
i
O A) Application of a soft cervical collar ,\ t .' •
'•
.---

1.. " ' :,

O B) Physical therapy
.. •
O C) Mithramycin therapy
.""
O D) Tamoxifen therapy
~

O E) Spinal cord decompression and cervical stabilization . '


..

I
-
I

42. A 37-year-old man is brought to the emergency department 6 hours after the onset of constant,increasinglysever~ abdominal pain and nausea. His symptoms awoke him from sleep,
and he has vomited once since that time. He has no history of similar syn1ptoms,and he does not take any medications or use alcohol or illicit drugs. Family history is noncontributory.He
is in acute distress and lying in the fetal position. Any movement exacerbates the pain. His temperature is 37.8°C (100°F), blood pressure is 108/68 mm.Hg,.pulse is 112/min, and .
respirations are 24/min. The lungs are clear to percussion and auscultation.Examinationshows a rigid abdomen; bowel sounds are absent. Laboratory studies sho_w: •
' ';,
Hemoglobin 14 g/dL
Leukocyte count 18,200/mm3
Platelet count 150,000/mm3
Serum
Urea nitrogen 34 mg/dl
Creatinine 1.9 mg/dl
Total bilirubin 1.2 mg/dl

An x-ray of the chest shows a small amount of free air under the left diaphragm. Administration of antibiotics and fluids is begun. Which of the following is the most appropriate next step in
management?

O A) Barium swallow
0 8) CT scan of the abdomen
O C) Intravenousadministration of an HTreceptor blocking agent
O D) Upper endoscopy
0 E) Laparotomy



43. A 67-year-old woman has had anorexia, fatigue, and weakness for 4 months and low back pain for 3 monthsthat has been unresponsiveto therapy with nonsteroidal anti-inflammatory
drugs. She has been treated for hypertensionfor 5 years but does not ren1emberthe name of her medication._Thepatient has been taking calcium supplements for 5 years because her·
mother had severe osteoporosis. She appears pale. There is tenderness to palpation over the lumbar spine. An x-ray of the lumbar spine shows /yt_icchanges: A bone scan shows
increased uptake in the right hip consistent with osteoporosis. Her hemoglobin concentrationis 8 g/dl, and serum calcium concentration is 13.mg/dl. A blood sm,earshows roufeaux!
1 1
formation. Which of the following is the most likely diagnosis? :1 • 1_

''
I
!
q +,
II . I. I. : I
O A) Familial hypocalciuric hypercalcemia •I'
0 B) Metastatic malignancy ,\\ I I I'' I I .1
'
I
.:I
!
O C) Multiple myeloma l ' 1
' '
' .. • •

"I
O D) Osteitis deformans (Paget disease) I • 'I i 1 \
H
•I'l •.~. I",t l: '
0 E) Primary hyperparathyroidism \I I'\' 1 l
0 F) Squamous cell carcinoma
I.
I I I'I
1\1i~'.
;.i .jl\ \ \.
I• ' • •!

0 G) Vitamin A toxicity 1,'


0 H) Vitamin D toxicity .. '
•I I
., 1···1,
I 'I' •
l , I
\ , i I
I ·.l :I t 1
' ': \jil,I
I\

!\, 111
l '' I\i\.l I
\
'
:\
1 I
I I
,..
I,
l I,
I
:l \
II ',
I I '
I •;
.
l ' \l
• . l '\ I
l
! ,l
!• ·'11
l•
•: j

I I' , .
• I L '
. . '
44. An 87-year-old nursing home resident with dementia, Alzheimer's type, is admitted to the hospital because of progressive lethargyand decreased appetite for 3 days. She had a flu-like
illness followed by a deep cough 1 we~k ago Over the past 1O months,she has been hospitalized once for bacterial pneumonia. She is responsive only to painful stimuli. Her
temperature is 38.6°C (101.5°F), blood pressure is 110/60 mm Hg, pulse is 123/min and regular, and respirations are 28/min. Examination shows dry mucous membranes. There is no
adenopathy.Crackles are heard in the right lung base. An x-ray of the chest shows an infiltrate at the right lung base. The remainder of the examination shows no abnormalities. Which of
the following is the most likely predisposing factor for this patient's pneumonia? ,

0 A) Decreased airway elasticity 0 F) Impaired cardiac response to exercise


O B) Decreased baroreflex 0 G) Impaired T-lymphocytefunction
0 C) Decreased gag reflex 0 H) Impaired thirst
I
I I
I
0 D) Decreased thyroid function O I) Increased lung compliance I
I

0 E) Diastolic cardiac dysfunction 0 J) Renal salt wasting


I

\ \
\
\

I .I
I

'\ ' .I
\1' I \\ • I
I•
lI
I•
' I
.
; '( \
'.
I
11' I \\! •\ ) I '
l I
,\ lI \\ I
I I\
I I I I
11 I
I I
\ I • 1\' I\ I I l ' I
I
I I
I I II II II

1
t. ' ••I
' '
I
1
111 ·
1
" • I , . , ! : 1
• , '. ·
45. A 42-year-old man comes to the physician because of progressive swelling of the legs over the past 2 months. He has a history of stage IIA Hodgkin's disease treated:1 year1agowith . ,1
radiation therapy to the neck and chest. His temperature is 37°C (98.6°F), blood pressure is 102/80 mm '!-ig, pulse is 110/min, and respirations are 22/min. Exa.mination.showsjugular 1 •'

venous distention that increases with inspiration. The lungs are clear to auscultation.Cardiac examination shows a nondisplaced point of maximal /mpulse; hea~ sounds are,<:iistant.An
early diastolic sound is heard at the apex. Abdominal examination shows mild distention with shifting dullness.The liver is pulsatile, and its edge is palpated 4 cm beloy.ithe iight costa.1 1
.,
margin. There is 2+ peripheral edema extending up to ttle knees. Which of the following is the most likely mechanism of this patient's increased central venous press'ure? •• -1 r: lli. ,· 1
.l , I' . . •I • I ·.I ''ll
t• ,
:Ii..11 L ' l,•
ti, 1
l i ' I ' t ' i·

- O A) Constrictive pericarditis ' ii , : '..1111 11 '1,'.i, •


ii ;, i',: ·\ 1 \, \lj\i•\'l;
1
11
'1 '
·'..
;I
.,
l!I il > '
I' ''•I'·
11'111
1

l .I • ·11'1,
11 t I
1
'H I l •\ '
O C) Left-sided congestive heart failure 1 ·j :1 • .• 1 , ,, 1 • 1
,
l • l I. 1· •• ·
O 0) Mitral stenosis • I

O E) Tricuspid stenosis ,, I.;


'1·
·1\1\ I i-1.ii ll··
• l. -
i;l;fi'i :!\!,. 1 . .
' • . t
;!1 1
'q i 1 •l I '! ,
: • I I i fl 'I l ' I ;: ;

I \;.1 • \11'1·'·l ·i.i': l. ;·,l '· 1


ii
!Iil II • 1
I,,! 11-;.:. •' 1·
1 1 ••• •

J ! ' ~ ' . .
' . •. i
I I ·1 I' :.
I I ,': 1: Ii(
1111',1!1
I' . 1I. . . ..
I •' 4,,
'.ij1,1 ~ _; .,
t

' I Iii1': l . •; I : '.


·1 i
I

I'll .! l •
I ! 11:
I I 1,

I
I ''

!!,11!i-,1!1
I ;:

.·l ! l
''
, , l
II I !
' I .I 1 ..
• ! ' '
i ': I. • l I
I .
! • i !. I 1!. I''.,' I. '' .
11,1,l, l
I • I I ll• .
I
' \ H,
••
l t •
• !, I i •
46. A 20-year-old man is brought to the emergency department on a summer day 20 minutes after developing headache, nausea, and unsteady gait while rl)nningth~ last 2 miles of-~
mara~hon_.On arrival, he is confu_sedand _disoriented.His temperature is 40°C (104°F), blood pressure is 100/60 mm Hg, and pulse is 155/min. ~is skin is Y(~~- and dry. Ne_urologic, : :. .1
examination shows no focal findings. Which of the following is the most likely mechanism of this patient's condition? •• • . ,
' '
I
0 A) Depletion of total body potassium '
' '
II
I'11I ,· '
I''
•I
\, ' ,1 • .,

0 B) Depletion of total body sodium ., . l\


,'
I II
'I
ll .I
I ' .' ' •I
1·' n '
O C) High-outputcardiac failure I
. . I il
• I
'I
I •
0 D) Inadequate dissipation of body heat ,l
l
'
!•
' i\ •:,,.
O E) Release of creatine kinase from muscle cells I I I
I
1
'' l I. \ I I
I ..
'
' 11
11
1! l I
'.
'I
\ '
II
i ',
,I I
•'
j\I 'I ' ii
l' l.'
•·,!
.
'.
..
:_f

'
.
l . ,,
I,
,,
,I
~H
I
'
I I 1I I , r •'
i' ,.
l '; •\ '
.
r ' \l'
•• I\,,
.. ' I . •
I ' 'I \ I
I
' • -I
I
I '
' '
l·\l I 'I
'
,I l ';
l 11 ' ! .I

'
,' I ,I ' I ,' I, i\' .......
..
. .!,
I
• ,I 11
' ' I
I
I
.. . . ,,' \t

1· I'
'
'
' . I l •
,I '' \ I

I
I I !
''
"•
\
'

I '
I o
1. A 42-year-old woman comes to the physician for evaluation of persistently increased blood pressures. At her last two office visits during the past 3 months, her blood pressure has ranged
between 150-170/105-115 mm Hg. During this period, she has had occasional headaches. In addition, she has had an increased urine output over the past 6 weeks that she attributes to
a diet high in sod1um. She is otherwise healthy and takes no medications. Her blood pressure today is 168/115 mm Hg, pulse is 68/min, and respirations are 14/min. Funduscopic
examination shows mild arteriovenous nicking. The point of maximal impulse is not displaced. There is no edema, abdominal bruits, or masses. Serum studies show:
Na• 144 mEq/L
CI- 90 mEq/L
K• 2.9 mEq/L
HC0 3- 32 mEq/L
Urea nitrogen 20 mg/dl
Creatinine 1.2 mg/dl

Which of the following is the most likely underlying cause of this patient's hypertension?

O A) Autonomous production of aldosterone


O B) Catecholamine-producing tumor
O C) Decreased arterial distensibility caused by atherosclerosis
O D) Excess production of atrial natriuretic peptide
O E) Juxtaglomerular cell hypertrophy and sclerosis

hov. htddeh'icdns
2. A 32-year-old woman who is HIV positive has a CD4+ T-lymphocyte count of 800/mm3 (Normal>500). Her health maintenance regimen should include immunization against which of the
following pathogens?

0 A) Haemophilus influenzae type b


O 8) Hepatitis A
O C) Influenzavirus
0 0) Neisseria meningitidis
0 E) Streptococcus pyogenes (group A)

I
I I

I I
3. A 2-year-old boy is brought to the physician because of fever and cough for 2 days. He had Streptococcus pneumoniae meningitis at the age of 1 year, S. pneumoniae bacteremia at the
age of 18 months, and pneumonia at the age of 22 months. Two n1aternaluncles died before the age of 2 years from "infection." His temperature is 39.8°C (103.6°F), pulse is 150/min,
and respirations are 60/min. Examination shows subcostal retractions on inspiration. Laboratory studies show:
Hen1oglobin 1O g/dL
Leukocyte count 36,000/mm3 I
'
Segmented neutrophils 70%
Bands 20% I
Lymphocytes 8%
Monocytes 2%
Platelet count 240,000/mm3
Serum
lgA <5 n1g/dL l '
lgG 30 mg/dL
lgM <5 mg/dL I
l
An x-ray of the chest shows an infiltrate in the left upper lobe. Which of the following is the most appropriate next step in management?

0 A) Reexamination in 12 weeks I
0 B) Prednisone therapy only II I
I I III
0 C) Prednisone, vincristine, doxorubicin, asparaginase, and n1ethotrexatetllerapy I,
'
0 D) Zidovudine (AZT), lamivudine (3TC), and ritonavir tllerapy I'
0 E) Intravenousimmunoglobulin infusion
0 F) Bone marrow transplantation 11
t ii
I; Il .
0 G) Thymustransplantation l i\l
' I

I'I I

- ,1h1
l'""
I'
~~
.ri!l P' ,..
'I r 1 !'•1tous Ne,.t Lab Values Review Help Pause
4. A 19-year-old man is brought to the emergency department 45 minutes after sustaining a single, large stab wound to the right upper quadrant of the abdomen. He is obtunded. His blood
pressure is 60/palpable mm Hg, and pulse is 148/min. Breath sounds are equal bilaterally. Examination shows a 4-cm laceration in the right upper quadrant in the midclavicular line. The
abdomen is distended. Which of the following is the most appropriate next step in management?

0 A) X-rays of the abdomen and pelvis


O B) Angiography
0 C) Contrast study of the wound tract
0 D) CT scan of the abdomen
0 E) CT scan of the chest
0 F) Laparoscopy
0 G) Peritoneal lavage
0 H) Laparotomy
0 I) Local wound exploration

l
II
II I
II •I
'
l
I
II

I
\I II
l I I ,I
I
I
t I

I 1\ l \ I ,
1
I I
I
I
rL1tiu1Lll do~n-l ,_:i i,lt_.,;;.,-,·11L Xdll1i11•··:-.
■ Mark Comprehensive Clinical Science Self-Assessment 1 hr 1 min 11 se

5. A 17-year-old boy is brought to the emergency department by his parents because of bizarre behavior for 6 hours. Last night he was out with friends, and since returning, he has been
confused and has "trashed" his room. His blood pressure is 165/95 mm Hg. He is hypervigilant, has little spontaneous speech, and is disoriented to place and time. He appears catatonic
but abruptly becomes assaultive two times and needs to be restrained. Which of the following is the most likely substance taken?

0 A) Cocaine
0 B) Ecstasy
0 C) LSD
0 D) Methaqualone
0 E) PCP

\
I

\ \
6. A previously healthy 67-year-old man is admitted to the hospital because of lethargy, confusion, muscle cramps, and decreased appetite for 7 days. He appears ill. His temperature is
37°C (98.6°F), blood pressure is 120/70 mn1Hg, pulse is 98/min, and respirations are 20/min. Breath sounds are diminished at the right lung base. Neurologic examination shows no
abnormalities except for lethargy. Serum studies show:
Na+ 114 mEq/L
K+ 4.3 mEq/L
Creati nine 1 mg/dl
Thyroid-stimulatinghormone 4.1 µU/ml

An x-ray of the chest shows a 2-cm nodule in the right lower lobe and rnediastinal adenopathy.A biopsy specimen of the nodule is most likely to show which of the following?

O A) Adenocarcinoma
O 8) Clear cell carcinoma
0 C) Mesothelioma
O D) Small cell carcinoma
O E) Squamous cell carcinoma
7. A previously healthy 62-year-old man comes to the emergency department because of abdominal pain for 48 hours. His temperature is 38.6°C (101.5°F), blood pressure is
130/80 mm Hg, pulse is 110/min, and respirations are 15/min. Abdominal examination shows diffuse left lower quadrant tenderness with no peritoneal signs. Rectal examination shows no
abnormalities; test of the stool for occult blood is negative. His leukocyte count is 14,700/mmJ. Which of the following is the most appropriate next step in diagnosis?

O A) Barium enema
O B) CT scan of the abdomen
0 C) Colonoscopy
0 D) Cystoscopy
O E) Exploratory laparotomy
8. A 37-year-old woman, gravida 2, para 1, at 12 weeks' gestation comes to the physician for her first prenatal visit. She has hypertension treated with a j3-adrenergic blocking agent. Her first
pregnancy was complicated by premature rupture of membranes at 26 weeks' gestation, followed rapidly by a preterm vaginal delivery. She is a carrier of the sickle cell trait. She has a
history of an appendectomy at the age of 14 years. She does not smoke cigarettes, drink alcohol, or use illicit drugs. Her blood pressure is 130/82 mm Hg; other vital signs are within
normal limits. Examination shows a uterus consistent in size with a 12-week gestation. Which of the following is the greatest risk factor for preterm delivery in this patient?

O A) History of an abdominal operation


0 B) Maternal age
O C) Maternal hypertension
O D) Previous preterm delivery
O E) Sickle cell status
9. A newborn is in severe respiratory distress immediately following delivery. She was born at 35 weeks' gestation to a 35-year-old woman, gravida 2, para 1, aborta 1, who did not receive
prenatal care. The newborn's pulse is 60/min, and respirations are irregular and labored. Examination shows pallor with perioral cyanosis, anasarca, hepatosplenomegaly, and scattered
petechiae. Cord blood hemoglobin is 4 g/dl, and reticulocyte count is 18%. A direct antiglobulin (Coombs') test is positive. Which of the following sets of blood groups is most likely in the
mother and her newborn?

Mother Newborn
0 A) A, Rh-positive 0, Rh-positive
0 B) A, Rh-positive 0, Rh-negative
0 C) A, Rh-negative 0, Rh-negative
0 0) 0, Rh-positive 0, Rh-negative
0 E) 0, Rh-negative 0, Rh-positive
10. A 32-year-old man receiving intensive chemotherapy for Hodgkin's disease has a temperature of 39°c (102.2°F). His respirations are 40/min. Widespread crackles are heard in all lung
fields. An x-ray of the chest shows a diffuse alveolar and interstitial pattern. Which of the following is the most likely causal org~nism? •

0 A) Aspergillus species
0 B) Candida albicans
0 C) Coccidioides immitis
0 D) Pneumocystis jiroveci (formerly P. carini1)
0 E) Streptococcus pneumoniae


11 A 67-year-Oldrrian comes to the physician because of a 6-month history of double VJS1on. slurred speech, and difficulty swallov1ingliquids Initially,his symptoms v1ereintermittent but nCY11
occur daily and are v-,orseat tht end of the day. Over the past v,eek, he has had shortness of breath with exertion He has p1?rn1c1ous anemia treated ,,,ith monthly
vrtafTllnB-JcyanocobalaminJ ,n,ections and auto1mrr,unethyroid d1seasE:currentlytrr::atedv,L~thyroid replac1;:ment therapy E1.am1nallon shows bilateral ptos1sand disconjugate gaze.
There is bilateral facial v,eal(ness and hypernasal speech The tonguE:is v,eak, and the gag refle1.1sreduced IAusclestrength rs 4/5 ,n the proximal muscles of the upper and lov,er
extremttJes.Deep tendon reflE:xesare 2+ diffusely Bab1nsk1'ssign 1sabsent Sensory 1::1am,nat1on shov,s no abnormaht1esA CT scan of the chest is shc,,11n. Which of the follov1ingis the
mr.Jsthketyd1agnos1sof thrs patJent's,ntrathorac,clesion?

r..,/AJ Asp1rationpneumonia -., DJ Sarco1dos,s


":, B) Bronchogen1ccaranorrra .,...,,
EJ Thymoma
--=.,CJ Lung metastasis ./ FJ Thyroid carcrnoma
12. A 67-year-old man has had shortness of breath on exertion for 3 months; he has had an 11.3-kg (25-lb) weight loss during this period. He has smoked two packs of cigarettes daily for
25 years. He appears chronically ill. Examination shows decreased breath sounds on the left; heart sounds are normal. An x-ray of the chest shows a large left-sided pleural effusion.
Which of the following is the most appropriate next step in diagnosis?

O A) Bronchoscopy
0 B) Thoracoscopy
0 C) Closed pleural biopsy
0 D) Open pleural biopsy
0 E) Thoracentesis
13. The genetic disease institute at a university hospital has developed a rapid screening test for a serious but treatable inherited metabolic disorder. Although this disorder is predominantly
found in a particular ethnic group, it is also found sporadically throughout the entire population. The screening test has a sensitivity of 95% and a specificity of 90%. When used in an
ethnically prescreened population where the prevalence of this disorder is 30%, the positive predictive value is 80% and the negative predictive value is 96%. The institute proposes to
use this screening test on the general population where the prevalence of this disease is 0.1 %. Which of the following is the most likely result of this screening program?

O A) Negative predictive value decreases


O B) Positive predictive value decreases
O C) Sensitivity decreases
O D) Specificity decreases
14. A 16-year-old girl is brought to the physician because of episodes of palpitations over the past 6 months. The episodes occur when she runs or plays basketball. She is otherwise
asymptomatic. Her blood pressure is 124/46 mm Hg, pulse is 78/min, and respirations are 18/min. She weighs 55 kg (121 lb) and is 180 cm (71 in) tall. Her arm span is 188 cm (74 in),
and the upper segment to lower segment ratio is 0.85. Her fingers appear long and are hyperextensible. A grade 4/6, early diastolic murmur is heard along the upper and middle left
sternal border with radiation to the apex. Peripheral pulses are bounding. Which of the following is the most likely cause of these findings?

O A) Aortic incompetence
O B) Aortic stenosis
0 C) Mitra! incompetence
O D) Mitra! stenosis
O E) Pulmonary incompetence
O F) Pulmonary stenosis
0 G) Tricuspid incompetence
O H) Tricuspid stenosis
15_ A previously healthy 16-year-old high school wrestler comes to the physician because of a rash on his forearms and the back of his legs for 1 week. He is allergic to pollen and dust
Examination shows patches of erythema with mild lichenification over the antecubital and popliteal fossae. There are clusters of painful umbilicated vesicles at sites of active skin
inflammation. Which of the following is the most likely diagnosis?

0 A) Eczema herpeticum
0 B) Herpes zoster
O C) Keratosis pilaris
0 D) Lichen planus
0 E) Pityriasis rosea


16. Three days after being hospitalized for treatment of a hip fracture sustained in a fall, a 62-year-old woman becomes acutely short of breath and coughs up a small amount of blood-tinged
sputum.She appears anxious. Her blood pressure is 110/70 mm Hg, pulse is 11O/n1in,and respirations are 24/min. Examinationshows no other abnormalities. Arterial blood gas
analysis on 40% oxygen by face n,ask shows:
pH 7.40
PCOz 38 mn, Hg
Po2 70 mm Hg

Ventilation-perfusion lung scans show multiple segmental areas of mismatch on the right. Which of the following is the most appropriate next step in management?
0 A) Pulmonaryangiography
0 B) Dopamine therapy
0 C) Heparin therapy
0 D) Urokinase therapy
0 E) Intubation
17. A 28-year-old man is brought to the emergency department by police because of severe pain in the right lower quadrantof the abdomen for 36 hours. He has been incarcerated in the
countyjail, and his court hearing is scheduled in 12 hours. He takes no n1edications.Abdominal examinationshows no rebound.Complete blood count, liver tests, and erythrocyte
sedimentation rate are within normal limits. An x-ray of the abdomen shows a normalgas pattern.Test of the stool for occult blood is negative. Which of the following is the most likely
diagnosis?

0 A) Adjustment disorder
0 B) Conversion disorder
0 C) Depressive disorder not otherwise specified
0 D) Factitious disorder
0 E) Hypochondriasis
0 F) Malingering
0 G) Somatization disorder

•\

' '
I,
' 1
I
l! l
I

' I
'•'
'•(
I \
\ I
18. A ~2-year-old man comes to the physician for a routine health maintenance examination. He has smoked one pack of cigarettes daily for 20 years and does not drink alcohol. He has a
sedentary lifestyle and requests advice about beginning a regular exercise program. His father died of a myocardial infarction at the age of 62 years. His 77-year-old mother and 51-year-
old sister are healthy. He is 178 cm (5 ft 10 in) tall and weighs 80 kg (176 lb); BMI is 25 kg/m2.His pulse is 80/min, respirations are 12/min,and blood pressure is 140/90 mm Hg.
Examination shows no other abnormalities. In addition to measurement of serum cholesterol concentration,which of the following is the most appropriate next step prior to starting this
patient on an exercise program?

O A) Complete blood count


O B) Measurement of serum homocysteine concentration
O C) Exercise stress test
0 D) Spi rometry
O E) Cardiac catheterization
19. A 6-month-oldgirl is brought to the physician for a routine health maintenanceexamination.She was born with a lumbosacralmyelomeningocelewhich was successfullyrepaired at 2
days of age. The anterior fontanelle is 6 x 8 cm and bulging, and the posterior fontanelleis 3 x 4 cm and bulging.She has severe motor and sensorydeficits involvingboth lower
extremities. A head growth chart shows the following values:
Age Head circumference(cm)
Birth 34.2
1 month 36.6
2 months 38
4 months 44
6 months 47

Which of the following is the most likely cause of increased intracranialpressure?

O A) Acute cerebral edema


O B) Decreased absorption of cerebrospinal fluid
O C) Dilation of cerebral arteries
O
O
D) lntracranialmass lesion
E) Obstructionof lateral sinus l
O F) Obstructionof superior vena cava
O G) Obstructionof cerebrospinal fluid flow
O H) Overproductionof cerebrospinal fluid
20. A 47-year-old man comes to the physician because of fever and chills for 1 day. He has a 20-year history of alcoholism and cirrhosis of the liver. One month ago, he underwent evacuation
of a subdural hematoma. His postoperative course was complicated by generalized tonic-clonic seizures; he has been receiving carbamazepine therapy since then. He continues to drink
12 beers daily. His temperature is 38.9°C (102°F), and blood pressure is 120/80 n1n1Hg. Examination shows scleral icterus, ascites, and splenomegaly. Laboratory studies show:
Hematocrit 39%
Mean corpuscular volun1e 102 µm3
Leukocyte count 1200/mm3
Segmented neutrophils 6%
Eosinophils 1%
Basophils 1%
Lymphocytes 80%
Monocytes 12%
Platelet count 21O,OOO/n1mJ

Which of the following is the most likely cause of this patient's decreased leukocyte count?

0 A) Carbamazepine therapy
0 B) Folic acid deficiency
0 C) Hypersplenism
0 D) Sepsis
0 E) Suppression of bone marrow by alcohol
21. A 32-year-old woman comes to the physician because of vaginal discharge for 2 weeks. She has been sexuallyactive with one female partner for 5 years. Her last Pap smear was
6 years ago when she was sexuallyactive with a male partner. She has not used illicit drugs or alcohol. Examinationshows a grayish vaginal discharge with a pH greater than 4.5. A wet
mount preparation of the vaginal discharge is most likely to show which of the following?

0 A) Budding yeast
0 B) Clue cells
0 C) Ferning
0 D) Leukocytes in sheets
0 E) Trichomonas vagina/is

)
I•
I

I II
II
I

\
I I
l
11 l

I
l
I
I
\ . I I
,l •I
I
I
' \ Ii!l I l ! I


1 I!
I
1 l I
\l I II I
II

: \
\ I

.\ \

\I I I I l
\I
1
l •
II \ 11 II l I I
j I1J l
'
I Ip 1
I,
I 1 i:
1 1
1 •
I I \ •
I •
' l II \ I

I
• I I
l ' 1. I
l :I
I

.I
I I I I II'
•'
I

I .
,.
~ p r. .,
Previous Next i• Lab Values Re111e\v
·--·- -· ... -· -....-.. .
--·-·-······-··
Hel1> Pause
. I II
'I \
The response options for the next 2 items are the same. Select one answer for each item in the set.1 \
,I
• ;
I
,\ II I \ \I II 11
l \ ' I
For each patient with weakness, select the most appropriate test to establish the cause of the condition.
I \ \ l 1
I I
• I
\
'
1
I

\
\\ I\\ I\ \
0 A) Brain stem auditory evoked potentials 0 G) Muscle biopsy 'I
I '
I
0 B) Carotid ultrasonography 0 H) Repetitive nerve stimulation ''
I
0 C) Chromosomal analysis for trinucleotide repeat 0 I) Somatosensoryevoked potentials .\
0 D) EEG
'
\ I
0 J) SPECT scan
' .
\

0 E) Electromyographyand nerve conduction studies 0 K) Temporal artery biopsy I ' '

I I
0 F) MRI of the spine 0 L) Visual evoked potentials I
l I

22. A 57-year-old man is brought to the emergency department 6 hours after the onset of weakness of his right face, arm, and leg. Three days ago, he had an episode of sudden visual loss
in his left eye that he describes as "a shade coming down." The episode resolved completelywithin 10 minutes.He has hypertensionand type 2 diabetes mellitus, both poorly controlled
with lisinopril and glybunde. Examination shows expressive aphasia and right lower facial droop. There is moderate weakness on the right, worse in the upper extren1itythan the lower
extremity. Deep tendon reflexes are 3+ in the right extremities and 2+ in the left extremities. Babinski's sign is present on the right. Sensory examination shows'no abnormalities.

I I
'
For each patient with weakness, select the most appropriate test to establish the cause of the condition.

0 A) Brain stem auditory evoked potentials 0 G) Muscle biopsy


0 B) Carotid ultrasonography 0 H) Repetitive nerve stimulation
0 C) Chromosomal analysis for trinucleotide repeat 0 I) Somatosensoryevoked potentials
0 D) EEG 0 J) SPECT scan
0 E) Electromyographyand nerve conduction studies 0 K) Temporal artery biopsy
0 F) MRI of the spine 0 L) Visual evoked potentials

23. A previously healthy 27-year-old woman is brought to the emergency department because of a 2-day history of weakness of her arms and legs and numbnessof her hands and feet and a
4-hour history of mild shortness of breath while supine. The weakness began in her feet and has progressed to involvethe hands. She describes a sensation of "electrical shocks"
extendingfrom the buttocks to the feet. Today, she tripped and fell several times and was unableto button clothesor hold utensils.Three weeks ago, she had a mild upper gastrointestinal
illness that resolved within 5 days. Her respirations are 20/min and shallow.There is mild facial weakness; cranial nerves are otherwise intact. Muscle strength in the upper and lower
extremities is 4/5 proximally and 2/5 distally. There is areflexia. Babinski's sign is absent bilaterally.Sensationto vibration is slightlydecreased at the fingers and toes.
24. A 72-year-oldman with hypertensionhas had increasinglysevere back pain overthe past 2 months.He had a myocardialinfarction4 years ago. He has marked tenderness over
T11, T12, L1, and L2. An x-rayof the lumbosacralspine shows osteoblasticlesions in these vertebrae.Which of the followingis the most likely diagnosis?

0 A) Abdominalaneurysm
O B) Fibrosarcoma
0 C) Metastaticprostate carcinoma
O D) Multiplemyeloma
O E) Osteosarcoma
I

25. A 47-year-old man is admitted to the hospital after threateningto harm a radio announcerhe believed was broadcastinghis thoughts.Overthe past 20 years, he has had multiple
psychiatric hospitalizationsfor threatening people who he believed were plotting against him, trying to control his mind, or causing him to hear voices by implantingdevices in his head.
Past symptoms improved with neuroleptic therapy; after discharge, he discontinuedthe medication and his symptomsworsened.Which of the following is the most appropriate
1
phannacotherapyto decrease this patient's risk for future hospitalization? \ 11 \ 1 \ : 1

O A) Clozapine 11 11\ I\
O B) Fluphenazinehydrochloride I I11

O C) Haloperidol decanoate
0 D) Risperidone
O E) Trifluoperazinehydrochloride
26. A 38-year-oldwoman, gravida 2, para 1, at 38 weeks' gestation has had no fetal movementfor 36 hours. Her prenatal course, prenatal tests, and fetal growth have been normal. Fetal
hearttones are heard by Doppler. Which of the following is the most appropriate next step in management?

O A) Routineprenatalvisit in 1 week
0 B) Maternalhydration
O C) Nonstresstest
I
O D) Immediateinduction of labor
0 E) Amniocentesis

ll
I\
I \
I

1
I t
II
I I l I
I I
11 II \
l' l l'' I
t

i1\\ \ \\1
I l ii I
I I I I I
I

I
l l
I. \f l ll I II I \'\
I
\
l 1
I \ \I I
ii I II I
' l
I '\ I \ I l I
!I l
j
I

I
i I
I 11 I I •Il !
I
I
ll \
!
I\ I \
\• l
'
.\
I
I
'
1
l
t
l I
d 1!1i • \ • I \ I I ' t I

1 I I
l i I I
l
'
• l I • I '
( .
27. A 3-year-oldboy is brought for a follow-up examination. He just completed a 10-day course of amoxicillin that has not resolved his right ear pain. He appears irritable. His temperature is·
38.9°C (102°F). Examination shows downward and lateral displacement of the right auricle with tendernessto palpation of the posterior auriculararea; his neck is supple. Which of the
following is the most appropriate next step in diagnosis?

O A) Bone scan
0 B) CT scan of the head
0 C) Tympanometry
0 D) Lumbar puncture
0 E) Tympanocentesis
r1,,::,11.1i
0 r'<id[C, ()f ,,Jc.,lj, ,I: :-:,•111i:•,·1,

■ Mark Comprehensive Clinical Science Self-Assessment 47 min 36 sec

28. An 18-year-old man comes for an examination prior to participation in school sports. He states that he has had a dull ache in the scrotum since being hit in that area during a basketball
game 2 months ago. Examination shows a 2-cm, hard, nontender mass in the right testicle. The mass does not trans1lluminateor change in size when the patient is placed in the supine
position. Which of the following is the most likely cause?

O A) Cystic dilations of the efferent ductules


0 B) Dilated pampiniform venous plexus
O C) Fluid accumulationwithin the tunica vaginalis testis
O D) Germinal cell tumor
0 E) Vascular trauma
29. A 32-year-old man with alcoholism is brought to the emergency department by friends because he has been unable to stand without support and has had "funny eye movements"; they
report that he has been drinking approximately 18 beers daily over the past month and has been increasingly confused over the past 5 days. He is awake and confused but is
noncombative. His speech is slurred, and his breath smells of alcohol. His temperature is 37.2°c (99°F), blood pressure is 180/60 mm Hg, pulse is 110/min, and respirations are 18/min.
Physical examination shows sixth cranial neNe palsy, horizontal diplopia, strabismus, and an asymmetric horizontal-gaze evoked nystagmus. Neurologic examination shows no focal
weakness or numbness. When helped up and told to walk, he has a broad-based, uncertain gait. When asked how he arrived.at the emergency department, he relates that "l drove to this
place to visit some friends." The most likely cause of these findings is a deficiency of which of the following?

0 A) Folic acid
0 B) Magnesium
0 C) Vitamin 8 1 (thiamine)
0 D) Vitamin B 12 (cyanocobalamin)
0 E) Zinc
30. A 67-year-old man with long-standing signs and symptoms of congestive heart failure is admitted to the hospital becau~e of progressiv~ shortness of breath. Examination shows no other
abnormarities.An x-ray of the chest shows cardiomegaly, cephalization of blood vessels, and a right-sided pleural effusion.Which of the following sets of pleural fluid findings is most
likely in this patient?

Protein Glucose Leukocyte count Segmented neutrophils Monocytes


(g/dl) (mg/dl) (/mm 3) (%) (%)
0
0
A)
B)
2.5
2.5
10
90
10,000
2000
50 50 ,
60 40
0 C) 3.8 40 30,000 80 20
0 D) 4.5 60 10,000 20 80
0 E) 4.5 90 2000 50 50

\
•I
.
.
'


The response options for the next 2 items are the same. Select one answer for each item in the set
'

For each patient with peripheral nerve dysfunction, select the most likely site of nerve injury.

0 A) Axillary nerve 0 J) Radial nerve above the elbow


0 B) Cervical nerve root at the cervical foramen 0 K) Radial nerve at the elbow
0 C) Long thoracic nerve O L) Radial nerve at the wrist
0 D) Median nerve above the elbow 0 M) Suprascapular nerve
0 E) Median nerve at the elbow 0 N) Thoracodorsal nerve
O F) Median nerve at the wrist 0 0) Ulnar nerve above the elbow
O G) Musculocutaneousnerve above the elbow O P) Ulnar nerve at the elbow
O H) Musculocutaneousnerve at the elbow O Q) Ulnar nerve at the wrist
0 I) Musculocutaneousnerve at the wrist

31. A previously healthy 42-year-old carpenter comes to the physician because of a 6-month history of pain and numbnessin his dominant hand that awakens him at night. He describes
numbness in his long and index fingers after driving for extended periods of time. Examinationshows minimal atrophy of the thenar muscles. There is normal sensation to light touch on
the little finger and the palm of the right hand. Sensation to light touch is decreased at the tip of the thumb, index finger, and long finger. X-rays of the right elbow and wrist show no
abnormalities.
For each patient with peripheral nerve dysfunction,select the most likely site of nerve injury.

0 A) Axillary nerve 0 J) Radial nerve above the elbow


0 B) Cervical nerve root at the cervical foramen 0 K) Radial nerve at the elbow
0 C) Long thoracic nerve 0 L) Radial nerve at the wrist
0 D) Median nerve above the elbow 0 M) Suprascapularnerve
0 E) Median nerve at the elbow 0 N) Thoracodorsal nerve
0 F) Median nerve at the wrist 0 0) Ulnar nerveabove the elbow
0 G) Musculocutaneousnerve above the elbow 0 P) Ulnar nerve at the elbow
0 H) Musculocutaneousnerve at the elbow 0 Q) Ulnar nerve at the wrist
0 I) Musculocutaneousnerve at the wrist

32. A previouslyhealthy 37-year-old man comes to the physician because of a 2-monthhistory of pain in the forearm and little finger of his dominant hand; he has been working as a
receptionist for 6 months. He describes numbnessin his little finger and weakness of his grip. There is decreased sensationto light touch at the tip of the little finger. Paresthesias are
elicited with compression of the cubital tunnel.
;,111,-.,11:I"uc,ar, I o f"l,.
••• ~-, 1·
1c.i I' ,-_:-:::1111·,,:Ic

■ l\1ark ComprehensiveClinical ScienceSelf-Assessment 45 min 14 sec

33. A 77-year-oldwomanis broughtto the physicianby her son for a routinehealthmaintenanceexamination.She saysthat she feels well. Her son reports that 1 monthago, she got lost
whiledrivinghomefrom the local supermarket.Two weeks ago, she forgot to turn off the stove after cooking dinner.She has been wearing bilateral hearing aids since audiometry 2 years
ago showedbilateralhigh-frequencyhearingloss. Her visual acuitycorrectedwith glasses is 20/25 in both eyes. Neurologicexaminationshows mild fine tremors of the hands when the
arms are outstretched;the tremor is not presentat rest Musclestrengthis 5/5 in all extremities.Deep tendon reflexesare decreased at the ankles and 2+ elsewhere.Her gait is normal.
1
Sensationto vibrationis mildlydecreasedover the toes. On mentalstatusexamination,she is awake,alert, and conversant.Her languagefunctionis normal. She is oriented to Pfrson,
place,and time and recallsone out of three objectsafter 1Ominutes.Which of the followingfindings in this patientwarrantsfurtherevaluation?
0 A) Decreaseddeep tendonreflexesat the ankles
0 B) Decreasedsensationto vibrationover the toes
0 C) High-frequencyhearingloss
0 D) Memoryloss
0 E) Tremorof the outstretchedhands

l
34. An asyn1ptomatic32-year-oldwon1anconies for a routinehealthn1aintenanceexamination.Her motherand sister have a historyof low back pain and disc herniation.The patient is a
postalworker.She weighs 86 kg (190 lb) and is 165 cn1(65 in) tall. Her blood pressureis 130/78mm Hg, pulse is 74/min,and respirationsare 16/min.Range of motion of the spine is
normaland withoutpain. Thereis no scoliosis or excessivekyphosisof the back. Which of the followingis the most effectivestrategyto decrease this patient's risk for developing low
back pain?

O A) Changein job
O B) Lin1itphysicalactivity
0 C) Stretchingexercises
O D) Weight-lossprogran,
0 E) Nonsteroidalanti-inflammatorydrug therapy
0 F) Prophylacticbrace


35. A 42-year-oldman comes to the emergencydepartmentbecause of a 2-week historyof increasinglysevere headachesand a 2-day historyof nausea,vomiting, neck stiffness,and
unsteadiness.He has type 2 diabetes mellitustreated with glyburide.His temperatureis 38.1°c (100.5°F).Funduscopicexaminationshows bilateral papilledema. Neurologic
examinationshows mild meningismus and diffuselybrisk deep tendon reflexes.He walks with a moderatelybroad-basedgait. He is able to recall two out of three objects after 5 minutes
and makes several errors on serial sevens.A CT scan of the head shows no abnormalities.Cerebrospinalfluid analysisshows a glucoseconcentrationof 18 mg/dl, a protein
concentrationof 108 mg/dl, and a leukocytecount of 59/mm3(1% segmented neutrophilsand 99% lymphocytes);a cryptococcalantigenassay is positive. Which of the following is the
most appropriate pharmacotherapyfor this patient?

0 A) Acyclovir
O B) Amphotericin B
0 C) ltraconazole
O D) Penicillin
0 E) Vancomycin


36 A 40 yeaJ-<>ld man 1sbroughtto the emergencydepartment 1 hour after a high-speed motor vehicle collision. On arrival, he is awake and alert but has severe pain over the sternum. His
S)'Sti»cbk>odPfesStSers 80 mm Hg, pulse 1s80/min,and respirationsare 10/min An ECG shows multifocalprematureventricularcontractions but no ST-segment changes. His Po 2 is
100 nwnHg Mer 1 L of ladated Ringer's solutJon1sadministered,his Po 2 decreases to 60 mm Hg while breathing4 Umin of oxygen by nasal cannula. Pulmonary capillary wedge
pressu-ehasrnoeasedfTom14 mm Hg to 24 mm Hg (N=1-10). Which of the followingis the most likelyexplanationfor the patient's poor response to fluid resuscitation?
0 A) n-deQlate adn"Nntstration
of flUJds
u a, t.Arocard1a1
cortus1on
:., C) t.tyoca,dial Infarct,on
,. 0) ~ry c.ortus,on
f) lrat.ma11c1t4>tu1e of the aorta

I"
37. A countyhealthofficer investigatesan outbreakof illness among personsattendinga churchpicnic. The illness is characterizedby the onset of nausea and vomiting 3 to 4 hours after
attendingthe picnic.All affected persons recoverwithoutspecific therapy.The investigationimplicates egg salad as the vehicle of transmission.This episode is consistent with a
foodborneoutbreakcaused by which of the following?

0 A) Clostridium perfringens
0 B) Giardia lamblia
0 C) Salmonella species
0 D) Staphylococcus aureus
38. A 62-year-oldwomancomes to the physicianbecause of bloating and cramping abdominal pain and intermittentdiarrhea over the past 5 years. Her symptoms have increased over the
past monthsince she started a new diet that emphasizesyogurtand cottage cheese as low-fatsources of calcium and protein. Vital signs are within normal limits. Abdominal
examinationshows diffusetendernessto palpationwith no reboundtenderness;there are no masses or organomegaly.Bowel sounds are increased. Test of the stool for occult blood is
negative.Which of the followingis the best explanationfor this patient's diarrhea?
0 A) Impairedintestinalmotility
0 B) Inflammatoryprocess
0 C) Malabsorption
0 D) Secretoryprocess
39_ A 52-year-01dwoman comes to the physician because of a 3-monthhistory of intermittentbloody discharge from the right breast. She does not perform regular monthlybreast self-
examinations.She has a 3-year history of major depressive disorder treated with fluoxetine.Examinationof the breasts shows no abnormalities_No masses are noted on palpation.
Serosanguineousfluid can be expressed from the nipple of the right breast by pressing on the left side of the areola. Which of the followingis the most likely diagnosis?

O A) Cystosarcomaphyllodes
0 B) Fat necrosis
0 C) Fibroadenoma
O D) Fibrocystic changes of the breast
O E) Galactorrhea
O F) Hyperprolactinemia
O G) lntraductalpapilloma
0 H) Mastitis
O I) Paget's disease of the breast

.,

..
40. A 42-year-Oldwoman comes to the physician because of right-sided abdominal cramps and nausea. During this period, she has noticed that her eyes are yellow.She has not had any
other symptoms and has no other history of jaundice. Two months ago, she underwenta laparoscopic cholecystectomyfor acute cholecystitis;intraoper~tivecholangiographywas not
performed at that time. She currentlytakes no medications. Vital signs are within normal limits. Exarninationshows scleral icterus. Cardiopulmonaryexaminatio·nsh<;>ws
no abnormalities.
The abdomen is nondistended with mild right upper quadranttenderness.There are no peritoneal signs. Bowel sounds are normal. Serum studies show:
,, ii '
j; ' +

:i
Bilirubin,total 8 mg/dl ,., '
Direct 5 mg/dl
; !
Alkaline phosphatase 650 U/L
. I

AST 20 U/L 'I,



ALT 18 U/L ....
.•
Abdominal ultrasonographyshows dilation of the intrahepatic biliary ducts. Which of the following).Sthemost appropriate next step in management? I"
" ''

,, '
'
I;
O A) CT scan of the abdomen ' '

0 B) HIDA scan
O C) Intravenousantibiotic therapy
O D) Endoscopic retrograde cholangiopancreatography
O E) Surgical exploration of the abdomen ',.
II .,,
..,
' I
,I

"
41. A 26-year-oldwoman is broughtto the emergency department because of marked confusionfor 2 hours; she also has had a flu..likeillness for 3 days. Overthe past 6 weeks, she has
had increased fatigue, weakness, and nausea. She recentlystarted thyroid hormone replacementtherapy for autoimmunethyroiditis; 1 week ago, her serum thyroid-stimulating
hormoneconcentrationwas 3 µU/mL. Her temperature is 38°C (100.4°F), blood pressure is 80/40 mm Hg, and pulse is 140/min. She appears confused and lethargic. Examination
shows cool, mottled skin. There is generalized hyperpigmentation,especially involvingthe palmar creases. The lungs are clear to auscultation.Abdominal·examination shows diffuse
mild tendernessand no rebound. Laboratory studies show:
Hemoglobin 10 g/dL
Leukocyte count 9000/mm3
Segmented neutrophils 55%
Eosinophils 20%
Lymphocytes 25%
Serum
Na• 124 mEq/L
c1- 92 mEq/L
K• 6.4 mEq/L
HCO3 - 16 mEq/L

An x-ray of the chest and urinalysis show normal findings. An ECG shows sinus tachycardia with peaked T waves. Which of the following is most likely to confirm the primary cause of
this patient's condition?

0 A) Measurementof pulmonaryartery pressure


0 B) Measurementof right atrial pressure
0 C) Measurementof serum antithyroglobulinantibody concentration
0 D) Measurementof serum lactate dehydrogenase activity
0 E) Measurementof serum thyroid-stimulatinghormone concentration
0 F) ACTH stimulationtest
0 G) Dexamethasonesuppression test
0 H) Blood cultures
0 I) Echocardiography
42. An 18-year-old man comes to the physician for an initial examination because of a 3-year history of fatigue and migrating joint pain. He has brought a large folder containing information
about previous medical consultations, laboratory tests, and x-rays. He takes no medications. I-le weighs 50 kg (11 O lb) and is 173 cm (68 in) tall. Physical examination shows no other
abnormalities. On mental status examination, he is preoccupied with his symptoms. When asked about his mood, he states that the future appears bleak, and that he is too tired to think
about it. Which of the following is the most appropriate next step in management?

0 A) Ask about further symptoms of obsessions and compulsions


0 B) Ask about sexual history including sexual orientation and practices
0 C) Ask about suicidal feelings
0 D) Ask about travel history over the past 6 months
0 E) Obtain a detailed exercise history
0 F) Measurement of serum Lyme (Borrelia burgdorfen) antibody concentration
O G) Urine toxicology screening

+ ,+ I
. . l I I :,, " I ' :' ' . .

43. A SO-year-oldwoman has had progressive dyspnea over the past 2 weeks and constant sharp chest pain for 4 days. The pain is localized to the center of the\:hest and is worse whlle ••I
) , t , 1, l

supine. She underwent a right, modified radical mastectomy and adjuvant chemotherapyfor breast cancer 3 years ago. She has a history of hypothyroi,dism·treaJed with thyroid ; ·i,,
replacement therapy. She has smoked one pack of cigarettes daily for 30 years and drinks two ounces of alcohol daily. She is dyspneic and diaphoretic. Her temperatur~ is.. 1 ': •• ••

37.2°C (99°F), blood pressure is 90/70 mm Hg with a pulsus paradoxus of 20 mm Hg, pulse is 110/min, and respirations are 28/min. Examination shows jug~lar venous.distention·to~~~-('., l:
angle of the mandible. The liver span is 14 cm with 4 cm of shifting abdominal dullness.Arterial blood gas analysis on room air shows a pH of ?'.50, Pc,o2 ;of 30 mn) Hg; and Po2.of . ' . : ,.:::".
70 mm Hg. An x-ray of the chest shows an enlarged cardiac silhouette with a globular configuration.An ECG·shows sinus tachycardia with nonspecific·ST-segment changes diffusely., ' , f
1
Which of the following is the most appropriate next step in management? I ' • ., • :1. , ' ...
f ·1 i1 . ,- 11;1 i.':1 •'1h·1i
111 i,1r!!i : •
O A) Echocardiography ' , I :id • '!i ' ,11'· 'l • !111: I;'· • '...'. I ,,

.I • I . d: t , I • ~
O B) CT scan of the abdomen I' • ! I • 11 I["~

l' I
i• :-.~~;
I! ' I•
I, 1,I : • ,. ~ fl°. I f_·
Ij1: 1
O C) Ventilation-perfusion lung scans •I l
' • : '1 ·l 'I l
, . • ...

Il .
!' II .•
l' I' • ,: I· I ' \
• ·~ : : 1l • ·1 1·
OD) Bronchoscopy • • : •, ~ • I I

.: : '' . ,....
1
0 E) Paracentesis 1 1 .
: 1 ' t

' ,,,'q )1'1'1·~IiI I 1;,• •


ISi ii P 1 I , I
l'i\ 1·\j'i'l1i i ' '
'l • ' I • I
:i!;·'I' I.
I l ,..
i'F . •
l I • I
1',:, I ; l
'.
Ii•
.. I l1 :
1

' 11·
•i '
Il
'
t,·O
:• f :-~
l
• L ii ' :i
1
1
i I
•t• .l I
.' ..
l
! • '1·1,
•'
•jl l'I • . ' '. I·1
,• I
..' •
II II 11'i1•;,'1
l•lI
-: ' I

u
'• II l •

111
!1
t
I
!:i
.I
qI :I
I
, :i
(
'• .
'I
I III..' I' I I .. '
' '
t
•'
'
'
I •

' "

,.
11I ••
·It):
. h, •
.• .
, !.
••

.'
1• i '• I '
'
44. A previouslyhealthy67-year-old man comes to the physician because of a 3-month history of right leg pain after walkingtwo blocks. He has smoked two packs of cigarettes daily_for 50
years. He is 165 cm (5 ft 5 in) tall and weighs 77 kg (170 lb); BMI is 28 kg/m2_Examinationof the right lower extremityshows shiny skin and decreased hair growth. Peripheralpulses are:
Femoral Popliteal Dorsalis Pedis Posterior Tibial
I I
Right 3+ 2+ 0 0
Left 3+ 2+ 2+ 2+ ., ;\ I
• '\ I I
l
The right ankle brachiaI index is 0.6 {N>1), and the left is 0.9. In addition to smoking cessation,which of the following is the most appropriate next step in m.anag·ement?

O A) Daily exercise program l


II
0 B)Angioplasty l'
1,

O C) Arteriography
I I
tiI' J
O D) Femoropopliteal bypass grafting
1 j·
•'

I ,'..
I' I

' .
.'
'
45. A 64-year-old woman has moderately severe postoperative pain 1 day after a total abdominal hysterectomy and bilateral salpinga-oophorectomy. Which of the following is the most
appropriate analgesic pharmacotherapy? • · ,l
I 11.11 ., , !
0 A) Oral aspirin-codeine compound
O B) Oral diazepam
' '
0 C) Oral ibuprofen
..
O D) Intermittent intravenous naloxone
0 E) Patient-controlled intravenous morphine I

0 F) Transcutaneous administration of fentanyl l


. '
I I ' l
I l
• I

' .
I •
Ii • J'

I, •
I I

I II :I I
,
)
I
I II I IIi,I. I I
I ;

Ii , I I
I I'
II I I II l
I, ' I
I

I
I •

Ij ,1 I II
I I Ir I. .
I
'
1,
'I
• l! J
j'
Iii . I

I
l
1 11
fI
II
I
• 1 'II
II
I
!,

,
,..I
,11'
Il
lI 1
I

I
I i I
I I '
I
I I I
I
I
• I I II
'
I
I l1
II
/II
1
/) . l,
•1/11/!
ll1i
l/1/, ·:1., (
'.I
46. Two days after admission to the hospital for treatment of severe left lower lobe pneumonia,a 72-year-old man has increasinglyworse renal function. On admission, his blood pressure
was 80/40 mm Hg. Serum studies showed a urea nitrogen concentrationof 14 mg/dl and a creatinine concentrationof 0.7 mg/dl. He received 4 L of intravenousfluids. He has been
receiving ceftriaxone and azithromycintherapy since admission. He has hypertension,type 2 diabetes metlitus,and osteoarthritis.Medications also include glyburide,
hydrochlorothiazide,and naproxen. His temperature is 37.8°C (100°F), pulse is 92/min, respirations are 18/min,and blood pressure is 126/80 mm Hg. Skin turgor is normal.Jugular
venous pressure is 4 cm H20. Pulmonary examination shows egophony in the left posterior tungfield Serum studies today show a urea nitrogen concentrationof 32 mg/dl and a
creatinine concentrationof 1.9 mg/dl. His urinaryfractional excretion of sodiun1is 3%. His urine outputis 300 mU24 h. His postvoid residual volume is 70 ml. Urinalysis shows muddy
brown casts. Renal ultrasonographyshows no abnormalities. Which of the following is the most likely cause of this patient's increasinglyworse renal function?

O A) Acute interstitial nephritis


0 B) Acute tubular necrosis
0 C) lntravascularvolume depletion
0 D) Obstructive uropathy
O E) Progression of diabetic nephropathy
'
1_ A 17-year-old boy sustains a head injury and loses consciousness after diving into a freshwaterlake from a platform.After being submergedfor 3 minutes,he is rescued_He is initially
cyanotic but then begins to cough and breathe spontaneouslyafter a 5-minute resuscitation.Over the next 36 hours,which of the following complicationsis most likely? ·\
) !• '
O A) Acute respiratory distress syndrome I ' ,
1 a,1
Ij
O B) Bacteremia \l ' I • I .
O C) Bacterial pneumonia ' •I l\,
I •: i
OD) Hypernatremia
O E) Metabolic alkalosis

\\
,,
I \I'
1•
rl,'
. l

.
I I

l
'
II ,IiI
..,'! . 1,
' I
I
I
I
I•,·
' •
II
I

I I " ,ii•
~...1 1\
·!

[\ 1]·1 .. '.

'I
•I

T
!l • ' ,
I
• I

1 l

II
I I
a
2. A 30-year-old woman comes to the physician for follow-up examination 4 months after starting treatment with lithium carbonate; during this period she has had a 3.2-kg (7-lb) weight gain.
Her medication was prescribed soon after the birth of her son to treat racing thoughts, increased spending, and overtalkativeness; she has had difficulty sleeping for more than 2 hours .
nightly. Her symptoms decreased after 2 weeks of lithium carbonate therapy. She continues to take lithium carbonate (300 mg three times daily). Her temperature is 37°C (98.6°F), blood
pressure is 120/80 mm Hg, and pulse is 70/min. Examination shows normal findings. The most appropriate next step is measurement of which of the following?
I
O A) Fasting serum glucose concentration ••
' j

0 B) Leukocyte count
O C) Serum alkaline phosphatase activity
O D) Serum creatinine concentration
.,
O E) Serum thyroid-stimulating hormone concentration

'I
3. A 62-year-old woman is brought to the physician by her husband because of a 2-day history of fatigue and a 1-hour history of headache and confusion. She has not had fever, nausea, or
vomiting. She has a 6-year history of hypertension treated with hydrochlorothiazide and lisinopril. Her temperature is 36.1 °C (97°F), pulse is 90/min, respirations are 22/min, and blood
pressure is 250/135 mm Hg. Funduscopic examination shows bilateral papilledema. The lungs are clear to auscultation. Cardiac examination shows an S 4 and no murmurs. An ECG
shows left ventricular hypertrophy with a strain pattern. Which of the following is the most appropriate initial pharmacotherapy?

0 A) Clonidine
O B) Hydralazine
0 C) Nifedipine
O D) Nitroglycerin
O E) Nitroprusside
•' I I t ,: l 'I
j " • + •

4. A 42-year-oldman is broughtto the physician by his wife because of a 2-monthhistoryof staring spells.that1a·st1 to ,2 minuteseach. Duringepis9des, he alSQismacks.his lips and pioks ~t -~
his shirt collar_Four years ago, he was comatose for 2 weeks after sustaininga head injuryin a motorcyclecpllision;he r~quired.6monthsof reh~bilitation:H~ repo,rts,·_tt:iat,
ayerthe past· :, •
year, he has had intermittentepisodes of smelling burnt rubber that occur approximatelyevery2 weeks. He hears an intei'nse ,hissingsound duringthese episodes:'Exan:iination·shows ~o
abnormalities.Which of the following is the most likely diagnosis? i 11
.,
. •
.
• '
,
, . .. , .' : • ,•
i, . •. . II
I :~,\\ :f, t -~, (
' I ! • '
O A) Absence seizures • I I
I .' ·•1, ··1 , . • •' •
.,.
l , . .' .'.
O 8) Complexpartial seizures ,:
..
O C) Transient ischemic attack I l1
lI ' I

O D) Tourette's disorder . ..
O E) Limbic encephalopathy ''
,
I
I I
. '
I
I "
•I

. :

.
!
.
I,

.'
,,
: •'
..'

.
'
l 1

' ' I
I l\ ' I• l\ •
. ' "·
.:l
''
l . ,
' I
I I
.
'

II , \
....
I
' .
I '
I· •
' '
5. An asymptomatic 21-year-old woman is found to have an adnexal mass on pelvic examination. She uses oral
contraceptives. A photograph of the mass is shown. Which of the following is the most likely diagnosis?
O A) Benign cystic teratoma
0 B) Corpus luteum cyst
O C) Oysgerminoma
OD) Endometrioma
O E) Mucinous cystoadenoma


I I

..•
I
' '
6. A previously healthy47-year-old woman comes to the emergency department because of a 36-hour history of nausea, vomiting, and abdominal pain that radiates to her back. Over the
past 3 years, she has had intermittent episodes of cramping abdominal pain 1 to 2 hours after meals; the pain lasts for several hours and resolves spontaneously.She does not smoke
and drinks one to two glasses of wine each evening. There is a family history of coronary artery disease and hypertension.Her temperature is 37°C (98.6°F), blood pressure is
100/60 mm Hg, pulse is 120/min, and respirations are 20/min. Abdominal examination shows moderate epigastric and right upper quadrant tenderness with no guarding'or rebound;
bowel sounds are decreased. Laboratory studies show:
Hematocrit 45%
Leukocyte count 9000/mm3 with a normal differential
Serun,
Total bilirubin 1.5 mg/dL
Alkaline phosphatase 120 U/L
AST 78 U/L
Amylase 365 U/L
Lipase 1223 U/L (N=1-160)
Triglycerides 300 mg/dl

Which of the following is the most likely diagnosis?

O A) Acute cholecystitis
O B) Alcoholic hepatitis
O C) Alcoholic pancreatitis
O D) Ascending cholangitis
O E) Gallstone pancreatitis
0 F) Hepatitis A
0 G) Pancreatic cancer
0 H) Peptic ulcer disease
.: ...; •
l: .1
.

0 I) Triglyceride-induced pancreatitis ..<• . )

)
7. A 44-year-old woman comes to the emergency de·partment 2 days after being discharged from the hospital for abdominal pain; an exploratory laparoscopy showed no abnormalities. over
the past 5 years, she had been admitted to the hospital numerous times for the evaluation of a variety of symptoms; all work-ups had been negative. Her temperature is 39.2°C (102.6°F).
Examination shows a reddened, indurated laparoscopic wound. She is admitted to the hospital and given intravenous antibiotics. On the second hospital day, a nurse wi·tnesses the
patient rubbing saliva into her laparoscopy site. Which of the following is the most likely diagnosis?

O A) Adjustment disorder with disturbance of conduct


O B) Conduct disorder
O C) Conversion disorder
O D) Factitious disorder
O E) Hypochondriasis
O F) Major depressive disorder
O G) Oppositional defiant disorder
O H) Somatization disorder
8. A 57-year-old man comes to the physician because of intermittenturinaryincontinenceoverthe past 6 months.He has loss of small amountsof urine when he coughsor sneezes.He has
not had pain or blood with urination. He has a 15-year histo[Yof ty e 2 diabetes mellitus\vith peripheralneuropathy,retinopathy,and gastroparesis. Currentmedicationsinclude
metoclopramide and glyburide. He appears well. Rectal examination shows a normal-sizedprostate. Neurologicexaminationshowsdecreased sensationin a ~toeking.cglove distnbUtion..,
Achilles tendon reflexes are absent bilaterally.Test of the stool for occult blood is negative.Urinalysisshows2+ proteinwith no leukocytesor erythrocytes.Hi~ posi'J-d reStdlBlvokmeis
1 1
500 mLJ.Which of the following is the most likely mechanism of this patient's incontinence? 1 ' ',

- A) Central nervous system disorder


B) Functional incontinence
C) Intrinsic sphincter deficiency
D) Overflowincontinence from acontractile bladder
E) Overflowincontinence from bladder outlet obstruction
F) Pelvic floor muscle weakness
G) Retroperitonealfibrosis
H) Retroperitoneal lymphadenopathy
I) Urinarytract infection


·g_ A 6-year-old boy is brought to the physician by his mother because of progressive visual loss over the past year. Overthe past 2 years, he has had deteriorationof his hearing,speech,
writing, and intellectual performance. His maternal uncle had similar symptoms.Visual acuity is 20/200 bilaterally.Funduscopicexaminationshowsoptic atrophy.His hearingis markedly
impaired. There is weakness and spasticity of all extremities. Deep tendon reflexesare extremelyhyperactive.Babinski's sign is presentbilaterally.On mentalstatusexamination,he is
not oriented to place, year, month, or the names of his siblings. An MRI of the brain shows marked symmetricwhite matterdisease involvingall lobes. Diagnpsticstudiesare most likelyto
show which of the following?

O A) Abnormally decreased serum cholesterol concentration


O B) Acanthocytes on blood smear
O C) An excess of very long chain fatty acids
O D) Normal nerve conduction studies
O E) Vitamin E deficiency

i1. II I I
II I
' f:•

II
ii

I I II
. l,]11 /1 I)
'I • I1 11 I II I I
I

'
10. A 32-year..oldwoman comes to the physician because of bright red rectal bleeding and severe stabbing pain with each bowel movementover the past 2 weeks. She has blood-streaked
stools, and there is blood on the toilet paper. Over the past 2 months,she has had mild constipationwith no change in the caliber of the stool. Examination shows a small anal fissure at
the posterior midline. Rectal examination is painful, but no abnormalities are detected except for a small amountof bright red blood from the fissure. Which of the following is the most
appropriate next step in management?

0 A) Anesthetic ointmentand stool softeners


0 B) Anal dilatation under anesthesia
0 C) Oebridementand closure of the fissure under anesthesia
0 D) Surgicalflaps
0 E) Lateral internalsphincterotomy
11. A 45-year-old \Von1ancon1es to the en1ergency departn1ent because of shortness of breath, chest pain, dizziness, and mild nun1bness and tingling around the lips for 2 hours. She says
that she feels like she is going to die. She had three sin1ilar episodes last week when she was vacationing at the Grand Canyon; the first episode occurred while crossing a narrow bridge
on a donkey. She takes a hypoglycemic drug for type 2 diabetes mellitus, verapamil for hypertension, and sumatriptan as needed for migraine. She is mildly diaphoretic and appears
pale. Her blood pressure is 130/90 mm Hg, pulse is 120/niin, and respirations are 28/min. Serun1 glucose concentration is 120 mg/dl. An ECG shows sinus tachycardia. Sublingual
nitroglycerin therapy does not relieve her syn1ptoms and gives her a headache. The most appropriate next step in n1anagement is administration of which of the following?

0 A) Haloperidol
O B) Lorazepam
0 C) Oxygen
O D) Sumatriptan
0 E) Verapamil
12. A 57-year-oldmancomesto the physicianbecauseof a 1-dayhistoryof severepain and swellingof his right kneethatworsenswith movementof the knee. He has no historyof serious
illness.His ten1peratureis 37.2°C (98.9°F),pulseis 80/min,respirationsare 16/min,and blood pressureis 138/78mm Hg. Examinationof the right knee showswarmth,erythema,and
diffusetendernesswith a n1oderateeffusion.Aspirationof the kneejoint yieldscloudyfluid;fluid analysisshowsa leukocytecountof 9000/mm3. Microscopicexan1inationshows
negativelybirefringent,needle-shapedcrystals.Whichof the followingis the mostappropriateinitial pharmacotherapy?

0 A) Allopurinol
0 B) Hydroxychloroquine
O C) lndomethacin
0 D) Prednisone
0 E) Probenecid

\\

\
13. A previouslyhealthy87-year-oldwoman comes to the physicianbecauseof a 4-monthhistoryof vulvaritching.Examinationshowsexcoriatedareas from scratchingand a white,thin
vulva.The labia minora are absent,and there are smallfissures at the introitus.The remainderof the examinationshowsno abnormalities.Which of the followingis the most likely
diagnosis?

0 A) Escherichia coli infection


O B) Lichensclerosus
0 C) Squamouscell carcinoma
O D) Trichomoniasis ' .
O E) Vulvarmelanoma \ .l
0 F) Vulvarvestibulitis ii

' ,
' .
l I

rn

II. 1
• '
'

l
,,i\

11 ' 'I I
\
'I l '
1
, I
• t
14. A 20-year-oldAfrican American man with sickle cell disease comes to the physicianbecause of a 1-week historyof shortnessof breath on exertion,fatigue, and generalizedweakness
He has had no fever, chills, night sweats, or cough productiveof sputum.His only medication is oxycodonefor joint pain. He weighs 68 kg (150 lb) and is 168 cm (66 in) tall. His
temperatureis 36.7°C (98°F}, blood pressure is 120/70 mm Hg, pulse is 76/min,and respirationsare 18/min.Examinationshows no abnormalities. Laboratory studies show:
Hematocrit 20°/o
Mean corpuscularvolume 110 µm 3
Leukocytecount 2300/mm3
Reticulocytecount 1.8%

Whichof the following is the most likely mechanismfor these findings?

0 A) Adverse drug reaction •

0 B) Atrophyof gastric mucosa


0 C) Bacterialovergrowthin the small intestine
0 D) Increaseddemandfor folic acid
0 E) Increaseddemandfor vitamin B 12 (cyanocobalamin)
0 F) Malabsorption
0 G) Vitamin 8 1 (thiamine)deficiency


I
I! I I + : I ' ! I ' •' '

15. A 16-year-oldboy with neurofibromatosis is brought for a follow-up examination. His uncle also has neurofibrom.atosis.He has a 1-year history of t{eadachesduring which.hispar~nts s.ay' ,..'.!
he appears pale. Six months ago, he underwentoperative treatment for an optic nerve glioma. His blood pressure is 164/105 mm Hg, pulse is 102/min,and respirations are 14/min.The . •
thyroid ~lands_arenot enlarged. No murmurs are heard, and radial pulses are equal. Abdominal examin'ationshows no abnprmalities.Which of the.followin~ is the r,nostlikely cau~e~f.., • ,;1
this patient's high blood pressure? '; • l 1
1
1! 1. ,, • . ,l 1 . j • !·
' I II 1· ,·i\ ''! ,,''·

O B) Carc1no.ma of the thyroid gland \j I ii


·,\\•
1
.,: 1'1 ,1
I I •
0 C) Essential hypertension
O D) Overproductionof aldosterone from an adrenal adenoma
O E) Postsubclaviancoarct.ationof the aorta
I I

I
.I
\I 1? •I \'1
Il .' •
•II , .
I.
li •'
'
1•

,ll
II
l

.
I •
"

1• •
II

,ti I.
1 •I
.'
1l •I
'
1
• •
·16. A 67-year-old woman is brought to the emergency department because of severe chest pain 4 hours after undergoing outpatient endoscopy and dilatation of an esophageal stricture
caused by reflux. At discharge, she reported no chest pain. Three hours later, she vomited a small amount of blood and had severe pain. She is pale. Her temperature is 38°C (100.4°F),
blood pressure is 140/85 mm Hg, pulse is 125/min, and respirations are 22/min. Examination shows crepitus in the neck and moderate epigastric tenderness. The lungs are clear to
auscultation, and breath sounds are equal bilaterally. Rectal examination shows no masses; test of the stool for occult blood is positive. Which of the following is the most likely cause of
these symptoms?

0 A) Bleeding from erosive esophagitis


0 B) Esophageal perforation
0 C) Mallory-Weiss syndrome
0 0) Myocardial infarction
0 E) Perforated gastric ulcer

•I
!
I
II
• I II,
111 ·11• 11!1· 1i
., 'ill
11
.11· ·l
I I
17. Over the past 3 months, a 30-year-old woman has had intermittent episodes of headache, palpitations, sweating, and irritability. Her blood pressure is 150/100 mm Hg while supine and
149/100 n1m Hg while standing, and pulse is 90/min while supine and 110/min while standing. Examination shows no abnormalities except for pallor Which of the following is the most
likely location of this patient's lesion?

0 A) Adrenal gland
0 B) Brain
OC)Heart
0 D) Kidney
0 E) Thyroid gland
18. A 42-year-old man comes to the physician because of a 3-month history of progressive shortness of breath with exertion that began after he had an upper respiratory tract infection. He .,
says he has had progressive respiratory difficulty during the past 5 years. He has no other history of serious illness and takes no medications. His father and paterr)al grandfather had
"lung and liver problems." The patient has smoked one pack of cigarettes daily for 25 years. He drinks one beer daily. He works as an automobile mechanic. Respirations are 16/min.
Pulse oximetryon room air shows an oxygen saturation of 90%. On pulmonaryexamination,expiratorywheezes are heard bilaterally.The remainder of the examination shows no
abnormalities.Serum studies show an alkaline phosphatase activity of 100 U/L, AST activity of 60 U/L, and ALT activity of 76 U/L. Which of the following is the most likely diagnosis?

O A) Asbestosis
0 B)Asthma
0 C) Emphysema
0 D) Hypersensitivitypneumonitis
0 E) Lung cancer
19. A 70-year-oldman comes to the physician because of urinaryhesitancyand frequencyfor 9 months.His temperature is 37.5°C (99.5°F). Examinationshows a circumcised penis with no
urethraldischarge. Testicular examination shows no abnormalities. Rectal examinationshows an enlarged rubberyprostate that is nontenderto palpation. Urinalysisshows many
leukocytesand no erythrocytes.Gram stain of urine shows gram-negativerods. Which of the following is the most likely cause of this patient's condition?
0 A) Infectionof the epididymis
O B) Infectionof the prostate
O C) Infectionof the urethra
0 D) Neurogenicbladder
0 E) Outflowobstructionof the bladder
20. A 32-year-oldman comes to the physician because of intermittentdiarrhea for 6 months. He has had a 5.4-kg (12-lb) weight loss during this period. Vital signs are within normal limits.
Examinationshows no abnormalities. Serum studies show a calcium concentrationof 7.8 mg/dl and a phosphorus concentration of 2.2 mg/dl. Examination of a stool specimen shows
fat globules. Which of the following is the most likely explanationfor this patient's hypocalcemia?
0 A) Parathyroid hormone deficiency
I
0 8) Vitamin A excess :1
0 C) Vitamin 8 1 (thiamine) deficiency \'
I
.!
I 'I
0 0) Vitamin 8 12 (cobalamin) deficiency
0 E) Vitamin D deficiency
0 F) Vitamin E deficiency
0 G) Vitamin K deficiency

I .'
''
21. A healthy24-year-old woman comes for a routine health maintenanceexamination.Menses occur at regular28-day intervalsand last 5 to 6 days. Her last menstrualperiod was 3 week~
ago. She takes no medications. Bi manual examination shows a 5-cm, mildlytender left adnexa.A pregnancytest is negative.Which of the followingis the most appropriate:nextstep in
management? II !I •I
' , ••
.
:1 I
O A) Repeat examination in 2 weeks ,, '·
O B) Measurement of serum CA 125 concentration !
11·111
I .I,I I
O C) Measurement of serum a-fetoprotein concentration .. I
!
O D) CT scan of the pelvis I ii

O E) Diagnostic laparoscopy I'

r J .' '
I '
I ''

.'
'Ii 'I
Ill
11 Ii

I
II Iii
11 ../
11:
11
.'
22. A 72-year-old man comes to the physician because of a 2-month history of urination twice nightly and occasional urinary frequency and urgency. He has a 15-year history of type 2
diabetes mellitus now moderately well controlled with glyburide. His father was diagnosed with prostate cancer at the age of 70 years, and his sister died of complications from systemic
lupus erythematosus.His blood pressure is 135/86 mm Hg. Cardiopulmonaryexamination shows no abnormalities. Abdominal examination shows no suprapubic fullness or tenderness.
There is mild enlargement of the prostate with no palpable nodules. His postvoid residual volume is 10 ml. Serum studies show a urea nitrogen concentration.of 45 mg/dl and creaUnioe
concentrationof 3.8 mg/dl. Urine dipstick shows 3+ protein. Which of the following is most likely to have prevented progression of this patient's renal disease? I
0 A} IntermittentFoley catheterization
0 B} Intravenousmannitol therapy
O C} Oral cyclophosphamide and prednisone therapy
0 D} Oral enalapril therapy
0 E} Oral finasteride therapy
0 F} Oral prednisone therapy only
0 G} Oral terazosin therapy

I l
I
23. A 22-year-old primigravid woman at 16 weeks' gestation is brought to the emergency department because of progressive shortnessof breath over the past 48 hours. Her temperatureis
37°C (98.6°F), blood pressure is 120170mm Hg, pulse is 100/min, and respirations are 24/min. Scattered wheez.esare heard. Pelvic examination shows·a uterusthat extendsto the
umbilicus. Fetal heart tones are absent. Her hematocrit is 32%, leukocyte count is 11,000/mm3,and serum 13-hCGconcentrationis 300,000 mlU/mL. Pulse oximetry on room air showsan•
oxygen saturation of 92%. An x-ray of the chest shows multiple round densities throughoutall lungfields. Which of the following is the most likely diagnosis?
'

O A) Bacterial pneumonia
O B) Choriocarcinoma
O C) Pulmonaryembolism
O D) Tuberculosis
O E) Viral pneumonia

I
I
I'

II •
'
24. A healthy 23-year-old woman, gravida 1, para 1, comes to the physician requesting postcoital contraceptive advice. One day ago, her partner's condom broke during sexual iritercourse.
Her last menstrual period was 2 weeks ago. She takes no medications. Physical examination shows no abnormalities. A urine pregnancy test is negative. Which of the following is the
most appropriate recommendation for this patient?

O A) lntravaginal spermicide now


0 B) Oral gonadotropin-releasing hormone agonist now and again in 12 hours
O C) Oral levonorgestrel now and again in 12 hours
0 0) Oral prostaglandin F 20 now and again in 12 hours
0 E) Dilatation and curettage immediately
25. A 67-year-oldwoman comes for a routine health maintenance examination. She drinks 1 ounce of alcohol daily. Her blood pressure is 138/62 mm Hg, pylse is 76/min and regular,and
respirations are 14/min. The lungs are clear to auscultation.The remainder of the examinationshows no abnormalities. Laboratory studies show;
Hemoglobin 12.8 g/dl
Serum
Ca 2+ 11.9 mg/dl
Creatinine 0.8 mg/dl
Phosphorus 2.8 mg/dl
Total protein 6.5 g/dl
Albumin 4.2 g/dl
Ionized calciurn 5.8 mg/dl (N=4.5-5.1)
•I

Which of the following is the most appropriate next step in management?

0 A) Measurementof serum alkaline phosphatase activity


0 B) Measurementof serum parathyroid hormone concentration
0 C) Measurementof serum vitamin D concentration
0 D) Serum and urine protein electrophoresis
0 E) X-ray of the chest
0 F) Skeletal survey
0 G) Bone scan

• '
26. A 67-year-old man is brought to the emergency department because of a 3-day history of fever and headache. Five years ago, he underwent placement of a mechanical aortic valve for
treatment of sequelae of rheumaticfever. He appears ill. His temperature is 40°C (104°F), blood pressure is 110/65 mm Hg, pulse is 110/min, and respirations are 22/min. A grade 3/6,
systolic ejection murmur is heard. Neurologic examination shows mild left hemiparesis. Babinski's sign is present on the left. There is no nuchal rigidity. This patient is at greatest risk for
which of the following complications?

O A) Brain abscess
O B) Carotid artery occlusion
0 C) Encephalitis
0 D) Hydrocephalus
0 E) Venous sinus thrombosis


Theresponseoptionsfor the next2 items are the same. Select one answerfor each item in the set.

For each patientwith loss of consciousness,select the most likely diagnosis.

O A) Aortic stenosis 0 G) Orthostatichypotension


0 B) Carotid sinus hypersensitivity 0 H) Pulmonaryembolus
0 C) Conversionreaction 0 I) Seizure
O 0) Hypertrophicobstructivecardiomyopathy 0 J) Vasovagalsyncope
0 E) Hypoglycemia 0 K) Vertebrobasilarinsufficiency
0 F) Mitra!valve prolapse

27. A 15-year-oldboy is broughtto the emergencydepartment30 minutesafter a 2-niinuteepisode of loss of consciousnessafter completinga 400-meter race. On awakening,he says that
he feels fine exceptfor shortnessof breath. He weighs 82 kg (180 lb) and is 191 cn1(75 in) tall. His blood pressureis 110/70mm Hg, pulse is 70/minand regular,and respirationsare
15/min.The lungsare clear to auscultation.A grade 2/6 systolicmurmuris heard at the left sternalborderwrthn1inimalradiationto the neck; the murmurbecomes louder when he stands.
For each patient with loss of consciousness, select the most likely diagnosis.
I'
O A) Aortic stenosis 0 G) Orthostatic hypotension I\
O B) Carotid sinus hypersensitivity 0 H) Pulmonaryembolus
O C) Conversion reaction 0 I) Seizure
II
0 0) Hypertrophicobstructive cardiomyopathy 0 J) Vasovagal syncope
O E) Hypoglycemia 0 K) Vertebrobasilar insufficiency
O F) Mitra!valve prolapse

28. A 62-year-old woman is brought to the emergency department 1 hour after a 1-minuteepisode of loss of consciousness;her symptoms began when she stood up after she passed a
dark, watery stool. She has had diarrhea and dark stools for 2 days. She has been receivingwarfarin therapyfor deep venous thrombosis for 2 weeks. On arrival, her blood pressure is
82/60 mm Hg, and pulse is 150/min and regular. She is unable to stand. The lungs are clear to auscultation.A grade 2/6 systolic murmuris heard at the second right intercostal space
with no radiation. Examination shows a soft, nontender abdomen. There is 1+ edema of the right lower extren1itywith no tenderness.Test of the stool for occult blood is positive..
I .
11

l I II
I
29. A 47-year-old woman comes to the physician for a follow-up examination. She has a 15-year history of poorly controlledtype 2 diabetes mellitus. She currentlyrequires 100 U of insulin
daily. She has had no medical complications of her diabetes. She has significant limitations in activities of daily living as a result of obesity. Multiple attempts at weight toss have been
unsuccessful.She is compliant with her diet, insulin regimen, and daily monitoring of blood glucose concentrations.She is 163 cm (5 ft 4 in) tall and weighs 198 kg (437 lb); BMI is
75 kg/m2• Her blood pressure is 150/95 mm Hg. The remainder of the examination shows no abnormalities. Her serum creatinine concentration is 0.8 mg/dl. Which of the following is the
most appropriate next step in long-term management? 11

0 A) Liquid protein diet


0 B) Measurement of blood glucose concentrations every 4 hours
O C) Implanted insulin pump
O D) Insulininfusion pump I
0 E) Gastric bypass
0 F) Pancreas transplant
30_ An 18-year-oldman comes to the physician 1 week after he had a blood pressure of 140/11o n1mHg during a routine precollege examination.His temperature is 37.1°C (98.7°F),blood
pressureis 140/100mm Hg, pulse is 92/min, and respirationsare 12/min.The upper extremitiesappear to be n1oremuscularthan the lower extremities. Radial pulses are normal;
femoral,posterior tibial, and dorsalis pedis pulses are decreased. A grade 2/6 systolic murmuris heard over the precordium,anterior chest, and back. An ECG shows left ventricular
hypertrophy.Which of the following is the most appropriate next step in management?

O A) Limiting physicalactivity
0 B) Repeat blood pressure measurementin 1 month
0 C) Initiatea low-sodiumdiet and exercise program
0 D) Pharrnacologicmanagement
0 E) Operativetreatment

I I
31. A previouslyhealthy3-month-oldgirl is broughtto the emergencydepartmentbecauseof a 3-dayhistoryof gruntingand increasingdifficultybreathing.She appearsill. Hertemperatureis
36.7°C (98°F),pulse is 160/min,and respirationsare 76/min.Examinationshowsgrunting,nasalflaring,and markedintercostalretractions.Bronchialbreathsoundsand occasional
bilateralcracklesare heardon auscultation.Serumstudiesshow:
Ca2+ 5.6 mg/dl
Phosphorus 11 mg/dl
Alkalinephosphatase 250 U/L

Capillaryblood gas analysison 100% oxygen:

7.36
38 mm Hg
46 mm Hg

An x-rayof the chestshows bilateral,diffuseinterstitialinfiltratesand absenceof the thyn,icshadow.Bronchoalveolar


lavageis positivefor numerousPneumocystisjiroveci (formerlyP.
carini1).Whichof the followingis the most likely n1echanisn1 of thesefindings?

O A) Adenosinedeaminasedeficiency
O B) Consumptionof complement
0 C) Defectiveopsonization
O D) Destructionof CD4+ T lymphocytes
O E) Developmentalarrest of maturationof B lymphocytes
O F) Dysmorphogenesisof the third and fourthpharyngealpouches
O G) Impairedchemotaxis
O H) Impairedphagocyticoxidativemetabolism
tL1,ic;11c1i '::,,,11,i ,,j f.h 0
1ii, ,,I I x;i·1,i·1,.-1,;
■ Mark Comprehensive Clinical Science Self-Assessment 39 n1in 23 sec

32. A 28-month-old boy has a history of cyanosis since birth with episodes of syncope. Examination shows cyanosis and clubbing. The lungs are clear to auscultation. There is a right
ventricular heave, a systolic click, a single S 2, and a grade 3/6 systolic murmur. Which of the following is the most likely diagnosis?

O A) Bicuspid aortic valve


O B) Coarctation of the aorta
O C) Mitral stenosis
O D) Patent ductus arteriosus
O E) Tetralogy of Fallot

I I I
I

33. A 13-year-old boy is brought to the physician by his mother for an examination prior to participation in sports. He has no history of serious illness and takes no medicati9ns. His maternal
and paternal grandfathers have hypertension and type 2 diabetes mellitus. The patient is at the 95th percentile for height and above the 95th percentile for weight and BMI. His
temperature is 36. 7°C (98.1 °F), pulse is 84/min, respirations are 16/min, and blood pressure is 130/83 mm Hg (95th percentile). Examination shows no abnormalities. In addition to
recommending participation in sports, which of the following is the most appropriate pharmacotherapy?

0 A) ACE inhibitor 1
1•
t '

0 B) 13-Adrenergicblocker
O C) Calcium channel blocker
O 0) Thiazide diuretic
O E) No pharmacotherapy is indicated ' .'
I

I r l !

I
I I '•'
1 •

''
I I •\ •
I ..
I I
..
\ •
I !
I 11, 'i •
.
l

\ 11
34. A 72-year-oldman comes to the physician with his wife because of chronic abdominal pain and headachesfor 4 months.His wife states that he has become more forgetful over the past
6 months.He has a history of gout. He has smoked one pack of cigarettes daily for 50 years and drinks 1O oz of homemade whiskey daily. He takes no medications. His temperatureis
36.8°C (98.3°F),blood pressure is 160/98 mm Hg, pulse is 74/min, and respirations are 16/min. Neurologic examinationshows mild short-term memory loss and decreased sensationto
pinprick in the distal extremities. He has an ataxic gait. There are gouty tophi on the dorsal aspect of the left elbow. Laboratory studies show:
Hematocrit 33%
Mean corpuscularvolume 70 µm3
Serum
Urea nitrogen 17 mg/dl
Glucose 90 mg/dl
Creatinine 2 mg/dl • l
!i
Uric acid 14 mg/dl

Which of the following is the most appropriate next step in management?

0 A) Measurementof blood lead concentration


O B) Measurementof serum porphobilinogen concentration
0 C) CT scan of the abdomen
0 D) MRIof the brain
0 E) Carbidopa-levodopatherapy

'
35. A 4-week-oldnewbornis broughtto the physician because of increasing difficultyfeeding, poor weight gain, and excessive perspiration since birth. She was born at term after an
uncomplicatedpregnancyand delivery and weighed 3500 g (7 lb 11 oz.);initial examinationshowed no abnormalities.Family history is noncontributory.She currentlyweighs
3800 g (8 lb 6 oz). Her temperature is 37°C (98.6°F), pulse is 160/min, and respirations are 80/min and labored. A grade 2/6 holosystolicmurmur is heard over the lower left sternal
border.The lungsare clear to auscultation.A chest x-ray shows cardiomegalywith increased pulmonaryvascular markings.An ECG shows right ventricular hypertrophy.Which of the
following is the most likely explanationfor the absence of the murmuron initial examination? 1

0 A) High pulmonaryvascular resistance


0 B) High systemic vascular resistance
0 C) Low pulmonaryvascular resistance
0 D) Low systemic vascular resistance
0 E) Patentductusarteriosus
Th~
re~p·onseoptions for the next 2 item~'~re the::li~~e. Select one answer for each item in the set.
I'

For each patient with back pain, select the most likely diagnosis.

0 A) Herniated disc
I\ I
0 B) Lumbar spinal stenosis
O C) Metastatic cancer
0 D) Muscle strain II
I:
'
O E) Osteoporotic compression fracture I I
I
0 F) Sacroiliitis I II
O G) Spinal epidural abscess 11

0 H) Spondylolisthesis
I

36. A 57-year-old woman is brought to the physician 2 days after the sudden onset of severe low back pain;-1tljepain does not radiate to the lower extremities.The pafn1 began when she was
nence.She has a 10-yearhistoryof rheumatoidarthritis treated with
lifting her grandson. She does not have weakness or sensory loss in the legs and has had no uri~aryinc;:pr,iti
prednisone. Her temperature is 37°C (98.6°F), blood pressure 1s130/60 mm Hg, and pulse is 6~/min. liiiRamination shows deformities of the interphalangealjoints of the hands and
d1q~s not increase the pain. Musclestrengthand sensation are intact in the lower
exquisite tenderness to percussion over the lumbar spine. Bilateral straight-legraising to 80 deg:r.ees
11
1
extremities. Deep tendon reflexes are 2+ bilaterally. Babinski's sign is absent bilaterally. .
1•
1,1
I
ii I ,

. '
For each patient with back pain, select the most Iikely diagnosis.

O A) Herniated disc
O B) Lumbar spinal stenosis
O C) Metastatic cancer
0 D) Muscle strain
O E) Osteoporotic compression fracture
O F) Sacroiliitis
O G) Spinal epidural abscess
0 H) Spondylolisthesis

37. A previously healthy 32-year-old plumber comes to the physician because of a 3-week history of constant,dull, low back pain that does not radiate to the extremities.The pain began after
he unloaded heavy equipment from his van. It increases with activity and is ten1porarilyrelieved by bed rest and ibuprofen. Examinationshows tenderness to palpation over the lumbar
paraspinal region bilaterally. The pain increases with forward or lateral movementsof the spine. Musclestrengthand sensationare intact in the lower extremities. Bilateral straight-leg
raising to 80 degrees does not increase the pain. Deep tendon reflexes are 2+ bilaterally.Babinski's sign is absent bilaterally.
38. A 57-year-oldman is brought to the physician by his wife because his skin has appeared yellow for 3 weeks. Examinationshows jaundice and scleral icterus. His total serum bifirubin
concentrationis 8 mg/dl with a direct component of 6.2 mg/dl. A CT scan of the abdomen shows a large lesion in the head of the pancreas. When the results are initially discussed, the
patient says that he does not want to hear the report, and his wife agrees to abide by his wishes. Which of the following is the most appropriate course of action?

O A) Withholdthe results as the patient wishes


0 B) Contactthe patient's children to discuss the results
O C) Consultwith the hospital ethics comn1ittee
O D) Insist on telling the patient the results
O E) Refer him to another physician


39. An 80-year-old woman has had bleeding gums for 3 weeks. Her diet has consisted of onlytea and toast for 2 years. She appears thin and weak. Vital signs are normal.Her gums.are
hypertrophiedand exude blood with pressure. There are ecchymoses of the inner thighs and small hernorrhagesaroundthe hair follicles. The remainder of the examinationshows norma.l
findings. Supplementationwith which of the following vitamins is most likely to have preventedthis condition?

OA)A
0 B) B 12 (cyanocobalamin)
0 C)C
0 0)0
0 E)E
40. Four days after undergoing resection of an obstructing sigmoid colon cancer and colostomy,a 47-year-oldman has a temperatureof 39.3°C (102.8°F). During the procedure,a central
venous catheterwas inserted into the left subclavianvein. His pulse is 94/min, respirations are 20/min, and blood pressure is 128170mm Hg. The abdomen is soft and nontender.
Examinationof the colostomy shows no abnormalities. Blood culturesgrow Staphylococcus aureus. Which of the following is the most likely source of the bacteria?

O A) Centralvenous catheter
O B) Intra-abdominalabscess
0 C) Lungs
0 D) Operativewound
O E) Urinarytract
' l' I

41. One month after undergoing an uneventfulrenal transplant for chronic renal failure secondary to glomerulonephritis,a 38-year-oldwoman is hospitalized because of increJsed serum urea.
nitrogen and creati~i~ concentrations. Prior to transplantation,she ha? been receiving hemodialysisfor 3 yea~s.Cu~rentmedications include cycl_osporine
an~ pr~nison~. Examination·
1
shows no abnormaht1es.Over the past 48 hours, urine output has remained stable. Both renal biopsy and a radionuclide scan confirm the d1agnos1sof acute reJect1on.
Which of the
1
following is the most effective treatment? ' •
.I
l I
O A) Immediate discontinuation of cyclosporine
;1
O B) Increased dosage of corticosteroids '
O C) Diuresis and alkalinization of the urine
O D) Renal dialysis for 1-2 weeks
O E) Transplant nephrectomy
I

'
42. A 5-year-01d girl is brought to the physician because of temperatures to 40°C (104°F), tachypnea, and a nonproductive cough for 12 hours. Four days ago she was treated with an oral
antibiotic for suspected pneumococcal pneumonia. Examination shows diminished breath sounds over the lower right lung fields and dullness to percussion at the right costophrenic
angle. Which of the following is the most likely diagnosis?

O A) Bronchopleural fistula
0 B)Empyema
0 C) Lung abscess
0 D) Pleurodynia
O E) Pneumothorax
43. A previously healthy 52-year-old woman comes to the physician because she has had a large pimple on her right hand for 2 weeks that has failed to heal. She resides in southeastern
USA where she owns a nursery and garden shop. Examination shows a painless red papule on the hand with several nontender subcutaneous nodular lesions above it. Which of the
following is the most likely diagnosis?

O A) Blastomycosis
O B) Candidiasis
O C) Coccidioidomycosis
0 D) Histoplasmosis
0 E) Sporotrichosis
44. A study is conductedto compare the effectiveness of lorazepam and midazolam in providing sedation for childrenwho require mechanicalventilation.One hundredchildrenfrom the
ages of 2 to 12 years who require mechanical ventilation are randomlyassigned to receive a continuousinfusionof lorazepam or midazolam. Paralytic agents are administeredto 26 of
the 50 children in the lorazepam group and to 2 of the 50 children in the midazolam group to allow more synchronizedmechanicalventilation.A sedation score of 1 to 5 is used to assess
the amount of movementfor each child over the course of 3 days (with 1 indicating no movementand 5 indicating thrashingmovements).Resultsshow a sedation score of 1 for 92% of
the lorazepam group and 12% of the midazolam group. The authors concludethat lorazepamwas significantlymore effectivethan midazolam in providing complete sedation. Which of
the following features of this study raises the most concern regarding the validity of this conclusion?

O A) Confoundingvariables
O B) Lack of a control group
0 C) Sample size
O D) Statistical power
0 E) Univariate scale
45. A 67-year-old woman has been intubated for 1 week after undergoing a left tobectomy for lung cancer. She has chronic obstructive pulmonary disease. Her preoperative functional vi'tal
capacity was 40% of predicted. She is awake and alert Her blood pressure is 130/75 mm Hg, and pulse is 72/min. The ventilator setting.s are a synchronized intermittent mandatory
ventilation of 8/min, Fio 2 of 40%, and positive-end expiratory pressure of 5 cm H 20. Arterial blood gas analysis shows:
pH 7.42
PC02 47 mm Hg
Po 2 90 mm Hg
0 2 saturation 96%

Which of the following is the most appropriate next step in management?

O A) Antibiotic therapy O H) Incentive spirometry


O B) Bronchodilator therapy O I) Increase Fio 2
O C) Chest physiotherapy O J) Increase inotropes
OD) Decrease inotropes O K) Increase respiratory rate
0 E) Diuretic therapy O L) Placement of thoracostomy tube
O F) Fiberoptic bronchoscopy OM) Tracheostomy
0 G) Heparin therapy O N) Wean from the ventilator
46. A 37-year-old primigravid woman at 9 weeks' gestation is admitted to the hospital because of progressive nausea and vomiting. Her symptomsbegan at 6 weeks' gestation but have
worsened so that she can no longer keep food or fluids down. Her temperatureis 36.9°C (98.4°F), pulse is 105/min,respirations are 16/min, and blood pressure is 110/60 mm Hg.
Examinationshows a uterus consistent in size with a 9-week gestation. Serum electrolyteconcentrationsare within the referencerange. Urinalysisshows 3+ ketones. In addition to
beginning intravenoushydration,which of the following is the most appropriate initial step in management?

O A) High-proteindiet
0 B) Total parenteral nutrition
0 C) Antiemetic therapy
O D) Insertionof a nasogastric tube
0 E) Therapeutic abortion

You might also like

pFad - Phonifier reborn

Pfad - The Proxy pFad of © 2024 Garber Painting. All rights reserved.

Note: This service is not intended for secure transactions such as banking, social media, email, or purchasing. Use at your own risk. We assume no liability whatsoever for broken pages.


Alternative Proxies:

Alternative Proxy

pFad Proxy

pFad v3 Proxy

pFad v4 Proxy